270
500 Most Important Number Series Questions www.ibpsguide.com | estore.ibpsguide.com| www.sscexamguide.com For Free Online Mock Test for IBPS PO/Clerk 2017 – Visit: estore.ibpsguide.com 1 1 500 Most Important Number Series (Missing Number) Directions (Q. 1-10): find the missing number in the following number series? 1). 125, 152, 161, 169, 173, ?, 175 A) 180 B) 174 C) 176 D) 184 E) 173 2). 27, 54, 162, 648, ?, 19440 A) 3274 B) 3280 C) 3240 D) 3365 E) 3384 3). 161, 320, 636, ?, 2512, 4992 A) 1329 B) 1340 C) 1264 D) 1380 E) 1348 4). 4032, 4290, 4556, 4830, 5112, ? A) 5329 B) 5402

500 Most Important Number Series (Missing Number)

Embed Size (px)

Citation preview

500 Most Important Number Series Questions

www.ibpsguide.com | estore.ibpsguide.com| www.sscexamguide.com

For Free Online Mock Test for IBPS PO/Clerk 2017 – Visit: estore.ibpsguide.com

1 1

500 Most Important Number Series (Missing Number)

Directions (Q. 1-10): find the missing number in the following number series?

1). 125, 152, 161, 169, 173, ?, 175

A) 180

B) 174

C) 176

D) 184

E) 173

2). 27, 54, 162, 648, ?, 19440

A) 3274

B) 3280

C) 3240

D) 3365

E) 3384

3). 161, 320, 636, ?, 2512, 4992

A) 1329

B) 1340

C) 1264

D) 1380

E) 1348

4). 4032, 4290, 4556, 4830, 5112, ?

A) 5329

B) 5402

500 Most Important Number Series Questions

www.ibpsguide.com | estore.ibpsguide.com| www.sscexamguide.com

For Free Online Mock Test for IBPS PO/Clerk 2017 – Visit: estore.ibpsguide.com

2 2

C) 5302

D) 5529

E) 5482

5). 16, ?, 296, 1192, 4776, 19112

A) 90

B) 80

C) 100

D) 75

E) 72

6). 584, 559, 608, 527, ?, 479

A) 627

B) 648

C) 623

D) 608

E) 671

7). 36, 90, ?, 1417.5, 7796.25, 50675.625

A) 275

B) 517

C) 364

D) 315

E) 385

8). 28, 29, ?, 195, 796, 4005

A) 62

B) 68

C) 81

D) 83

E) 74

9). 15, 40, 70, 150, ?, 590

A) 310

500 Most Important Number Series Questions

www.ibpsguide.com | estore.ibpsguide.com| www.sscexamguide.com

For Free Online Mock Test for IBPS PO/Clerk 2017 – Visit: estore.ibpsguide.com

3 3

B) 280

C) 290

D) 300

E) 305

10). 205, 989, 364, 848, 487, ?

A) 748

B) 745

C) 643

D) 648

E) 743

Solution With Answer Key:

1). B)

The series is +33 , +32, +23, +22, +13, +12, …..

125 + (3)3 = 152 ; 152 + (3)2 = 161 ; 161 + (2)3 = 169; 169 + (2)2 = 173

173 + (1)3 = 174; 174 + (1)2 = 175

2). C)

The series is x2, x3, x4, x5, x6,….

27 × 2 = 54; 54×3 = 162; 162 × 4 = 648; 648 × 5 = 3240

3240 × 6 = 19440

3). C)

The series is x2 — 21, x2 — 22, x2 - 23, x2 - 24, ...

ie 161 x 2 — 2 = 320,

320 x 2 — 22 = 636,

636 x 2 — 23 = 1264,

1264 x 2 — 24 = 2512,

2512 x 2 — 25 = 4992,

4). B)

The series is 632 + 63 = 4032,

652 + 65 = 4290, 672 + 67 = 4556,

500 Most Important Number Series Questions

www.ibpsguide.com | estore.ibpsguide.com| www.sscexamguide.com

For Free Online Mock Test for IBPS PO/Clerk 2017 – Visit: estore.ibpsguide.com

4 4

692 + 69 = 4830, 712 + 71 = 5112,

732 + 73 = 5402, ...

5). E)

The series is

16 x 4 + 8 = 72

72 x 4 + 8 = 296

296 x 4 + 8 = 1192

1192 x 4 + 8 = 4776

4776 x 4 + 8 19112

6). B)

The series is —(5)2, +(7)2 - (9)2, +(11)2, —(13)2….

ie 584 — 52 = 559,

559 + 72 = 608,

608 - 92 = 527,

527 + 112 = 648,

648 — 132 = 479,

7). D)

The series is

36 x 2.5 = 90

90 x3.5 = 315

315 x4.5 = 1417.5

1417.5 x5.5 = 7796.25

7796.25 x6.5 = 5.675.625

8). A)

The series is x1 + 12, x2 + 22, x3 + 32, x4 + 42,

je 28 x 1 + 12= 29,

29 x 2 + 22 = 62,

62 x 3 + 32 = 195,

195 x 4 + 42 = 796,

796 x 5 + 52 = 4005,

500 Most Important Number Series Questions

www.ibpsguide.com | estore.ibpsguide.com| www.sscexamguide.com

For Free Online Mock Test for IBPS PO/Clerk 2017 – Visit: estore.ibpsguide.com

5 5

9). C)

The series is x2 + 10, x2 — 10, x2 + 10, x2 - 10, ...(repeated alternately)

ie 15 x 2 + 10 = 40,

40 x 2 — 10 = 70,

70 x 2 + 10 = 150,

150 x 2 — 10 = 290,

290 x 2 + 10 = 590,

10). E)

The series is 205 + (28)2 = 989,

989 - (25)2 = 364,

364 +(22)2 = 848.

848 — (19)2 = 487,

487 + (16)2 = 743,

Directions (Q. 11-20): What will come in place of question mark (?) in the given number

series?

11). 3, 72, 129, 173, 203, ?

A) 248

B) 227

C) 254

D) 252

E) 218

12). 4, 3, 5, 14, 55, ?

A) 285

B) 274

C) 319

D) 315

E) 265

13). 3, 3, 12, 108, ?, 43200

A) 2700

500 Most Important Number Series Questions

www.ibpsguide.com | estore.ibpsguide.com| www.sscexamguide.com

For Free Online Mock Test for IBPS PO/Clerk 2017 – Visit: estore.ibpsguide.com

6 6

B) 1728

C) 972

D) 432

E) 650

14). 24, 12, 12, 18, 36, ?

A) 92

B) 90

C) 94.5

D) 81.5

E) 108

15). 126, 64, 34, 20, ?, 12

A) 18

B) 12

C) 14

D) 16

E) 15

16). 455, 212, 131, 104, 95, ?

A) 84

B) 92

C) 45

D) 61

E) 49

17). 2, 3, 8, 27, 112, ?

A) 565

B) 650

C) 316

D) 290

E) 430

18). 45, 57, 67, 75, 81, ?

500 Most Important Number Series Questions

www.ibpsguide.com | estore.ibpsguide.com| www.sscexamguide.com

For Free Online Mock Test for IBPS PO/Clerk 2017 – Visit: estore.ibpsguide.com

7 7

A) 89

B) 85

C) 105

D) 91

E) 78

19). 36, 37, 33, 42, 26, ?

A) 51

B) 41

C) 61

D) 45

E) 49

20). 5, 7.5, 15, 37.5, ?, 393.75

A) 80

B) 112.5

C) 160

D) 48

E) 72

Solution:

11). E)

The series is

3 + 69=72 ; 72+57=129; 129+44=173; 173+30=203; 203+15=218

69 – 57 = 12; 57 – 44 = 13; 44-30 = 14; 30-15 = 15

12). B)

The series is

4 × 1 – 1 = 3

3 × 2 – 1 = 5

5 × 3 – 1 = 14

14 × 4 – 1 = 55

55 × 5 – 1 = 274

500 Most Important Number Series Questions

www.ibpsguide.com | estore.ibpsguide.com| www.sscexamguide.com

For Free Online Mock Test for IBPS PO/Clerk 2017 – Visit: estore.ibpsguide.com

8 8

13). B)

The series is

3 × (1)2 = 3; 3×(2)2 = 12; 12×(3)2 = 108; 108×(4)2 = 1728; 1728×(5)2 = 43200

14). B)

The series is

24 ×0.5 = 12; 12×1 = 12; 12×1.5 = 18, 18×2=36; 36×2.5=90

15). C)

The series is

126 × (1/2)+1 =64

64 × (1/2) + 2 = 34

34 × (1/2) + 3 = 20

20 × (1/2) + 4 = 14

14 × (1/2) + 5 = 12

16). B)

The series is

455 – 212 = 243; 212 – 131 = 81; 131 – 104 = 9; 95 – 92 = 3

243 ÷ 3 = 81; 81 ÷ 3 = 27; 27÷3 = 9; 9÷3 = 3

17). A)

The series is

2 × 1 + 1 = 3

3 × 2 + 2 = 8

8 × 3 + 3 = 27

27 × 4 + 4 = 112

112 × 5 + 5 = 565

18). B)

The series is

45 + 12 = 57; 57 + 10 = 67; 67 + 8 = 75; 75 + 6 = 81; 81 + 4 = 85

19). A)

500 Most Important Number Series Questions

www.ibpsguide.com | estore.ibpsguide.com| www.sscexamguide.com

For Free Online Mock Test for IBPS PO/Clerk 2017 – Visit: estore.ibpsguide.com

9 9

The Series

36 + (1)2 = 37

37 - (2)2 = 33

33 + (3)2 = 42

42 – (4)2 = 26

26 + (5)2 = 51

20). B)

The series is

5 × 1.5 = 7.5; 7.5 × 2 = 15; 15 × 2.5 = 37.5; 37.5 × 3 = 112.5; 112.5×3.5 = 393.75

Directions (Q. 21-30): What will come in place of question mark (?) in the given number

series?

21). 10, 11, 15, 24, 40, ?

A) 90

B) 87

C) 114

D) 95

E) 65

22). 25, 35, 49, 67, 89, ?

A) 24

B) 81

C) 115

D) 107

E) 93

23). 11, 9, 15, 41, 159, ?

A) 740

B) 607

C) 751

D) 789

E) 785

500 Most Important Number Series Questions

www.ibpsguide.com | estore.ibpsguide.com| www.sscexamguide.com

For Free Online Mock Test for IBPS PO/Clerk 2017 – Visit: estore.ibpsguide.com

10 10

24). 8.5, 10.5, 16.5, 28.5, 48.5, ?

A) 57.5

B) 78.5

C) 93.5

D) 64.5

E) 85.5

25). 18, 9, 9, 18, 72, ?

A) 460

B) 372

C) 576

D) 484

E) 380

26). 68, ?, 77, 104, 168, 293

A) 69

B) 70

C) 68

D) 74

E) None of these

27). 18, 19.7, 16.3, 23.1, 9.5, ?

A) 36.5

B) 36.8

C) 36.7

D) 36.9

E) None of these

28). 2, 5, 12, 27, 58, ?

A) 122

B) 121

C) 123

D) 120

E) None of these

500 Most Important Number Series Questions

www.ibpsguide.com | estore.ibpsguide.com| www.sscexamguide.com

For Free Online Mock Test for IBPS PO/Clerk 2017 – Visit: estore.ibpsguide.com

11 11

29). 16, 8, 12, 30, ?, 472.5

A) 104

B) 103

C) 106

D) 105

E) None of these

30). 334, ?, 226, 217, 214, 213

A) 253

B) 251

C) 252

D) 254

E) None of these

Explanation With Answer Key:

21).

Answer: E)

22).

Answer: C)

23).

Answer: D)

500 Most Important Number Series Questions

www.ibpsguide.com | estore.ibpsguide.com| www.sscexamguide.com

For Free Online Mock Test for IBPS PO/Clerk 2017 – Visit: estore.ibpsguide.com

13 13

Answer: D)

30).

Answer: A)

Directions (Q. 31-40): What will some in place of question mark (?) in the given number

series?

31). 5, 9, 25, 89, ?, 1369

A) 343

B) 355

C) 349

D) 341

E) 345

32). 6, 17, 50, 149, ?, 1337

A) 454

B) 446

C) 442

D) 452

E) 432

33). 61, 62, 54, 81, 17, ?

A) 142

B) 145

C) 138

D) 144

E) 140

34). 6, 11, 18, 29, 46, ?

A) 60

B) 75

500 Most Important Number Series Questions

www.ibpsguide.com | estore.ibpsguide.com| www.sscexamguide.com

For Free Online Mock Test for IBPS PO/Clerk 2017 – Visit: estore.ibpsguide.com

14 14

C) 69

D) 71

E) 79

35). 26, 13, 13, 19.5, ?, 97.5

A) 39

B) 45

C) 48

D) 41

E) 35

36). 82, 93, ?, 148, 192, 247

A) 124

B) 122

C) 117

D) 115

E) 120

37). 163, ?, 188, 216, 253, 298

A) 175

B) 168

C) 171

D) 170

E) 169

38). 224, 228, 219, 235, ?, 246

A) 210

B) 209

C) 220

D) 225

E) 215

39). 52, 102, 303, 1208, ?, 36204

A) 6240

500 Most Important Number Series Questions

www.ibpsguide.com | estore.ibpsguide.com| www.sscexamguide.com

For Free Online Mock Test for IBPS PO/Clerk 2017 – Visit: estore.ibpsguide.com

15 15

B) 6040

C) 6035

D) 6020

E) 6340

40). 121, 128, 144, 168, 199, ?

A) 237

B) 236

C) 226

D) 216

E) 246

Explanation With Answer Key:

31). E)

The series is +4, +16, +64, +256, +1042,…..

Ie 5 + 4 = 9; 9 + 16 = 25, 25 + 64 = 89

89 + 256 = 345, 345+1024 = 1369

32). B)

The series is ×3 – 1, (repeateD)

Ie 6 × 3 – 1 = 17, 17 × 3 – 1 = 50, 50 × 3 – 1 = 149 , 149 × 3 – 1 = 446, 446 × 3 – 1 = 1337

33). A)

The Series +(1)3 , -(2)3, +(3)3, …..

Ie 61 + 13 = 62, 62 – 23 = 54, 54 + 33 = 81

81 – 43 = 17, 17 + 53 = 142

34). D)

6 + 5 = 11, 11 +7 = 18, 18 + 11 = 29, 29 + 17 = 46, 46 + 25 = 71

35). A)

The series is ×0.5, ×1, ×1.5, ×2, ×2.5,…

Ie 26 × 0.5 = 13, 13 × 1 = 13, 13 × 1.5 = 19.5, 19.5 × 2 = 39, 39 × 2.5 = 97.5, …..

500 Most Important Number Series Questions

www.ibpsguide.com | estore.ibpsguide.com| www.sscexamguide.com

For Free Online Mock Test for IBPS PO/Clerk 2017 – Visit: estore.ibpsguide.com

16 16

36). D)

The series is

82 + 11 = 93

93 + 22 = 115

115 + 33 = 148

148 + 44 = 192

192 + 55 = 247

37). D)

The Series is

163 + 7 = 170 ; 170 + 18 = 188 ; 188 + 28 = 216; 216 + 37 = 253; 253 + 45 = 298

7 + 11 = 18; 18 + 10 = 28; 28 + 9 = 37; 37 + 8 = 45

38). A)

The series is

224 + 22 = 228

228 – 32 = 219

219 + 42 = 235

235 – 52 = 210

210 + 62 = 246

39). C)

The series is 52 × 2 – 2 = 102

102 × 3 – 3 = 303

303 × 4 – 4 = 1208

1208 × 5 – 5 = 6035

6035 × 6 – 6 = 36204

40). B)

The series is

121 + 7 = 128; 128 + 16 = 144

144 + 24 = 168 ; 168 + 31 = 199; 199 + 37 = 236

7 + 9 = 16; 16 + 8 = 24; 24 + 7 = 31; 31 + 6 = 37

500 Most Important Number Series Questions

www.ibpsguide.com | estore.ibpsguide.com| www.sscexamguide.com

For Free Online Mock Test for IBPS PO/Clerk 2017 – Visit: estore.ibpsguide.com

17 17

Directions (Q. 41-50): What should come in place of question mark in the following number

series?

41). 5, ?, 41, 191, 941, 4691

A) 12

B) 15

C) 18

D) 21

E) 11

42). 5000, 2508, 1262, 639, ?, 171.75

A) 329.6

B) 339

C) 327.5

D) 326

E) 335.5

43). 9, 5, ?, 10.5, 23, 60

A) 8

B) 7.5

C) 9

D) 6

E) 5.5

44). 551, 600, 636, 661, 677 ?

A) 686

B) 676

C) 689

D) 645

E) 690

45). 5, 2.75, ?, 7.87, 19.75, 55.625

A) 7.5

B) 7.875

500 Most Important Number Series Questions

www.ibpsguide.com | estore.ibpsguide.com| www.sscexamguide.com

For Free Online Mock Test for IBPS PO/Clerk 2017 – Visit: estore.ibpsguide.com

18 18

C) 3.75

D) 7.1

E) 8.5

46). 16, 18, 27, 55, 120, ?

A) 232

B) 246

C) 254

D) 242

E) 248

47). 13, 10.8, 5.4, 8.2, 17.8, ?

A) 5.8

B) 5.6

C) 6.2

D) 7.2

E) 6.8

48). 4, 19, 64, 139, ?, 379

A) 254

B) 233

C) 236

D) 244

E) 262

49). 7, 17, 37, 77, ?, 317

A) 167

B) 160

C) 157

D) 147

E) 151

50). 12, 8, 10, 17, 36, ?

A) 92

500 Most Important Number Series Questions

www.ibpsguide.com | estore.ibpsguide.com| www.sscexamguide.com

For Free Online Mock Test for IBPS PO/Clerk 2017 – Visit: estore.ibpsguide.com

19 19

B) 84

C) 76

D) 98

E) 104

Explanation with Answers Key:

41). E)

The series is

5 + [(6/5)×5] = 11, 11 + (6×5) = 41, 41 + (30×5) = 191

191 + (150×5) = 941, 941 + (750×5) = 4691

42). C)

The series is 5000 ÷ 2 + 8 = 2508, 2508 ÷ 2 + 8 = 1262, 1262 ÷ 2 + 8 = 639, 639 ÷ 2 + 8 =

327.5, 327.5 ÷ 2 + 8 = 171.75,….

43). D)

The series is 9 x 0.5 + 0.5 = 5, 5 × 1 + 1 = 6, 6 x 1.5 + 1.5 = 10.5, 10.5 × 2 + 2 = 23, 23 x 2.5 +

2.5 = 60

44). A)

The series is +72, +62, +52, ... .

551 +49 = 600, 6

00 + 36= 636, 636 + 25 = 661, 661+16 = 677, 677+9 = 686

45). C)

The series is

5 x 0.5 + 0.25 x 12 = 2.75,

2.75 x 1 + 0.25 x 22 = 3.75,

3.75 x 1.5 + 0.25 x 32 = 7.875,

7.875 x 2 + 0.25 x 42 = 19.75,

19.75 x 2.5 + 0.25 x 52 = 55.625, ...

46). B)

500 Most Important Number Series Questions

www.ibpsguide.com | estore.ibpsguide.com| www.sscexamguide.com

For Free Online Mock Test for IBPS PO/Clerk 2017 – Visit: estore.ibpsguide.com

20 20

The series is +13 + 1, +23 + 1, +33 + 1, +43 + 1, +53 + 1, ...

ie 18 + 23 + 1 = 27,

27 + 33 + 1 = 55,

55 + 43 + 1 = 120,

120 + 53 + 1 = 246, ...

47). B)

The series is a combination of two series

48). D)

The series is +15 x 1, +15 x 3, +15 x 5, +15 x 7, +15 x 9, ...

ie 4 + 15 x 1 = 19,

19 + 15 x 3 = 64,

64 + 15 x 5 = 139,

139 + 15 x 7 = 244,

244 + 15 x = 379, ...

49). C)

The series is +10, +20, +40, +80, +160,

ie 7 + 10 = 17,

17 + 20 = 37,

37 + 40 = 77,

77 + 80 = 157,

157 160 = 317, ...

50). A)

The series is x0.5 + 2, x 1 + 2, x1.5 + 2, x2 + 2, ...

500 Most Important Number Series Questions

www.ibpsguide.com | estore.ibpsguide.com| www.sscexamguide.com

For Free Online Mock Test for IBPS PO/Clerk 2017 – Visit: estore.ibpsguide.com

21 21

ie 12 x 0.5 + 2 = 8,

8 x 1 + 2 = 10,

10 x 1.5 + 2 = 17,

17 x 2 + 2 = 36,

36 x 2.5 +2 = 92, ...

Directions (Q. 51-60): Find out the missing number in place of question mark (?) in the

following number series.

51). 3, 7, 29, 143, 779, ?

A) 4997

B) 4649

C) 4847

D) 4799

E) 4999

52). 4, 5, ?, 88, 620, 5585

A) 16

B) 18

C) 17

D) 20

E) 22

53). 11, 20, 42, ?, 166, 330

A) 72

B) 66

C) 86

D) 80

E) 82

54). 128, 64, 96, 240, ?, 3780

A) 740

B) 840

500 Most Important Number Series Questions

www.ibpsguide.com | estore.ibpsguide.com| www.sscexamguide.com

For Free Online Mock Test for IBPS PO/Clerk 2017 – Visit: estore.ibpsguide.com

22 22

C) 850

D) 845

E) 745

55). 8484, ?, 8486, 8495, 8488, 8493

A) 8485

B) 8584

C) 8684

D) 8497

E) 8496

56). 68, 132, 268, ?, 1068, 2132

A) 394

B) 464

C) 325

D) 532

E) 484

57). 23, 12, 13, 21, ?, 112.5

A) 33

B) 44

C) 55

D) 66

E) 65

58). 8, 10, 18, 44, 124, ?

A) 366

B) 263

C) 194

D) 276

E) 286

59). 4, 4, 21, 68, ?, 2154

A) 435

500 Most Important Number Series Questions

www.ibpsguide.com | estore.ibpsguide.com| www.sscexamguide.com

For Free Online Mock Test for IBPS PO/Clerk 2017 – Visit: estore.ibpsguide.com

23 23

B) 365

C) 462

D) 572

E) 425

60). 7, 20, 59, 176, ?, 1580

A) 527

B) 526

C) 627

D) 562

E) 270

Explanation:

51). D)

The series is x2 + 13, x3 + 23, x4 + 33, x5 + 43, ...

ie 3 x 2 +13 = 7,

7 x 3 + 23 = 29,

29 x 4 + 33 = 143,

143 x 5 +43 = 779,

779 x 6 + 53 = 4799,

52). C)

The series is x1 + 1, x3 + 2, ×5 + 3, x7 + 4, x9 + 5, x11 + 6,

Ie 4 x 1+1 = 5,

5 x3+2=17,

17 x 5 + 3 = 88,

88 x 7 + 4 = 620,

620 x 9 + 5 = 5585,

53). E)

The series is x2 - 2, x2 + 2, (Alternately repeateD)

500 Most Important Number Series Questions

www.ibpsguide.com | estore.ibpsguide.com| www.sscexamguide.com

For Free Online Mock Test for IBPS PO/Clerk 2017 – Visit: estore.ibpsguide.com

24 24

54). B)

The series is x0.5, x1.5, x2.5, x3.5, ...

55). D)

The series is +13, —11, +9, —7, +5, ... +13

56). D)

The series is x2 - 4, x2 + 4, (repeateD)

57). B)

The series is ×1/2 + 0.5, ×1+1, ×1.5+1.5…..

58). A)

The series is

59). B)

500 Most Important Number Series Questions

www.ibpsguide.com | estore.ibpsguide.com| www.sscexamguide.com

For Free Online Mock Test for IBPS PO/Clerk 2017 – Visit: estore.ibpsguide.com

25 25

The Series is

60). A)

The series is ×3 – 1, ×3 – 1 (repeateD)

7 × 3 = 21 – 1 = 20

20 × 3 = 60 – 1 = 59

59 × 3 = 177 – 1 = 176

176 × 3 = 528 – 1 = 527

527 × 3 = 1581 – 1 = 1580

Directions (Q. 61-70): what should come in the place of question mark (?) in the given

number series?

61). 14, 8, 7, 11.5, 22, ?

A) 54

B) 64

C) 62

D) 58

E) 56

62). 8, 14, 25, 46, 82, ?

A) 132

B) 130

C) 138

D) 128

E) 142

63). 13, 14, 30, 93, ?, 1885

A) 358

B) 336

C) 364

D) 376

500 Most Important Number Series Questions

www.ibpsguide.com | estore.ibpsguide.com| www.sscexamguide.com

For Free Online Mock Test for IBPS PO/Clerk 2017 – Visit: estore.ibpsguide.com

26 26

E) 380

64). 65, 70, 63, 74, 61, ?

A) 78

B) 58

C) 72

D) 46

E) 68

65). 9, 11, 16, 33, 98, ?

A) 350

B) 355

C) 360

D) 365

E) 370

66). 4, 11, 26, 57, 120, ?

A) 247

B) 251

C) 187

D) 236

E) 215

67). 50, 81, 121, 172, 236, ?

A) 392

B) 309

C) 361

D) 379

E) 315

68). 84, 82.4, 85.6, 80.8, ?, 79.2

A) 72.8

B) 86.4

C) 88.8

500 Most Important Number Series Questions

www.ibpsguide.com | estore.ibpsguide.com| www.sscexamguide.com

For Free Online Mock Test for IBPS PO/Clerk 2017 – Visit: estore.ibpsguide.com

27 27

D) 87.2

E) 85.7

69). 900, 448, 220, 104, 44, ?

A) 8

B) 16

C) 24

D) 42

E) 12

70). 60, ?, 120, 80, 40, 16

A) 90

B) 110

C) 120

D) 70

E) 100

Explanation:

61). E)

62). C)

63). D)

64). A)

500 Most Important Number Series Questions

www.ibpsguide.com | estore.ibpsguide.com| www.sscexamguide.com

For Free Online Mock Test for IBPS PO/Clerk 2017 – Visit: estore.ibpsguide.com

29 29

Directions (Q. 71-80): what should come in place of question mark (?) in the following

questions?

71). 2, 2, 4, 16, ?, 2048

A) 138

B) 128

C) 256

D) 124

E) 40

72). 2, ?, 27, 113, 561, 3369

A) 11

B) 13

C) 9

D) 15

E) 18

73). 4, 14, ?, 149, 295, 293

A) 40

B) 45

C) 50

D) 51

E) 48

74). 5, 12, 38, ?, 772, 4634

A) 150

B) 158

C) 160

D) 162

E) 154

75). 37, ?, 222, 666, 1332, 3996

A) 120

B) 110

C) 111

500 Most Important Number Series Questions

www.ibpsguide.com | estore.ibpsguide.com| www.sscexamguide.com

For Free Online Mock Test for IBPS PO/Clerk 2017 – Visit: estore.ibpsguide.com

30 30

D) 119

E) 118

76). 14, 12.4, 15.6, ?, 17.2, 9.2

A) 10.8

B) 12

C) 12.8

D) 14.6

E) 18.6

77). 28, ?, 168, 672, 3360, 20160

A) 60

B) 84

C) 72

D) 56

E) 40

78). ?, 256, 3375, 196, 2197, 144

A) 4319

B) 4923

C) 4913

D) 4139

E) 4096

79). 987, ?, 945, 1071, 903, 1113

A) 1040

B) 1039

C) 1030

D) 1029

E) 1027

80). 480, 240, 160, ?, 96, 80

A) 100

B) 120

500 Most Important Number Series Questions

www.ibpsguide.com | estore.ibpsguide.com| www.sscexamguide.com

For Free Online Mock Test for IBPS PO/Clerk 2017 – Visit: estore.ibpsguide.com

31 31

C) 136

D) 140

E) 135

Solution:

71). B) The series is

72). A) The series is 2 x 2 + 7 = 11,

11 x 3 - 6 = 27, 27 x 4 + 5 = 113, 113 x 5 - 4 = 561, 561 x 6 + 3 = 3369,

73). D) The series is 4 x 5 - 6 = 14,

14 x 4 – 5 = 51, 51 x 3 - 4 = 149, 149 x 2 — 3 = 295, 295 x 1 — 2 = 293, ...

74). E) The series is 5 x 2 + 2 = 12,

12 x 3 + 2 = 38, 38 x 4 + 2 = 154, 154 x 5 + 2 = 772, 772 x 6 + 2 = 4634, ...

75). C) The series is

76). A) The series is 14 +1.6 x – 1 = 12.4, 12.4 + 1.6 × 2 = 15.6, 15.6 + 1.6 x -3 = 10.8, 10.8 +

1.6 x 4 = 17.2, 17.2 + 1.6 × -5 = 9.2

77). D) The series is 28 x 2 = 56, 56 x3 =168, 168 x 4 = 672, 672 x 5 = 3360,

3360 x 6 = 20160,

78). C) The series is (17)3, (16)2, (15)3, (14)2, (13)3, (12)2,…..

ie 4913, 256, 3375, 196, 2197, 144

79). D) The series is

500 Most Important Number Series Questions

www.ibpsguide.com | estore.ibpsguide.com| www.sscexamguide.com

For Free Online Mock Test for IBPS PO/Clerk 2017 – Visit: estore.ibpsguide.com

32 32

80). B) The series is

Directions (Q. 81-85): What will come in place of question mark (?) in the given number

series?

81). 28 39 63 102 158 ?

A) 232

B) 242

C) 233

D) 244

E) None of these

82). 7 16 141 190 919 ?

A) 1029

B) 1019

C) 1020

D) 1030

E) None of these

83). 12 17 32 57 92 ?

A) 198

B) 195

C) 137

D) 205

E) None of these

84). 19 25 45 87 159 ?

A) 254

B) 279

C) 284

500 Most Important Number Series Questions

www.ibpsguide.com | estore.ibpsguide.com| www.sscexamguide.com

For Free Online Mock Test for IBPS PO/Clerk 2017 – Visit: estore.ibpsguide.com

33 33

D) 269

E) None of these

85). 83 124 206 370 698 ?

A) 1344

B) 1324

C) 1364

D) 1334

E) None of these

Directions (Q. 86-90): In each of these questions a number series is given. In each series

only one number is missing. Find out the number.

86). 78 421 -91 ? -362 969

A) 645

B) 635

C) 640

D) 648

E) 638

87). 584 662 1246 1908 ? 5062

A) 3155

B) 3164

C) 3154

D) 3162

E) 4654

88). 51 74 143 350 971 ?

A) 2818

B) 2318

C) 2384

D) 2834

E) 2438

89). 740 ? 181 86.5 38.25 13.125

500 Most Important Number Series Questions

www.ibpsguide.com | estore.ibpsguide.com| www.sscexamguide.com

For Free Online Mock Test for IBPS PO/Clerk 2017 – Visit: estore.ibpsguide.com

34 34

A) 364

B) 368

C) 366

D) 378

E) 374

90). 4 25 ? 511 1537 3079

A) 125

B) 129

C) 137

D) 127

E) 139

Solution:

81). C) The series is +(10 x 1 + 12), +(10 x 2 + 22), +(10 x 3 + 32), +(10 x 4 + 42),

+(10 x 5 + 52),…..

ie 28 + 10 x 1 + 12 = 39,

39 + (10 x 2 + 22) = 63,

63 + (10 x 3 + 32) = 102,

102 + (10 x 4 + 42) = 158,

158 + (10 x 5 + 52) = 233, ...

82). E) The series +32, +53, +72, +93, +112, +133, ...

ie 7 + 32= 16, 16 + 53= 141, 141 + 72= 190, 190 + 93 = 919, 919 + 112 = 1040, ...

83). C) The series is +(5 x 1), +(5 x 3), +(5 x 5), +(5 x 7), +(5 x 9),….

ie 12 + (5 x 1) = 17, 17 + (5 x 3) = 32, 32 + (5 x 5) = 57, 57 + (5 x 7) = 92, 92 + (5 x 9) = 137,

84). D) The series is +22 + 2, +42 + 4, +62 + 6, +82 + 8, +102 + 10, ...

ie 19 + 22 + 2 = 25, 25 + 42 + 4 = 45, 45 + 62 + 6 = 87, 87 + 82 + 8 = 159, 159 + 102 + 10 = 269,

...

85). E) The series is +41, +82, +164, +328, +656,….

ie 83 + 41 = 124, 124 + 82 = 206, 206 + 164 = 370, 370 + 328 = 698, 698 + 656 =1354,…..

86). E) The series is 78 + 73 = 421

421 – 83 = -91, -91 + 93 = 638, 638 -103 = -362, -362 + 113 = 969,

87). C) The series is

500 Most Important Number Series Questions

www.ibpsguide.com | estore.ibpsguide.com| www.sscexamguide.com

For Free Online Mock Test for IBPS PO/Clerk 2017 – Visit: estore.ibpsguide.com

35 35

88). D) The series is

89). B) The series is 740 x (1/2) - 2 =368,

368 x (1/2) -3 =181, 181x (1/2) - 4 =-86.5, 86.5 x (1/2) = 38.25, 38.25 x (1/2) – 6= 13.125

90). D) The series is x6 + 1, x5 + 2, x4 + 3, ×3 + 4, x2 + 5,... ...

? = 25 x 5 +2 = 127

Directions (Q. 91-100): What should come in place of question mark (?) in the following

number series?

91). 6 -3 24 -57 186 ?

A) -582

B) 475

C) 627

D) 545

E) -543

92). 4 8 14 23 36.5 ?

A) 124

B) 91.75

C) 108.5

D) 84.5

E) 56.75

93). 9 5 6 10.5 23 ?

A) 60

B) 16

C) 18.5

500 Most Important Number Series Questions

www.ibpsguide.com | estore.ibpsguide.com| www.sscexamguide.com

For Free Online Mock Test for IBPS PO/Clerk 2017 – Visit: estore.ibpsguide.com

36 36

D) 42.5

E) 32.5

94).1 19 55 112 193 ?

A) 301

B) 336

C) 330

D) 252

E) 636

95). 3 7 15 31 63 ?

A) 95

B) 125

C) 128

D) 127

E) 98

96). 13 27 56 ? 234 473

A) 110

B) 112

C) 115

D) 120

E) 80

97). 48 144 288 864 1728 ?

A) 5164

B) 4185

C) 5284

D) 5184

E) 5418

98). 46656 1156 32768 ? 21952 676

A) 861

B) 900

500 Most Important Number Series Questions

www.ibpsguide.com | estore.ibpsguide.com| www.sscexamguide.com

For Free Online Mock Test for IBPS PO/Clerk 2017 – Visit: estore.ibpsguide.com

37 37

C) 2700

D) 961

E) 841

99). 4 14 32 60 ? 154

A) 90

B) 80

C) 95

D) 100

E) 108

100). 56 ? 80 102 128 161

A) 66

B) 67

C) 77

D) 75

E) 87

Solution:

91). The series is x-3 +15, x-3 +15, (repeateD)

ie 6 x -3 + 15 = -3, -3 x -3 + 15 24,

24 x -3 + 15 = -57, -57 x -3 + 15 = 186, 186 x -3 + 15 = -543,

Answer: E)

92). The series is x1.5 + 2 (repeateD)

ie 4 x 1.5 +2 = 8, 8 x 1.5 + 2 = 14, 14x 1.5 + 2 = 23, 23 x 1.5 + 2 = 36.5, 36.5 x 1.5 + 2 = 56.75,

Answer: E)

93). The series is x0.5 + 0.5, x1 + 1, ×1.5+ 1.5, x2 + 2, x2.5 + 2.5,

ie 9 x 0.5 + 0.5 = 5, 5 x 1 + 1 = 6, 6 x 1.5 + 1.5 = 10.5, 10.5 x 2 + 2 = 23, 23 x 2.5 + 2.5 = 60.

Answer: A)

94). The series is

500 Most Important Number Series Questions

www.ibpsguide.com | estore.ibpsguide.com| www.sscexamguide.com

For Free Online Mock Test for IBPS PO/Clerk 2017 – Visit: estore.ibpsguide.com

38 38

Answer: A)

95). The series is x2 + 1 (repeateD)

ie 3 × 2 + 1 = 7, 7 x 2 + 1 =15, 15 × 2 + 1 = 31, 31 x 2 + 1= 63, 63 x 2 +1 = 127,…

Answer: D)

96). The series is 13 x 2 + 1 = 27, 27 x 2 + 2 = 56, 56 x 2 + 3 = 115, 115 x 2 + 4 = 234, 234 x 2 +

5 = 473, ...

Answer: C)

97). The series is x3, x2 (alternatively)

Answer: D)

98). The series is (36)3, (34)2, (32)3, (30)2, (28)3, (26)2,

ie (36)3 = 46656, (34)2 = 1156, (32)3 = 32768, (30)2 = 900, (28)3 = 21952, (26)2 = 676,…..

Answer: B)

99). The series is +32 + 1, +42 + 2, +52 + 3, +62 + 4, +72 + 5,

le 4 + 32 + 1 = 14, 14 + 42+ 2 = 32, 32 + 52 + 3 = 60, 60 + 62 + 4 = 100, 100 + 72 +5 = 154,

Answer: D)

100). The series is +11, +13, +22, +26, +33, +39, ... ie 56 + 11 = 67, 67 + 13 = 80, 80 + 22 = 102,

102 + 26 = 128, 128 + 33 = 161, 161 + 39 = 200,

Answer: B)

500 Most Important Number Series Questions

www.ibpsguide.com | estore.ibpsguide.com| www.sscexamguide.com

For Free Online Mock Test for IBPS PO/Clerk 2017 – Visit: estore.ibpsguide.com

39 39

Direction (Q. 101-105): What value should come in the place of question mark (?) in the

following number series:

101).3, 11, 54, 339, 2732,?

A) 21650

B) 27345

C) 22852

D) 27356

E) 27375

102). 358, 320, 284, 250,?

A) 236

B) 258

C) 264

D) 218

E) 278

103). 8, 22, 80, 364, ?

A) 4805

B) 2010

C) 2007

D) 2560

E) 2700

104). 7, 22, 118, 1107, ?

A) 24334

B) 17756

C) 17779

D) 17772

E) 28986

105). 8, 37, 104, 233, 454, ?

A) 338

B) 837

C) 823

500 Most Important Number Series Questions

www.ibpsguide.com | estore.ibpsguide.com| www.sscexamguide.com

For Free Online Mock Test for IBPS PO/Clerk 2017 – Visit: estore.ibpsguide.com

40 40

D) 934

E) 803

Direction (Q. 106-110): What will come in place of the question mark :

106). 15, 16, 10, 33, ?, 66.25

A) 14.50

B) 12.50

C) 12.25

D) 22.25

E) None of these

107). 2, 9, 82, ?, 32801

A) 13130

B) 13456

C) 1313

D) 2675

E) None of these

108). 32, 321, 682, 1123, ?, 2277

A) 1702

B) 1652

C) 1834

D) 1567

E) None of these

109). 84, ?, 78, 109, 72, 113, 70

A) 109

B) 124

C) 144

D) 107

E) None of these

110). 11, 15, ?, 71, 333

A) 27

500 Most Important Number Series Questions

www.ibpsguide.com | estore.ibpsguide.com| www.sscexamguide.com

For Free Online Mock Test for IBPS PO/Clerk 2017 – Visit: estore.ibpsguide.com

41 41

B) 36

C) 29

D) 25

E) None of these

Solution:

Directions (Q. 101-105):

101). B) Series is x2+5, x4+10, x6+15, x8+20, x10+25

102). D) Series is 19^2-3,18^2-4, 17^2-5, 16^2-6, 15^2-7

103). C) Series is x2.5+2, x3.5+3, x4.5+4, x5.5+5

104). D) Series is x1^2+15, x2^2+30, x3^2+45, x4^2+60

105). E) Series is Difference between no. is, 3^3+2, 4^3+3, 5^3+4, 6^3+5, 7^3+6

Directions(Q. 106-110)

106). C) x1+1, ÷2+2, x3+3, ÷4+4, x5+5

107). C) x2^2-2+3, x3^2-3+4, x4^2-4+5, x5^2-5+6

108). B) +17^2, +19^2, +21^2, +23^2, +25^2

109). D) +23, -29, +31, -37, +41, -43

110).D) Diff of diff is multiples of 6

Direction (Q. 111-115): What value should come in the place of question mark (?) in the

following number series :

111). 6, 11, 32, 111, 464, ?

A) 2165

B) 2205

C) 2285

D) 2345

E) 2375

112). 3, 14, 39, 84, 155, ?

A) 236

B) 258

C) 264

D) 272

500 Most Important Number Series Questions

www.ibpsguide.com | estore.ibpsguide.com| www.sscexamguide.com

For Free Online Mock Test for IBPS PO/Clerk 2017 – Visit: estore.ibpsguide.com

42 42

E) 278

113). 8, 12, 24, 60, 180, ?

A) 480

B) 510

C) 630

D) 720

E) 780

114). 168, 171, 178, 191, 212, ?

A) 243

B) 247

C) 251

D) 254

E) 257

115). 4, 7, 10, 11, 22, 19, 46, ?

A) 33

B) 35

C) 37

D) 39

E) 41

Direction (Q. 116-120): What will come in place of the question mark :

116). 124, 228, 436, ?, 1684, 3348

A) 944

B) 852

C) 872

D) 444

E) None of these

117). 1, 3, 24, 360, 8640, ?, 14515200

A) 282400

B) 292400

500 Most Important Number Series Questions

www.ibpsguide.com | estore.ibpsguide.com| www.sscexamguide.com

For Free Online Mock Test for IBPS PO/Clerk 2017 – Visit: estore.ibpsguide.com

43 43

C) 302400

D) 25426

E) None of these

118). 10, 17, 48, 165, ?, 3475, 20892

A) 688

B) 712

C) 848

D) 918

E) None of these

119). 15, 17, 38, 120, 488, ?, 14712

A) 2450

B) 2650

C) 2850

D) 2950

E) None of these

120). 1, 6, 19, ?, 85, 146, 231

A) 46

B) 44

C) 48

D) 65

E) None of these

Solution:

111). D) Series is ×1+5, × 2 + 10, ×3 + 15, × 4 + 20

112). B) Series is 1 + 12 + 13 , 2 + 22 +23 , 3 + 32 + 33

113). C) Series is × 0.5 + 8, ×1.0 + 12, ×1.5 + 24

114). A) Series is + 12 + 2, + 22 + 3, + 32 + 4......

115). B) Series 1 , 4+6 = 10, 10 + 12 = 22, 22 + 24 = 26

Series 2 7, 7 + 4 = 11, 11 + 8= 19, 19 + 16 = 35

116). B) × 2 - 20, ×2 - 20, × 2 - 20, ------------------

500 Most Important Number Series Questions

www.ibpsguide.com | estore.ibpsguide.com| www.sscexamguide.com

For Free Online Mock Test for IBPS PO/Clerk 2017 – Visit: estore.ibpsguide.com

44 44

117). C) ×(22 - 1), × (32 - 1), (42 - 1), ×(52 -1), ×(62 -1) ----------

118). A) ×1 + 7, ×2 + 14, × 3 + 21, ×4 + 28 ----------------

119). A) × 1 + 2, ×2+ 4 , ×3 + 6, × 4 + 8, × 5 + 10 ------------

120). B)

Directions (121 – 130): What should come in place of the question mark (?) in the following

number series?

121). 7 5 7 17 63 ?

A) 308

B) 302

C) 309

D) 409

E) 390

122). 50 ? 61 89 154 280

A) 52

B) 51

C) 60

D) 62

E) 60

123). 17 19 25 37 ? 87

A) 47

B) 37

C) 57

D) 67

E) 75

124). 11 14 19 28 43 ?

A) 55

B) 44

500 Most Important Number Series Questions

www.ibpsguide.com | estore.ibpsguide.com| www.sscexamguide.com

For Free Online Mock Test for IBPS PO/Clerk 2017 – Visit: estore.ibpsguide.com

45 45

C) 77

D) 88

E) 66

125). 26 144 590 1164 ?

A) 1296

B) 1182

C) 2059

D) 1182

E) 1181

126). 2916 972 ? 108 36 12

A) 324

B) 234

C) 248

D) 391

E) None of these

127). 8, 27, 125, ?, 1331

A) 512

B) 216

C) 81

D) 343

E) 169

128). 2 11 30 97 ? 1975

A) 325

B) 360

C) 34

D) 394

E) 376

129). 8 47 234 935 ? 5607

A) 2804

B) 2802

C) 2808

D) 2801

E) 2800

130). 6 9 15 27 51 ?

500 Most Important Number Series Questions

www.ibpsguide.com | estore.ibpsguide.com| www.sscexamguide.com

For Free Online Mock Test for IBPS PO/Clerk 2017 – Visit: estore.ibpsguide.com

46 46

A) 84

B) 75

C) 99

D) 123

E) None of the above

Answer Key with Solutions:

121. C) 309

7 x 1 – 2 = 5

5 x 2 – 3 = 7

7 x 3 – 4 = 17

17 x 4 – 5 = 63

63 x 5 – 6 = 309

122. A) 52

50..50+(1^3+1) = 52

52+(2^3+1)=61.

61+(3^3+1)=89

89+(4^3+1)=154

154 + (5^3+1) = 280

123. C) 57

17 + 1 x 2 = 19

19 + 2 x 3 = 25

25 + 3 x 4 = 37

37 + 4 x 5 = 57

57 + 5 x 6 = 87

124. E) 66

3….5…9…15…23

…2….4….6….8…

Answer 43+23= 66

125. B) 1182

26 x 6 – 12 = 144

144 x 4 + 14 = 590

590 x 2 – 16 = 1164

1164 x 1 + 18 = 1182

126. A) 324

500 Most Important Number Series Questions

www.ibpsguide.com | estore.ibpsguide.com| www.sscexamguide.com

For Free Online Mock Test for IBPS PO/Clerk 2017 – Visit: estore.ibpsguide.com

47 47

2916/3 = 972

972/3 = 324

324/3 = 108

108/3 = 36

36/3 = 12

127. D) 343

Cube prime series 23, 33, 53, 73, 113

128. D)

2 x 1 + 9 = 11 x 2 + 8 = 30 x 3 + 7 = 97 x 4 + 6 = 394

129. A)

x 6 – 1, x 5 – 1, x 4 – 1, x 3 – 1 = 935 x 3 – 1 = 2804

130. C)

+3, +6, +12, +24, +48 = 51 + 48 = 99

Directions (Q.131-140) What should come in place of question mark (?) in the following number

series?

131). 15, 21, 39, 77, 143, ?

A) 243

B) 240

C) 253

D) 245

E) None of these

132). 33, 39, 57, 87, 129, ?

A) 183

B) 177

C) 189

D) 199

E) None of these

133). 15, 19, 83, 119, 631, ?

A) 731

B) 693

C) 712

500 Most Important Number Series Questions

www.ibpsguide.com | estore.ibpsguide.com| www.sscexamguide.com

For Free Online Mock Test for IBPS PO/Clerk 2017 – Visit: estore.ibpsguide.com

48 48

D) 683

E) None of these

134). 19, 26, 40, 68, 124, ?

A) 246

B) 238

C) 236

D) 256

E) None of these

135). 43, 69, 58, 84, 73,?

A) 62

B) 98

C) 109

D) 63

E) None of these

136). 1, 7, 49, 343,?

A) 16807

B) 1227

C) 2058

D) 2401

E) None of these

137). 13, 20, 39, 78, 145,?

A) 234

B) 244

C) 236

D) 248

E) None of these

138). 12, 35, 81, 173, 357, ?

A) 725

B) 715

500 Most Important Number Series Questions

www.ibpsguide.com | estore.ibpsguide.com| www.sscexamguide.com

For Free Online Mock Test for IBPS PO/Clerk 2017 – Visit: estore.ibpsguide.com

49 49

C) 726

D) 736

E) None of these

139). 3, 100, 297, 594, 991, ?

A) 1489

B) 1479

C) 1478

D) 1498

E) None of these

140). 112, 119, 140, 175, 224, ?

A) 277

B) 276

C) 287

D) 266

E) None of these

Solution:

131).

Answer: D)

132).

500 Most Important Number Series Questions

www.ibpsguide.com | estore.ibpsguide.com| www.sscexamguide.com

For Free Online Mock Test for IBPS PO/Clerk 2017 – Visit: estore.ibpsguide.com

51 51

Answer: E)

136).

Answer: D)

137).

Answer: D)

138).

Answer: A)

139). 3 + 97 = 100 + 100↓

100 + 197 = 297 + 100↓

297 + 297 = 594 + 100↓

594 + 397 = 991 + 100↓

991 + 497 = 1488

Answer: E)

500 Most Important Number Series Questions

www.ibpsguide.com | estore.ibpsguide.com| www.sscexamguide.com

For Free Online Mock Test for IBPS PO/Clerk 2017 – Visit: estore.ibpsguide.com

52 52

140).

Answer: C)

141). 1, 4, 9, 16, 25, ?

A) 49

B) 60

C) 30

D) 36

E) 48

142). 2, 5, 11, 23, ?

A) 46

B) 52

C) 47

D) 57

E) 48

143). 198, 194, 185, 169, ?

A) 92

B) 136

C) 144

D) 112

E) None of these

144). 101, 100, ?, 87, 71, 46

A) 92

B) 88

C) 89

D) 96

E) None of these

500 Most Important Number Series Questions

www.ibpsguide.com | estore.ibpsguide.com| www.sscexamguide.com

For Free Online Mock Test for IBPS PO/Clerk 2017 – Visit: estore.ibpsguide.com

53 53

145). 100, 50, 52, 26, 28, ?, 16, 8

A) 30

B) 36

C) 14

D) 32

E) None of these

146). 462, 420, 380, ?, 306

A) 322

B) 332

C) 342

D) 352

E) None of these

147). 0, 6, 24, 60, 120, 210, ?

A) 290

B) 240

C) 336

D) 504

E) None of these

148). 3, 15, 35, 63, ?, 143

A) 120

B) 110

C) 99

D) 91

E) None of these

149). 4, 9, 19, 39, 79, ?

A) 159

B) 119

C) 139

D) 169

500 Most Important Number Series Questions

www.ibpsguide.com | estore.ibpsguide.com| www.sscexamguide.com

For Free Online Mock Test for IBPS PO/Clerk 2017 – Visit: estore.ibpsguide.com

54 54

E) None of these

150). 5, 10, 8, 12, 11, 14, ?, 16

A) 17

B) 13

C) 20

D) 14

E) None of these

Solution:

141). The pattern is as follows

Answer: D)

142). The pattern is as follows

Answer: C)

143). The pattern is as follows

Answer: C)

144). The pattern is as follows

500 Most Important Number Series Questions

www.ibpsguide.com | estore.ibpsguide.com| www.sscexamguide.com

For Free Online Mock Test for IBPS PO/Clerk 2017 – Visit: estore.ibpsguide.com

55 55

Missing number = 100 – (2)2 = 100 – 4 = 96

Answer: D)

145). The pattern is as follows

Missing number = 28 / 2 = 14

Answer: C)

146). The pattern is as follows

Answer: C)

147). The pattern is as follows

03 – 0 = 0

23 – 2 = 6

33 – 3 = 24

43 – 4 = 60

53 – 5 = 120

63 – 6 = 210

73 – 7 = 336

? = 336

Answer: C)

148). The pattern is as follows

3 = 1 × 3 ; 15 = 3 × 5

35 = 5 × 7 ; 63 = 7 × 9

99 = 9 × 11; 143 = 11 × 13

500 Most Important Number Series Questions

www.ibpsguide.com | estore.ibpsguide.com| www.sscexamguide.com

For Free Online Mock Test for IBPS PO/Clerk 2017 – Visit: estore.ibpsguide.com

56 56

? = 99

Answer: C)

149). The pattern is as follows

? = 342

Answer: A)

150). There are two alternate series

Answer: D)

151). 3, 732, 1244, 1587, 1803, 1928, ?

A) 2144

B) 1999

C) 1995

D) 2053

E) None of these

152). 16, 24, ?, 210, 945, 5197.5, 33783.76

A) 40

B) 36

C) 58

D) 60

E) None of these

153). 5, 12, 36, 123, ?, 2555, 15342

A) 508

500 Most Important Number Series Questions

www.ibpsguide.com | estore.ibpsguide.com| www.sscexamguide.com

For Free Online Mock Test for IBPS PO/Clerk 2017 – Visit: estore.ibpsguide.com

57 57

B) 381

C) 504

D) 635

E) None of these

154). 8, 11, 17, ?, 65, 165.5, 498.5

A) 27.5

B) 32

C) 28

D) 30.5

E) None of these

155). 9, 15, 27, 51, 99, ?

A) 165

B) 195

C) 180

D) 190

E) None of these

156). 13, 21, 36, 58, 87, ?

A) 122

B) 128

C) 133

D) 123

E) None of these

157). 7, 9, 19, 45, 95, ?

A) 150

B) 160

C) 145

D) 177

E) None of these

158). 14, 15, 23, 32, 96, ?

500 Most Important Number Series Questions

www.ibpsguide.com | estore.ibpsguide.com| www.sscexamguide.com

For Free Online Mock Test for IBPS PO/Clerk 2017 – Visit: estore.ibpsguide.com

58 58

A) 121

B) 124

C) 152

D) 111

E) None of these

159). 20, 24, 36, 56, 84, ?

A) 116

B) 124

C) 120

D) 128

E) None of these

160). 117, 389, 525, 593, 627, ?

A) 654

B) 640

C) 6

D) 630

E) None of these

Solution:

151).

Answer: E)

152).

Answer: D)

500 Most Important Number Series Questions

www.ibpsguide.com | estore.ibpsguide.com| www.sscexamguide.com

For Free Online Mock Test for IBPS PO/Clerk 2017 – Visit: estore.ibpsguide.com

59 59

153). 5 × 1 + 1 × 7 = 12

12 × 2 + 2 × 6 = 36

36 × 3 + 3 × 5 = 123

123 × 4 + 4 × 4 = 508

508 × 5 + 5 × 3 = 2555

2555 × 6 + 6 × 2 = 15342

Answer: A)

154).8 ×0.5 + 7 = 11

11 × 1 + 6 = 17

17 × 1.5 + 5 = 30.5

30.5 × 2 + 4 = 65

65 × 2.5 + 3 = 165.5

165.5 × 3 + 2 = 498.5

Answer: D)

155).

Answer: B)

156).

Answer: D)

157). 7 + (1)2 + 1 = 9

9 + (3)2 + 1 = 19

500 Most Important Number Series Questions

www.ibpsguide.com | estore.ibpsguide.com| www.sscexamguide.com

For Free Online Mock Test for IBPS PO/Clerk 2017 – Visit: estore.ibpsguide.com

60 60

19 + (5)2 + 1 = 45

45 + (7)2 + 1 = 95

95 + (9)2 + 1 = 177

Answer: D)

158).

Answer: A)

159).

Answer: C)

160).

Answer: E)

Directions (Q.161-165): What will come in place of question mark(?)in the given number series?

161). 264 262 271 243 308 ?

A) 216

B) 163

C) 194

500 Most Important Number Series Questions

www.ibpsguide.com | estore.ibpsguide.com| www.sscexamguide.com

For Free Online Mock Test for IBPS PO/Clerk 2017 – Visit: estore.ibpsguide.com

61 61

D) 205

E) 182

162).1.5 2.5 7 24 100 ?

A) 345

B) 460

C) 525

D) 380

E) 505

163).71 78 92 120 ? 288

A) 160

B) 176

C) 199

D) 208

E) 164

164). 17 9 10 16.5 35 ?

A) 192

B) 80

C) 114

D) 90

E) 76

165).79 39 19 9 4 ?

A) 0.2

B) 1.5

C) 0.5

D) 2

E) 1

Directions (Q. 166-170): What will come in place of question mark (?)in the following number

series?

166). 13 13 19 43 103 ?

500 Most Important Number Series Questions

www.ibpsguide.com | estore.ibpsguide.com| www.sscexamguide.com

For Free Online Mock Test for IBPS PO/Clerk 2017 – Visit: estore.ibpsguide.com

62 62

A) 221

B) 227

C) 223

D) 217

E) 239

167). 27 13 12 16.5 ? 75

A) 31

B) 29

C) 37

D) 33

E) 35

168). 17 19 42 132 ? 2690

A) 532

B) 544

C) 528

D) 536

E) 512

169). 25 29 67 217 ? 4501

A) 927

B) 877

C) 885

D) 911

E) 893

170). 21 38 59 84 113 ?

A) 138

B) 152

C) 134

D) 146

E) 148

500 Most Important Number Series Questions

www.ibpsguide.com | estore.ibpsguide.com| www.sscexamguide.com

For Free Online Mock Test for IBPS PO/Clerk 2017 – Visit: estore.ibpsguide.com

63 63

Solution:

161). The series is -(13 + 1), +(23 + 1), -(33 + 1), +(43 + 1), -(53 + 1), …...

ie 264 – (13 + 1) = 262, 264 + (23 + 1) = 271

271 – (33 + 1) = 243, 243 + (43 + 1) = 308

308 – (53 + 1) = 182

Answer: E)

162). The series is ×1 + 1, ×2 + 2, ×3 + 3, ×4 + 4, ×5 + 5, ….

ie 1.5 ×1 + 1 = 2.5, 2.5 × 2 + 2 = 7,

7 × 3 + 3 = 24, 24 × 4 + 4 = 100,

100 × 5 + 5 = 505

Answer: E)

163). The series is +7, +14, +28, +56, +112,

ie 71 + 7 = 78, 78 + 14 = 92, 92 + 28 = 120,

120 + 56 = 176, 176 + 112 = 288,

Answer: B)

164). The series is ×0.5 + 0.5, ×1 + 1, ×1.5 + 1, ×2 + 2, ×2.5 + 2.5, …

ie 17 × 0.5 + 0.5 = 9, 9 × 1 + 1 = 10,

10 × 1.5 + 1.5 = 16.5, 16.5 × 2 + 2 = 35,

35 × 2.5 + 2.5 =90, …

Answer: D)

165). The series is /2 – 0.5, /2 -0.5, (repeateD)

ie 79 / 2 – 0.5 = 39, 39 / 2 – 0.5 = 19,

19 / 2 – 0.5 = 9, 9 / 2 – 0.5 = 4, 4 / 2 - 0.5 = 1.5, …

Answer: B)

166). The series is

500 Most Important Number Series Questions

www.ibpsguide.com | estore.ibpsguide.com| www.sscexamguide.com

For Free Online Mock Test for IBPS PO/Clerk 2017 – Visit: estore.ibpsguide.com

64 64

Answer: C)

167). The series is ×0.5 – 0.5, × 1 – 1,

× 1.5 – 1.5, × 2 – 2, × 2.5 – 2.5, …

ie 27 × 0.5 – 0.5 = 13, 13 × 1 – 1 = 12, 12 × 1.5 – 1.5 = 16 . 5,

16.5 × 2 – 2 = 31, 31 × 2.5 – 2.5 = 75,

Answer: A)

168). The series is × 1 + 2, ×2 + 4, ×3 + 6, × 4 + 8, × 5 + 10,

ie 17 × 1 + 2 = 19, 19 × 2 + 4 = 42, 42 × 3 + 6 = 132,

132 × 4 + 8 = 536, 536 ×5 + 10 = 2690, …

Answer: D)

169). The series is ×1 + 22, ×2 + 32, ×3 + 42, ×4 + 52, ×5 + 62, …

ie 25 × 1 + 22 = 29, 29 × 2 + 32 = 67, 67 × 3 + 42 = 217, 217 × 4 + 52 = 893,

893 × 5 + 62 = 4501,

Answer: E)

170). The series is+17, +21, +25, +29, +33, +37, ..

21+17 = 38, 38 + 21 = 59, 59 + 25 = 84, 84 + 29 = 113,

113 + 33 = 146, 146 + 37 = 183, …

Answer: D)

Directions (Questions 171 to 180): Find the missing number in the series given below.

171). 16, 22, 34, 58, 106, ?, 394

A) 178

500 Most Important Number Series Questions

www.ibpsguide.com | estore.ibpsguide.com| www.sscexamguide.com

For Free Online Mock Test for IBPS PO/Clerk 2017 – Visit: estore.ibpsguide.com

65 65

B) 175

C) 288

D) 202

E) 206

172). 10, 33, 102, 309, ?

A) 1030

B) 1050

C) 928

D) 930

E) 935

173). 121, 112, ?, 97, 91, 86

A) 102

B) 108

C) 99

D) 104

E) 106

174). 975, 864, 753, 642, ?

A) 431

B) 314

C) 531

D) 532

E) 542

175). 3, 4, 7, 11, 18, 29, ?

A) 31

B) 39

C) 43

D) 47

E) 32

176). 26, 37, 50, 65, ?, 101

500 Most Important Number Series Questions

www.ibpsguide.com | estore.ibpsguide.com| www.sscexamguide.com

For Free Online Mock Test for IBPS PO/Clerk 2017 – Visit: estore.ibpsguide.com

66 66

A) 77

B) 80

C) 81

D) 82

E) 83

177). 758, 753, 748, 744, 740, 736, ?

A) 732

B) 733

C) 734

D) 735

E) 736

178). 15, 17, 20, 22, 27, 29, ?, ?

A) 31, 38

B) 36, 38

C) 36, 43

D) 38, 45

E) 39, 46

179). 90, 61, 52, 63, 94, ?, 18

A) 72

B) 46

C) 54

D) 81

E) 82

180). 8, 24, 12, ?, 18, 54

A) 28

B) 36

C) 46

D) 38

E) 42

500 Most Important Number Series Questions

www.ibpsguide.com | estore.ibpsguide.com| www.sscexamguide.com

For Free Online Mock Test for IBPS PO/Clerk 2017 – Visit: estore.ibpsguide.com

67 67

Solutions:

171). Here, series moves with the addition of 6, 12, 24, 48, 96 and 192, respectively. Hence the

missing number is 202.

Answer is: D)

172). The pattern is as follows,

Therefore, ? = 930

Answer is: D)

173). Here, series is written in descending order with a difference of 9, 8, 7, 6 and 5, respectively.

Hence, the missing number will be 104.

Answer is: D)

174). The pattern is as follows

Therefore, ? = 531

Answer is: C)

175). Every third element is the sum of its previous two elements

3 + 4 = 7

4 + 7 + 11

7 + 11 = 18

11 + 18 = 29

18 + 29 = 47

Therefore, ? = 47

Answer is: D)

176). The pattern is as follows

500 Most Important Number Series Questions

www.ibpsguide.com | estore.ibpsguide.com| www.sscexamguide.com

For Free Online Mock Test for IBPS PO/Clerk 2017 – Visit: estore.ibpsguide.com

68 68

Therefore, ? = 82

Answer is: D)

177). Debuct the middle digit each time to obtain the next number

758 – 5 = 753

753 – 5 = 748

748 – 4 = 744

744 – 4 = 740

740 – 4 = 736

736 – 3 = 733

Therefore, ? = 733

Answer is: B)

178). The pattern is as follows

First ? = 36 and second ? = 38

Required answer = 36, 38

Answer is: B)

179). When the digit of the numbers are reversed these are the perfect squares 09, 16, 25, 36, 49,

64, 81 consecutive

Therefore, ? = Reverse of 64 = 46

Answer is: B)

180). The pattern is as follows

Therefore, ? = 36

Answer is: B)

181). 3, 6 18, 72, ?

A) 144

B) 216

500 Most Important Number Series Questions

www.ibpsguide.com | estore.ibpsguide.com| www.sscexamguide.com

For Free Online Mock Test for IBPS PO/Clerk 2017 – Visit: estore.ibpsguide.com

69 69

C) 288

D) 360

E) 152

182). 24, 60, 120, 210, ?

A) 300

B) 336

C) 420

D) 525

E) 250

183). 5, 16, 51, 158, ?

A) 1452

B) 483

C) 481

D) 1454

E) 1500

184). 4, ?, 144, 400, 900, 1764

A) 25

B) 36

C) 49

D) 100

E) 120

185). 8, 3, 11, 14, 25

A) 50

B) 39

C) 29

D) 11

E) 12

186). 980, 392, 156.8, ?, 25.088, 10.0352

A) 66.04

B) 61.28

C) 63.72

D) 64.85

E) None of these

187). 77, 59, 55, 35, 25, ?

500 Most Important Number Series Questions

www.ibpsguide.com | estore.ibpsguide.com| www.sscexamguide.com

For Free Online Mock Test for IBPS PO/Clerk 2017 – Visit: estore.ibpsguide.com

70 70

A) 24

B) 28

C) 20

D) 27

E) 29

188). 99, 82, 18, 11, ?

A) 5

B) 10

C) 2

D) 9

E) 8

189). 17, 36, 74, 150, ?, 606

A) 250

B) 303

C) 300

D) 302

E) 305

190). 51975, 9450, 2100, 600, 240, 160, ?

A) 80

B) 120

C) 320

D) 240

E) None of these

181).

182).

500 Most Important Number Series Questions

www.ibpsguide.com | estore.ibpsguide.com| www.sscexamguide.com

For Free Online Mock Test for IBPS PO/Clerk 2017 – Visit: estore.ibpsguide.com

71 71

183).

184).

185).

186). E)

Divided by 2.5 continuesly………

Answer: 62.72

187).

188).

500 Most Important Number Series Questions

www.ibpsguide.com | estore.ibpsguide.com| www.sscexamguide.com

For Free Online Mock Test for IBPS PO/Clerk 2017 – Visit: estore.ibpsguide.com

72 72

189).

190).

Directions (Q. 191-195): Find the missing number (P) in the following series

191). 126 158 174 P 186 188

A) 180

B) 182

C) 184

D) 178

E) None of these

192). 2.7 P 3.6 2.4 4.8 3.2

A) 1.5

B) 1.8

C) 2.1

D) 2.4

E) 2.5

500 Most Important Number Series Questions

www.ibpsguide.com | estore.ibpsguide.com| www.sscexamguide.com

For Free Online Mock Test for IBPS PO/Clerk 2017 – Visit: estore.ibpsguide.com

73 73

193). 15(2/3) 13(1/3) P 8(2/3) 6(1/3) 4

A) 8

B) 9

C) 10

D) 11

E) None of these

194). P 58 84 114 146 182 220

A) 28

B) 30

C) 32

D) 34

E) 36

195). 127 P 85 67 51 37 25

A) 105

B) 100

C) 95

D) 90

E) None of these

Directions (Q. 196-200): What should come in place question mark (?) in the following

number series?

196). 262 234 206 178 150 122 ?

A) 76

B) 78

C) 84

D) 89

E) 94

197). 4762 4627 4494 4363 4234 ?

A) 4147

B) 4137

C) 4127

500 Most Important Number Series Questions

www.ibpsguide.com | estore.ibpsguide.com| www.sscexamguide.com

For Free Online Mock Test for IBPS PO/Clerk 2017 – Visit: estore.ibpsguide.com

74 74

D) 4117

E) 4107

198). 672 560 448 336 224 ?

A) 172

B) 142

C) 132

D) 112

E) 102

199). 18 97 396 1197 2404 ?

A) 2816

B) 3215

C) 3612

D) 2415

E) 3600

200). 26 144 590 1164 ?

A) 1864

B) 1732

C) 1460

D) 1296

E) 1182

Solutions:

191. The logic of the sequence is:

+25, +24, +23, 22, +21

Answer: B)

192. The logic of the sequence is: ÷1.5, × 2 alternately

Answer: B)

193. The logic of the sequence is: -2(1/3)

500 Most Important Number Series Questions

www.ibpsguide.com | estore.ibpsguide.com| www.sscexamguide.com

For Free Online Mock Test for IBPS PO/Clerk 2017 – Visit: estore.ibpsguide.com

75 75

Answer: D)

194. The logic of the sequence is: +24, +26, +30, +32, +36, +38

Answer: D)

195. The logic of the sequence is -22, -20, -18, -16, -14, -12

Answer: A)

196. The series is

(29×9)+1=262

(29×8)+2=234

(29×7)+3=206

(29×6)+4=178

(29×5)+5=50

(29×4)+6=122

(29×3)+7=94

Answer: E)

197. The series is

(69)2+=4762

(68)2+3=4627

(67)2+5=4494

(66)2+7=4363

(65)2+9=4234

(64)2+11=4107

Answer: E)

198. The series is

666+6=672

555+5=560

444+4=448

333+3=336

222+2=224

111+1=112

500 Most Important Number Series Questions

www.ibpsguide.com | estore.ibpsguide.com| www.sscexamguide.com

For Free Online Mock Test for IBPS PO/Clerk 2017 – Visit: estore.ibpsguide.com

76 76

Answer: D)

199. The series is

2×6+6=18

18×5+7=97

97×4+8=396

396×3+9=1197

1197×2+10=2404

2404×1+11=2415

Answer: D)

200. The series is

2×8+10=26

26×6-12=144

144×4+14=590

590×2-16=1164

1164×1+18=1182

Answer: E)

Directions (Questions 201 to 210): What should come in place of question mark (?) in the

following number series?

201). 1 2 6 15 31 56 ?

A) 96

B) 94

C) 98

D) 92

E) 99

202). 41 39 43 37 45 ?

A) 55

B) 36

C) 33

D) 35

E) 49

500 Most Important Number Series Questions

www.ibpsguide.com | estore.ibpsguide.com| www.sscexamguide.com

For Free Online Mock Test for IBPS PO/Clerk 2017 – Visit: estore.ibpsguide.com

77 77

203). 3 8 23 68 203 ?

A) 607

B) 608

C) 604

D) 606

E) 610

204). 12 6 6 9 18 ?

A) 45

B) 54

C) 63

D) 36

E) 32

205). 9 15 27 45 69 99 ?

A) 137

B) 133

C) 135

D) 139

E) 149

206). 2 51 87 112 ? 137

A) 148

B) 121

C) 128

D) 118

E) None of these

207). 2 3 8 27 ? 565 3396

A) 34

B) 108

C) 110

D) 112

500 Most Important Number Series Questions

www.ibpsguide.com | estore.ibpsguide.com| www.sscexamguide.com

For Free Online Mock Test for IBPS PO/Clerk 2017 – Visit: estore.ibpsguide.com

78 78

E) 120

208). 8 5 ? 26.75 97.625 444.3125

A) 10.5

B) 9.5

C) 7.5

D) 12.5

E) 11.25

209). 27 125 ? 729 1331 2197

A) 216

B) 343

C) 512

D) 289

E) 256

210). 4 10 30 102 ? 2110

A) 610

B) 420

C) 121

D) 120

E) 510

Solutions:

201). The pattern of the number series is as given below:

1 + (1)2 = 2, 2 +(2)2= 6, 6 + (3)2 = 15, 15 + (4)2 = 31, 31 + (5)2 = 56, 56 + (6)2 = 56

+ 36 = 92

Answer is: D)

202). There are two alternating series of +2 and -2 pattern respectively.

500 Most Important Number Series Questions

www.ibpsguide.com | estore.ibpsguide.com| www.sscexamguide.com

For Free Online Mock Test for IBPS PO/Clerk 2017 – Visit: estore.ibpsguide.com

79 79

Answer is: D)

203). The difference gets multiplied by 3 successively.

The series is 3 + 5 = 8, 8 + 15 = 23,

23 + 45 = 68, 68 + 135 = 203, 203 + 405 = 608, ….

Answer is: B)

204). The series is 12 × 0.5 = 6, 6 × 1 = 6, 6 × 1.5 = 9, 9 × 2 = 18, 18 × 2.5 = 45, ….

Answer is: A)

205). The series is 9 + (1 × 6) = 15, 15 + (2 × 6) = 15 + 12 = 27, 27 + (3 × 6) = 27 + 18 = 45,

45 + (4 × 6) = 45 + 24 = 69, 69 + (5 × 6) = 69 + 30 = 99, 99 + (6 × 6) = 99 × 36 = 135, ….

Answer is: C)

206). The series is

+72, +62, +52, +42, +32, …..

Answer is: C)

207). The series is 2 × 1 + 1 = 3, 3 × 2 + 2 = 8, 8 × 3 + 3 = 27, 27 × 4 + 4 = 112, 112 × 5 + 5

= 565, 565 × 6 + 6 = 3396, ….

Answer is: D)

208). The series is 8 × 0.5 + 1 = 5, 5 × 1.5 + 2 = 9.5, 9.5 × 2.5 + 3 = 26.75, 26.75 × 3.5 + 4 =

97.625, 97.625 × 4.5 + 5 = 444.3125, ….

Answer is: B)

209). The series is (3)3, (5)3, (7)3, (9)3, (11)3, (13)3, ….

Answer is: B)

210). The series is 4 × 1 + 1 × 6 = 10

10 × 2 + 2 × 5 = 30

30 × 3 + 3 × 4 = 102

102 × 4 + 4 × 3 = 420

420 × 5 + 5 × 2 = 2110

610 × 6 + 6 × 1 = 3660 + 6 = 3666

Answer is: B)

500 Most Important Number Series Questions

www.ibpsguide.com | estore.ibpsguide.com| www.sscexamguide.com

For Free Online Mock Test for IBPS PO/Clerk 2017 – Visit: estore.ibpsguide.com

80 80

Directions (Q 211-220): What should come in place of questions mark (?) in the following

number series?

211). 7, 35, 105, 525, 1575, 7875, ?

A) 39375

B) 23625

C) 11815

D) 15750

E) None of these

212). 0, 5, 24, 75, 152, ?

A) 153

B) 304

C) 308

D) 312

E) None of these

213). 17, 19, 21, 22, 25, 25, 29, 28, ?

A) 31

B) 32

C) 33

D) 34

E) None of these

214). 16, 48, 24, 72, 36, 108, ?

A) 148

B) 196

C) 216

D) 224

E) None of these

215). 2/31, 5/17, 12.5/23, 31.25/19, 78.125/15, ?/11.

A) 156.25

500 Most Important Number Series Questions

www.ibpsguide.com | estore.ibpsguide.com| www.sscexamguide.com

For Free Online Mock Test for IBPS PO/Clerk 2017 – Visit: estore.ibpsguide.com

81 81

B) 156.3125

C) 234.375

D) 312.5

E) None of these

216). 11, 13, 17, 19, 23, 29, 31, 37, ?

A) 47

B) 43

C) 41

D) 39

E) None of these

217). 165, 195, 255, 285, 345, ?

A) 375

B) 420

C) 435

D) 390

E) None of these

218). 7, 26, 63, 124, 215, 342, ?

A) 481

B) 511

C) 391

D) 421

E) 451

219). 2, 4, 12, 48, 240, ?

A) 1920

B) 960

C) 1080

D) 1440

E) None of these

220). 8, 7, 11, 12, 14, 17, 17, 22, ?

500 Most Important Number Series Questions

www.ibpsguide.com | estore.ibpsguide.com| www.sscexamguide.com

For Free Online Mock Test for IBPS PO/Clerk 2017 – Visit: estore.ibpsguide.com

82 82

A) 20

B) 22

C) 24

D) 27

E) None of these

Solutions:

211).

212).

213).

214).

500 Most Important Number Series Questions

www.ibpsguide.com | estore.ibpsguide.com| www.sscexamguide.com

For Free Online Mock Test for IBPS PO/Clerk 2017 – Visit: estore.ibpsguide.com

83 83

215).

216). All are Prime Numbers

Answer: c

217).

500 Most Important Number Series Questions

www.ibpsguide.com | estore.ibpsguide.com| www.sscexamguide.com

For Free Online Mock Test for IBPS PO/Clerk 2017 – Visit: estore.ibpsguide.com

84 84

218).

219).

220). There are two series

8, 11, 14, 17, 20 and 7, 12, 17, 22

There for Next number is 20

Answer: a

Directions (221-230): What will come in place of question mark (?) in the following number

series?

500 Most Important Number Series Questions

www.ibpsguide.com | estore.ibpsguide.com| www.sscexamguide.com

For Free Online Mock Test for IBPS PO/Clerk 2017 – Visit: estore.ibpsguide.com

85 85

221). 13, 16, 22, 33, 51, ?

A) 89

B) 78

C) 102

D) 69

E) None of these

222). 39, 52, 78, 117, 169, ?

A) 246

B) 182

C) 234

D) 256

E) None of these

223). 656, 432, 320, 264, 236, ?

A) 222

B) 229

C) 232

D) 223

E) None of these

224). 62, 87, 187, 412, 812, ?

A) 1012

B) 1437

C) 1337

D) 1457

E) None of these

225). 7, 8, 24, 105, 361, ?

A) 986

B) 617

C) 486

D) 1657

E) None of these

500 Most Important Number Series Questions

www.ibpsguide.com | estore.ibpsguide.com| www.sscexamguide.com

For Free Online Mock Test for IBPS PO/Clerk 2017 – Visit: estore.ibpsguide.com

86 86

226). 9, 62, ?, 1854, 7415, 22244

A) 433

B) 309

C) 406

D) 371

E) None of these

227). 4, 8, 24, 60, ?, 224

A) 178

B) 96

C) 109

D) 141

E) None of these

228). 8000, 1600, 320, 64, 12.8, ?

A) 2.56

B) 3.5

C) 3.2

D) 2.98

E) None of these

229). 6, 9, 15, 27, 51, ?

A) 84

B) 99

C) 123

D) 75

E) None of these

230). 7, 8, 18, ?, 232, 1165

A) 84

B) 42

C) 57

D) 36

500 Most Important Number Series Questions

www.ibpsguide.com | estore.ibpsguide.com| www.sscexamguide.com

For Free Online Mock Test for IBPS PO/Clerk 2017 – Visit: estore.ibpsguide.com

87 87

E) None of these

Solutions:

221).

222).

223).

224).

225).

226).

500 Most Important Number Series Questions

www.ibpsguide.com | estore.ibpsguide.com| www.sscexamguide.com

For Free Online Mock Test for IBPS PO/Clerk 2017 – Visit: estore.ibpsguide.com

88 88

227).

228).

229).

230).

Direction (Q. 231–240): Find out the number in place of question mark (?) in the following

number series.

231). 529 841 961 1369 1681 1849 ?

A) 2809

B) 3249

C) 2208

D) 6424

E) 2209

232). 1108 1117 1142 1191 ? 1481

A) 1312

500 Most Important Number Series Questions

www.ibpsguide.com | estore.ibpsguide.com| www.sscexamguide.com

For Free Online Mock Test for IBPS PO/Clerk 2017 – Visit: estore.ibpsguide.com

89 89

B) 1272

C) 1300

D) 1204

E) None of these

233). 841 961 1089 1225 1369 1521 ?

A) 1785

B) 1581

C) 1681

D) 1881

E) 1781

234). 12 14 32 102 416 2090 ?

A) 15522

B) 12552

C) 13525

D) 17552

E) None of these

235). 384 381 372 345 264 ?

A) 25

B) 27

C) 44

D) 49

E) None of these

236). 5 41 321 1921 7681 15361 ?

A) 21381

B) 23710

C) 22173

D) 23121

E) 1

237). 1 3 7 13 22 34 51 71 96 124 ?

500 Most Important Number Series Questions

www.ibpsguide.com | estore.ibpsguide.com| www.sscexamguide.com

For Free Online Mock Test for IBPS PO/Clerk 2017 – Visit: estore.ibpsguide.com

90 90

A) 137

B) 147

C) 157

D) 167

E) 177

238). 664 334 85.5 16.25 4.03125 ?

A) 2.73012

B) 3.17203

C) 2.403125

D) 3.220175

E) 2.220175

239). 33 110 453 2282 13711 ?

A) 95673

B) 96747

C) 96312

D) 951321

E) 96000

240). 0.8 3.8 12.6 44.8 188.2 ?

A) 758.6

B) 868.8

C) 952.00

D) 1012.2

E) 1112.2

Solution:

231). The series is 232, 292, 312, 372, ….

Answer: E)

232). The series is +32, +52, +72, +112, +132,….

Answer: A)

500 Most Important Number Series Questions

www.ibpsguide.com | estore.ibpsguide.com| www.sscexamguide.com

For Free Online Mock Test for IBPS PO/Clerk 2017 – Visit: estore.ibpsguide.com

91 91

233). The series is +120, +128, +136, +…. +160, ….

Answer: C)

234). The series is ×1 + 2, ×2 + 4, ×3 + 6, ×4 + 8, ×5 + 10, ×6 + 12,…….

Answer: B)

235). The series is -3, -9, -27 ,-81 -243, ….

Answer: E)

236). The series is :

5 × 10 – 9 = 41

41 × 8 – 7 = 321

321 × 6 – 5 = 1921

1921 × 4 – 3 = 7681

7681 × 2 – 1 = 15361

15361 × 0 – (-1) = 1

Answer: E)

237). The series is :

1 + 1 + 1 = 3

3 + 2 + 2 = 7

7 + 3 + 3 = 13

13 + 4 + 5 = 22

22 + 5 + 7 = 34

34 + 6 + 11 = 51

51 + 7 + 13 = 71

71 + 8 + 17 = 96

96 + 9 + 19 = 124

124 + 10 + 23 = 157

First is number (1, 2, 3, 4…) etc and second is prime numbers (1, 2, 3, 5, 7, 11, 13…)

Answer: C)

238). The series is :

664 + 4 ÷ 2 = 334

500 Most Important Number Series Questions

www.ibpsguide.com | estore.ibpsguide.com| www.sscexamguide.com

For Free Online Mock Test for IBPS PO/Clerk 2017 – Visit: estore.ibpsguide.com

92 92

334 + 8 ÷ 4 = 85.5

85.5 + 12 ÷ 6 = 16.25

16.25 + 16 ÷ 8 = 4.03125

4.03125 + 20 ÷ 10 = 2.403125

Answer: C)

239). The series is :

13 × 2 + 7 = 33

33 × 3 + 11 = 110

110 × 4 + 13 = 453

453 × 5 + 17 = 2282

2282 × 6 + 19 = 13711

13711 × 7 + 23 = 96000

All are prime number ≥ 7.

Answer: E)

240). The series is :

0.8 × 1 + 3 = 3.8

3.8 × 2 + 5 = 12.6

12.6 × 3 + 7 = 44.8

44.8 × 4 + 9 = 188.2

188.2 × 5 + 11 = 952.00

Answer: C)

Directions (Q. 241-245): What should come in place of question mark (?) in the following

number series?

241). 19 42 88 180 364 ? 1468

A) 1046

B) 732

C) 472

D) 630

E) 595

500 Most Important Number Series Questions

www.ibpsguide.com | estore.ibpsguide.com| www.sscexamguide.com

For Free Online Mock Test for IBPS PO/Clerk 2017 – Visit: estore.ibpsguide.com

93 93

242). 51 57 102 324 ? 6390 38304

A) 1590

B) 1296

C) 1680

D) 1250

E) 1272

243). 1953.125 781.25 312.5 125 50 ?

A) 20

B) 25

C) 45

D) 15

E) 30

244). 4 11 32 74 144 ? 396

A) 289

B) 236

C) 205

D) 249

E) 196

245). 8 28 116 544 ? 13300

A) 3589

B) 5482

C) 2672

D) 7864

E) 9378

Directions (Q. 246-250): In each of the following questions, one number is missing in the

series. You have to understand the pattern of the series and then insert the missing

number.

246). 35 31 40 24 49 13 62 ?

A) 11

500 Most Important Number Series Questions

www.ibpsguide.com | estore.ibpsguide.com| www.sscexamguide.com

For Free Online Mock Test for IBPS PO/Clerk 2017 – Visit: estore.ibpsguide.com

94 94

B) -7

C) 18

D) -2

E) 5

247). 25 27.5 22.5 30 20 32.5 ?

A) 17.5

B) 7.5

C) 13.5

D) 22.5

E) 18.5

248). 11 12 26 81 328 ? 9876

A) 986

B) 1132

C) 1645

D) 1286

E) 1542

249). 1284 644 324 164 84 44 24 ?

A) 10

B) 19

C) 15

D) 9

E) 14

250). 4 5 18 81 388 ? 12606

A) 2933

B) 2468

C) 2065

D) 1979

E) 1732

Answers:

500 Most Important Number Series Questions

www.ibpsguide.com | estore.ibpsguide.com| www.sscexamguide.com

For Free Online Mock Test for IBPS PO/Clerk 2017 – Visit: estore.ibpsguide.com

95 95

1). B) 2).E) 3). A) 4). D) 5). C) 6). D) 7). A) 8). C) 9). E) 10). C)

Solution:

241) The series is +23, +46, +92, +184, +368, +736, …

Answer: b

242. The series is ×1 +6, ×2 -12, ×3 +18, ×4 -24, ×5 + 30, ×6 -36, ….

ie 51 × 1 + 6 = 57, 57×2-12 = 102, 102×3+18 = 324, 324×4-24 = 1272, 1272×5+30 = 6390,

6390×6-36 = 38304, …..

Answer: e

243. The series is ÷2.5, ÷2.5 (repeateD)

Answer: a

244. The series is +7 ×1, +7 ×3, +7 ×6, +7 ×10, +7 ×15, +7 ×21,

ie 4+7×1 = 11, 11+7×3 = 32

32+7×6 = 74, 74+7×10 =144,

144+7×15 = 249, 249+7× 21= 396, ….

Answer: d

245. The series is 8×5 – 12 = 28,

28 × 5 -24 = 116, 116 × 5 - 36 = 544,

544 × 5 - 48 = 2672, 2672 × 5 – 60 = 13300

Answer: c

500 Most Important Number Series Questions

www.ibpsguide.com | estore.ibpsguide.com| www.sscexamguide.com

For Free Online Mock Test for IBPS PO/Clerk 2017 – Visit: estore.ibpsguide.com

96 96

246. The series is –(2)2, + (3)², -(4)², +(5)², …

Answer: d

247. The series is +(5/2), -(10/2), (15/2), -(20/2) …

Answer: a

248. The series is ×1 + 1, ×2 +2, ×3 + 3, ×4 + 4, ….

Answer: c

249. The series is ÷2 + 2, ÷2 + 2, (repeateD)

Answer: e

250. The series is ×1 +1³, ×2 + 2³, ×3 + 3³, …

Answer: c

500 Most Important Number Series Questions

www.ibpsguide.com | estore.ibpsguide.com| www.sscexamguide.com

For Free Online Mock Test for IBPS PO/Clerk 2017 – Visit: estore.ibpsguide.com

97 97

Directions (251-26): What should come in place question mark (?) in the following number

series?

251).18 99 682 5443 ?

A) 48972

B) 48963

C) 48957

D) 48948

E) 48682

252). 1324 1721 2190 2737 3368 ?

A) 3986

B) 4089

C) 4136

D) 4290

E) 4068

253). 363 234 141 78 39 18 ?

A) 12

B) 11

C) 10

D) 9

E) 8

254). 665 463 307 191 109 ?

A) 79

B) 69

C) 55

D) 51

E) 47

255). 210 336 504 720 990 1320 ?

A) 1651

B) 1688

500 Most Important Number Series Questions

www.ibpsguide.com | estore.ibpsguide.com| www.sscexamguide.com

For Free Online Mock Test for IBPS PO/Clerk 2017 – Visit: estore.ibpsguide.com

98 98

C) 1716

D) 1794

E) 1720

256). 3 81 ? 1029 2187 3993

A) 375

B) 648

C) 192

D) 575

E) 243

257). 30 45 75 105 165 ?

A) 185

B) 205

C) 215

D) 195

E) 230

258). 8 24 12 36 18 54 ?

A) 64

B) 79

C) 34

D) 37

E) 27

259). 4320 720 144 ? 12 6 6

A) 56

B) 60

C) 26

D) 36

E) 16

260). 26 63 124 215 342 ?

A) 511

500 Most Important Number Series Questions

www.ibpsguide.com | estore.ibpsguide.com| www.sscexamguide.com

For Free Online Mock Test for IBPS PO/Clerk 2017 – Visit: estore.ibpsguide.com

99 99

B) 509

C) 504

D) 515

E) 525

Answers:

1). A) 2).b ) 3).d ) 4).c ) 5). C) 6). A) 7).d ) 8).e ) 9). D) 10). A)

Solution:

251). The series is:

18 × 6 – 9 = 99

99 × 7 – 11 = 682

682 × 8 – 13 = 5443

5443 × 9 – 15 = 48972

Answer: A)

252). The series is:

(11)3 – 7 = 1324

(12)3 – 7 = 1721

(13)3 – 7 = 2190

(14)3 – 7 = 2737

(15)3 – 7 = 3368

(16)3 – 7 = 4089

Answer: B)

253). The series is:

13 + 7 + 73 = 363

12 + 6 + 63 = 234

11 + 5 + 53 = 141

10 + 4 + 43 = 78

9 + 3 + 33 = 39

8 + 2 + 23 = 18

7 + 1 + 13 = 9

500 Most Important Number Series Questions

www.ibpsguide.com | estore.ibpsguide.com| www.sscexamguide.com

For Free Online Mock Test for IBPS PO/Clerk 2017 – Visit: estore.ibpsguide.com

100 100

Answer: D)

254). The series is:

93 – 82 = 665

83 – 72 = 463

73 – 62 = 307

63 – 52 = 191

53 – 42 = 109

43 – 32 = 55

Answer: C)

255). The series is:

5 × 6 × 7 = 210

6 × 7 × 8 = 336

7 × 8 × 9 = 504

8 × 9 × 10 = 720

9 × 10 × 11 = 990

10 × 11 × 12 = 1320

11 × 12 × 13 = 1716

Answer: C)

256). The series is 13 × 3, 33 × 3, 53 × 3, 73 × 3, ….

Answer: A)

257). Each number is a prime number multiplied by 15.

Thus, the series is 15 × 2, 15 × 3, 15 × 5, 15 × 7, 15 × 11, ….

Answer: D)

258). The series is ×3, ÷2, ×3, ÷2,…..

Answer: E)

259). The series is ÷6, ÷5, ÷4, ÷3 ….

Answer: D)

500 Most Important Number Series Questions

www.ibpsguide.com | estore.ibpsguide.com| www.sscexamguide.com

For Free Online Mock Test for IBPS PO/Clerk 2017 – Visit: estore.ibpsguide.com

101 101

260). The series is (33 – 1), (43 – 1), (53 – 1), (63 – 1), (73 – 1)

Answer: A)

Directions (261-270): What value should come in place of question mark (?) in the following

number series?

261). 796 199 50 13 4 ?

A) 4

B) 1

C) 5

D) 2

E) 3

262). 445 534 631 736 849 ?

A) 961

B) 965

C) 970

D) 958

E) 968

263). 7 9.21 16.05 29.94 53.3 ?

A) 88.55

B) 90.58

C) 92.48

D) 91.68

E) 94.78

264). 6 10 28 68 138 ?

A) 246

B) 236

C) 238

D) 232

E) 242

265). 16 9 6 5 ? 5.5

500 Most Important Number Series Questions

www.ibpsguide.com | estore.ibpsguide.com| www.sscexamguide.com

For Free Online Mock Test for IBPS PO/Clerk 2017 – Visit: estore.ibpsguide.com

102 102

A) 5

B) 4

C) 3

D) 1

E) 2

266). 24 12 ? 18 36 90

A) 15

B) 12

C) 10.5

D) 11

E) 18

267). 7 20 59 ? 215 332

A) 120

B) 155

C) 165

D) 124

E) None of these

268). ? 17 29 59 115 205

A) 14

B) 16

C) 15

D) 13

E) 11

269). 10 30 68 130 ? 350

A) 225

B) 250

C) 242

D) 222

E) None of these

500 Most Important Number Series Questions

www.ibpsguide.com | estore.ibpsguide.com| www.sscexamguide.com

For Free Online Mock Test for IBPS PO/Clerk 2017 – Visit: estore.ibpsguide.com

103 103

270). 170 196 248 ? 560 976

A) 350

B) 352

C) 452

D) 380

E) None of these

Answer:

1) D) 2) C) 3) A) 4) A) 5) A) 6) B) 7) D) 8) C) 9) D) 10) B)

Detailed Explanation:

261). The series is ÷ 4, + 1 ÷4, + 2 ÷ 4, + 3 ÷4,…

:. ? = (4 + 4) ÷ 4 = 2

Answer: D)

262). The series is (21)2 + 4, (23)2 + 5, (25)2 + 6, (27)2 + 7,….

:. ? = (31)2 + 9 = 961 + 9 =970

Answer: C)

263). The series is +1 + (1.1)2, + 2 + (2.2)2, + 3 + (3.3)2,…

:. ? = 53.3 + 5 + (5.5)2 = 88.55

Answer: A)

264). The series is + 32 – 5, + 52 – 7, + 72 – 9, + 92 – 11, ….

500 Most Important Number Series Questions

www.ibpsguide.com | estore.ibpsguide.com| www.sscexamguide.com

For Free Online Mock Test for IBPS PO/Clerk 2017 – Visit: estore.ibpsguide.com

104 104

:. ? = 138 + 121 – 13 =246

Answer: A)

265). The series is + 2 ÷ 2, + 3 ÷ 2, + 4 ÷ 2, + 5 ÷ 2,…..

:. ? = (5 + 5) * (1 / 2) = 5

Answer: A)

266). The series is × 0.5, ×1, × 1.5, × 2, × 2.5, ….

Answer: B)

267). The series is + (1 × 13), + (3 × 13), + (5 × 13), + (7 × 13),….

Answer: D)

268). The series is 15 + 1 × 2 = 17, 17 + 3 × 4 =29, 29 + 5 × 6 = 59, 59 + 7 × 8 = 115, 115 + 9

× 10 = 205, ….

Answer: C)

269). The series is 23 + 2, 33 + 3 , 43 + 4 , 53 + 5, 63 + 6 , 73 + 7,…..

Answer: D)

270). The series is + 26, + 52, + 104, + 208, + 416,…

Answer: B)

Directions (271-275): In each of the following questions, one number is missing in the

series. You have to understand the pattern of the series and then insert the missing number

in place of question mark (?).

271). 5 10 30 105 ? 2225

A) 860

B) 440

C) 1250

D) 320

E) 1575

272). 2 3 5 15 65 255 1265 ?

500 Most Important Number Series Questions

www.ibpsguide.com | estore.ibpsguide.com| www.sscexamguide.com

For Free Online Mock Test for IBPS PO/Clerk 2017 – Visit: estore.ibpsguide.com

105 105

A) 4295

B) 3405

C) 2560

D) 3185

E) 5055

273). 523 507 471 407 307 163 ?

A) 105

B) 29

C) -33

D) -17

E) None of these

274). 276 140 68 36 16 ? 3

A) 10

B) 15

C) 5

D) 7

E) 11

275). 105 185 247 293 ? 345 355

A) 225

B) 286

C) 315

D) 325

E) 338

Directions ( 276-280): What should come in place of question mark in the following

questions ?

276). 7 10 26 87 360 1815 ?

A) 13256

B) 12864

C) 10908

500 Most Important Number Series Questions

www.ibpsguide.com | estore.ibpsguide.com| www.sscexamguide.com

For Free Online Mock Test for IBPS PO/Clerk 2017 – Visit: estore.ibpsguide.com

106 106

D) 15069

E) 20185

277). 11 5 -2 -24 -120 ? -3816

A) -630

B) -600

C) -675

D) -720

E) -913

278). 4 6 18 81 486 ? 32805

A) 2562

B) 3218

C) 3985

D) 3645

E) 4286

279). 4 10 33 136 ? 4116 28819

A) 829

B) 972

C) 730

D) 523

E) 685

280). 175 174 173 169 161 152 ? 109

A) 108

B) 119

C) 125

D) 128

E) 135

Detailed Explanation:

271). The series is × 1 + 5, × 2 + 10, × 3 + 15, × 4 + 20,……

500 Most Important Number Series Questions

www.ibpsguide.com | estore.ibpsguide.com| www.sscexamguide.com

For Free Online Mock Test for IBPS PO/Clerk 2017 – Visit: estore.ibpsguide.com

107 107

Answer: B)

272). The series is × 4 – 5, × 5 – 10, × 4 – 5, × 5 – 10,

Answer: E)

273). The series is – (4)2, - (6)2, - (8)2,…

Answer: C)

274). The series is ÷ 2 + 2, ÷ 2 – 2, ÷ 2 + 2, ÷ 2 – 2,

Answer: A)

275). The series is + 92 - 1, + 82 – 2, + 72 – 3,

Answer: D)

276). The series is × 1 + 3, × 2 + 6, × 3 + 9, × 4 + 12, × 5 + 15,…

Answer: C)

277). The series is × 1 – 6, × 2 – 12, × 3 – 18,

500 Most Important Number Series Questions

www.ibpsguide.com | estore.ibpsguide.com| www.sscexamguide.com

For Free Online Mock Test for IBPS PO/Clerk 2017 – Visit: estore.ibpsguide.com

108 108

Answer: A)

278). The series is × 1.5, × 3 , × 4.5,..

Answer: D)

279). The series is × 2 + 2, × 3 + 3, × 4 + 4, × 5 + 5, × 6 + 6,….

Ie 4 × 2 + 2 =10, 10 × 3 + 3 =33,

33 × 4 + 4 = 136, 136 × 5 + 5= 685,

685 × 6 + 6 = 4116, 4116 × 7 + 7 = 28819

Answer: E)

280). The series is – (1)2 , - (1)3, - (2)2, -(2)3, -(3)2, -(3)3,….

Answer: C)

Directions (281-290): What will come in place of question mark (?) in the given number

series?

281) 8 11 20 47 128 ?

A) 372

B) 371

C) 308

D) 416

E) 364

282).13 27 44 ? 87 113

A) 64

B) 63

C) 62

D) 81

E) 69

500 Most Important Number Series Questions

www.ibpsguide.com | estore.ibpsguide.com| www.sscexamguide.com

For Free Online Mock Test for IBPS PO/Clerk 2017 – Visit: estore.ibpsguide.com

109 109

283). 2 11 38 119 362 ?

A) 1052

B) 1091

C) 1152

D) 1061

E) 1291

284).17 19 41 127 ? 2571

A) 508

B) 506

C) 513

D) 416

E) 516

285).108 113 103 ? 98 123

A) 116

B) 109

C) 115

D) 118

E) 123

286). 511 733 1177 ? 2731 3841

A) 1840

B) 1843

C) 1748

D) 1943

E) 2040

287). 3.5 8 36 225 1816 ?

A) 12736

B) 18165

C) 12737

D) 14737

E) 18185

500 Most Important Number Series Questions

www.ibpsguide.com | estore.ibpsguide.com| www.sscexamguide.com

For Free Online Mock Test for IBPS PO/Clerk 2017 – Visit: estore.ibpsguide.com

110 110

288). 7 21 105 735 ? 72765

A) 6315

B) 6625

C) 6605

D) 6615

E) 6251

289). 63 64 132 405 ? 8205

A) 1620

B) 1646

C) 1656

D) 1426

E) 1636

290). 79 88 106 142 214 ?

A) 258

B) 352

C) 358

D) 458

E) 268

Solution Hint:

281).The series is +3, +9, +27, +81, +243, ….

Answer: B)

282).The series is +14, +17, +20, +23, +26, +29, …

Answer: A)

283).The series is +32, +33, +34 , +35, +36, …

Answer: B)

284).The series is ×1 +2, ×2+3, ×3+4, ×4+5, ×5+6, …

500 Most Important Number Series Questions

www.ibpsguide.com | estore.ibpsguide.com| www.sscexamguide.com

For Free Online Mock Test for IBPS PO/Clerk 2017 – Visit: estore.ibpsguide.com

111 111

Answer: C)

285).The series is +5, -10, +15, -20, +25, …

Answer: D)

286).The series is +222×1, +222×2, +222×3, +222×4, …

∴?= 1177+666=1843

Answer: B)

287). The series is ×2+1², ×4+2², ×6+3², ×8+4², ×10+5², …

Answer: E)

288).The series is ×3, ×5, ×7, ×9, ×11, …

Answer: D)

289).The series is ×1+1², ×2+2², ×3+3², ×4+4², ×5+5², …

Answer: E)

290). The series is +9, +18, +36, +72, +144, …

Answer: C)

Directions (291-300): What value should come in place of question mark (?) in the following

questions?

291). 5 3 3 5 15 ?

A) 55

B) 5

C) 69

D) 21

E) 3

292). 18 19 42 135 556 ?

A) 2910

B) 2805

C) 2530

500 Most Important Number Series Questions

www.ibpsguide.com | estore.ibpsguide.com| www.sscexamguide.com

For Free Online Mock Test for IBPS PO/Clerk 2017 – Visit: estore.ibpsguide.com

112 112

D) 2790

E) 2525

293). 2 4 ? 18 64 330

A) 12

B) 20

C) 28

D) 10

E) 4

294). 7 24 38 86 162 ?

A) 259

B) 269

C) 324

D) 334

E) 314

295). 12 24 96 576 4608 ?

A) 19536

B) 46080

C) 40680

D) 27648

E) 34560

296). 9 17 65 385 3073 ?

A) 40704

B) 73251

C) 65506

D) 38521

E) 30721

297). 25 241 584 1096 1825 ?

A) 2625

B) 2525

500 Most Important Number Series Questions

www.ibpsguide.com | estore.ibpsguide.com| www.sscexamguide.com

For Free Online Mock Test for IBPS PO/Clerk 2017 – Visit: estore.ibpsguide.com

113 113

C) 2725

D) 2825

E) 2025

298). 50 25 37.5 93.75 328.125 ?

A) 1656.6525

B) 1476.5625

C) 1576.6225

D) 2025.5625

E) None of these

299). 468 516 984 1500 2484 ?

A) 3984

B) 4884

C) 2784

D) 2824

E) 3874

300). 11 30 87 258 771 ?

A) 2610

B) 2450

C) 2310

D) 2730

E) 2510

Solutions:

291). The series is ×1-2, ×2-3, ×3-4, ×4-5, ×5-6, …

Answer: C)

292). The series is ×1+1², ×2+2², ×3+3², ×4+4², ×5+5², …

Answer: B)

293). The series is ×1+2, ×2-4, ×3+6, ×4-8, ×5+10, …

500 Most Important Number Series Questions

www.ibpsguide.com | estore.ibpsguide.com| www.sscexamguide.com

For Free Online Mock Test for IBPS PO/Clerk 2017 – Visit: estore.ibpsguide.com

114 114

Answer: E)

294). ×2+10, ×2-10, ×2+10, ×2-10, …. Repeated

Answer: D)

295).12×2=24, 24×4=96, 96×6=576, 576×8=4608, 4608×10=46080

Answer: B)

296). The series is ×2-1, ×4-3, ×6-5, ×8-7, ×10-9, …

Answer: E)

297).The series is +63, +73, +83, +93, +103, …

Answer: D)

298). The series is

50×0.5=25,

25×1.5=37.5,

37.5×2.5=93.75,

93.75×3.5=328.125,

328.125×4.5=1476.5625

Answer: B)

299). 468+516=984;

516+984=1500;

984+1500=2484;

1500+2484=3984

Answer: A)

300). The series is

11×3-3=30,

30×3-3=87,

87×3-3=258,

258×3-3=771,

771×3-3=2310

500 Most Important Number Series Questions

www.ibpsguide.com | estore.ibpsguide.com| www.sscexamguide.com

For Free Online Mock Test for IBPS PO/Clerk 2017 – Visit: estore.ibpsguide.com

115 115

Answer: C)

Directions (301-310): What should come in the place of question mark in the following

number series?

301). 1224 ? 3022 4221 5620 7219

A) 2165

B) 1986

C) 1976

D) 1875

E) 2023

302). ? 208 501 986 1711 2724

A) 59

B) 63

C) 78

D) 54

E) 67

303). ? 4 17 126 1297 16808

A) 1

B) 2

C) 0

D) -3

E) 3

304). 11 29 ? 89 131 181

A) 59

B) 55

C) 76

D) 64

E) 68

305). 974 1366 1754 2146 2534 ?

500 Most Important Number Series Questions

www.ibpsguide.com | estore.ibpsguide.com| www.sscexamguide.com

For Free Online Mock Test for IBPS PO/Clerk 2017 – Visit: estore.ibpsguide.com

116 116

A) 2926

B) 2929

C) 2976

D) 2956

E) 2946

306). 8835 9023 9213 9405 ? 9795

A) 9899

B) 9599

C) 9539

D) 9509

E) None of these

307). 5477 5626 5777 ? 6085 6242 6401

A) 5830

B) 5960

C) 5840

D) 5950

E) 5930

308). 1716 2184 2730 3360 4080 ? 5814

A) 4896

B) 4876

C) 4796

D) 4696

E) 5096

309). 137 247 411 548 ? 822 959

A) 785

B) 635

C) 735

D) 685

E) 695

500 Most Important Number Series Questions

www.ibpsguide.com | estore.ibpsguide.com| www.sscexamguide.com

For Free Online Mock Test for IBPS PO/Clerk 2017 – Visit: estore.ibpsguide.com

117 117

310). 8 12 18 27 40.5 60.75 ?

A) 81.125

B) 92.125

C) 91.125

D) 94.125

E) 87.275

Solutions:

301). The series is

(35)2 -1, (45)2 -2, (55)2 -3, (65)2 -4, (75)2 -5, …

So the series is 1224, 2023, 3022, 4221, 5620, 7219, ...

Answer: E)

302). The series is 3*4*5-1, 5*6*7-2, 7*8*9-3, 9*10*11-4, 11*12*13-5, 13*14*15-6, …..

Answer: A)

303). The series is (2)0+1, (3)1+1, (4)2+1, (5)3+1, (6)4+1, (7)5+1, …

Answer: B)

304). The series is 3*5-4, 5*7-6, 7*9-8, 9*11-10, 11*13-12, 13*15-14, …

Answer: B)

305). The series is 195*5-1, 195*7+1, 195*9-1, 195*11+1, 195*13-1, 195*15+1, …

Answer: A)

306). The series is (94)2 -1, (95)2 -2, (96)2 -3, (97)2 -4, (98)2 -5, (99)2 -6,

ie, 8835, 9023, 9213, 9405, 9599, 9795

Answer: B)

307). The series is +149, +151, +153, +155, +157, +159, +161, …

Answer: E)

308). The series is (12)3 -12, (13)3-13, (14)3-14, (15)3-15, (16)3-16, (17)3-17, (18)3-18, (19)3-19, …

Answer: A)

500 Most Important Number Series Questions

www.ibpsguide.com | estore.ibpsguide.com| www.sscexamguide.com

For Free Online Mock Test for IBPS PO/Clerk 2017 – Visit: estore.ibpsguide.com

118 118

309). The series is +137, +137, repeated.

Answer: D)

310).c; The series is ×1.5, ×1.5, ×1.5……..

Answer: C)

311). 2 12 36 80 150 ?

A) 194

B) 210

C) 252

D) 258

312). 1, 8, 9, 64, 25, 216, ?, ?

A) 49, 64

B) 343, 64

C) 49, 512

D) 343, 512

313). 3, 13, 53, 213, ?

A) 553

B) 653

C) 753

D) 853

314). 4, 16, 36, ?, 100, 144

A) 72

B) 68

C) 81

D) 64

315). 8, 15, 28, 53, ...?

A) 98

B) 106

500 Most Important Number Series Questions

www.ibpsguide.com | estore.ibpsguide.com| www.sscexamguide.com

For Free Online Mock Test for IBPS PO/Clerk 2017 – Visit: estore.ibpsguide.com

119 119

C) 100

D) 102

316). 6, 12, 21, ?, 48

A) 38

B) 40

C) 45

D) 33

317). 1, 4, 27, 16, ?, 36, 343

A) 87

B) 120

C) 25

D) 125

318). 28, 33, 31, 36, ?, 39

A) 32

B) 34

C) 38

D) 40

319). 165, 195, 255, 285, 345, ?

A) 375

B) 390

C) 420

D) 435

320). 1, 5, 14, 30, 55, 91, ?

A) 130

B) 140

C) 150

D) 160

Solutions for the above aptitude Questions:

500 Most Important Number Series Questions

www.ibpsguide.com | estore.ibpsguide.com| www.sscexamguide.com

For Free Online Mock Test for IBPS PO/Clerk 2017 – Visit: estore.ibpsguide.com

120 120

311). 13 + 12 = 2, 23 + 22 = 12, 33 + 32 = 36 and so on 63 + 62 = 252

Answer: C)

312). The odd terms are squares of the numbers 1, 3, 5, ....while the even terms are cubes of the

numbers 2, 4, 6, ....

Answer: C)

313). The difference in consecutive nos. follows the sequence, 10, 40, 160.

So the next difference = 160 × 4 = 640

Therefore number = 213 + 640 = 853.

Answer: D)

314). The numbers in the series are square of 2,4,6 ...,10,12 .

Therefore, the missing number is square of 8 i.e. 82=64.

Answer: D)

315). 8 × 2 – 1 = 15, 15 × 2 – 2 = 28, 28 × 2 – 3 = 53, 53 × 2 – 4 =102

Answer: D)

316). The common differences between the consecutive terms of the series are 6, 9, 12, 15, etc.

Answer: D)

317). The series is 13, 22, 33, 42, 53, 62, 73.

Answer: D)

318). The pattern is +5, –2, +5, –2, .....

So, missing term = 36 –2 = 34.

Answer: B)

319). Each number of the series is 15 multiplied by a prime number

i.e. 15 × 11, 15 × 13, 15 × 17, 15 × 19, 15 × 23, .....

So, missing term = 15 × 29 = 435.

Answer: D)

320). The pattern is + 4, +9, + 16, + 25, + 36, ..... i.e. + 22, + 32, + 42, + 52, + 62, .....

So missing term = 91 + 72 = 91 + 49 = 140.

Answer: B)

Q(321-6-325) Find the missing number in the given seriese.

321) 2, 7, 10, 22, 18, 37, 26,?

A. 42

500 Most Important Number Series Questions

www.ibpsguide.com | estore.ibpsguide.com| www.sscexamguide.com

For Free Online Mock Test for IBPS PO/Clerk 2017 – Visit: estore.ibpsguide.com

121 121

B. 52

C. 46

D. 62

E.None of these

322) 279936, 46656, 7776, 1296, 216, ?

A. 60

B. 46

C. 36

D. 66

E.None of these

323)12, 38, 116, 350, 1052, ?

A. 1800

B. 2200

C. 2800

D. 3158

E.None of these

324) 46080, 3840, 384, 48, 8, 2, ?

A. 1

B. 1/64

C. 1/8

D.2

E. None of these

325)5,28,57,88,125

A.156

B.147

C.166

D.186

E. None of these

500 Most Important Number Series Questions

www.ibpsguide.com | estore.ibpsguide.com| www.sscexamguide.com

For Free Online Mock Test for IBPS PO/Clerk 2017 – Visit: estore.ibpsguide.com

122 122

321.B

There are two series here

2, 10, 18, 26, ... (Increase by 8)

7, 22, 37, ... (Increase by 15)

Hence, next term is 37+15 = 52

322.C

Go on dividing by 6 to the next number

323.D

12

12 × 3 + 2 = 38

38 × 3 + 2 = 116

116 × 3 + 2 = 350

350 × 3 + 2 = 1052

1052 × 3 + 2 = 3158

324.A

46080 /12 = 3840

3840 /10 = 384

384 /8 = 48

48 /6 = 8

8/4 = 2

2 /2 = 1

325.C

28=23+5

57=29+28

88=31+57

125=37+88

166=41+125

326) 42, 40, 38, 35, 33, 31, 28, ?, ?

A. 25, 22

500 Most Important Number Series Questions

www.ibpsguide.com | estore.ibpsguide.com| www.sscexamguide.com

For Free Online Mock Test for IBPS PO/Clerk 2017 – Visit: estore.ibpsguide.com

123 123

B. 26, 23

C. 26, 24

D. 25, 23

E. 26, 22

327) 8, 12, 9, 13, 10, 14, 11, ?, ?

A. 14, 11

B. 15, 12

C. 8, 15

D. 15, 19

E. 8, 5

328) 36, 31, 29, 24, 22, 17, 15, ?, ?

A. 13, 11

B. 10, 5

C. 13, 8

D. 12, 7

E. 10, 8

329) 3, 5, 35, 10, 12, 35, 17, ?, ?

A. 22, 35

B. 35, 19

C. 19, 35

D. 19, 24

E. 22, 24

330) 13, 29, 15, 26, 17, 23, 19, ?, ?

A. 21, 23

B. 20, 21

C. 20, 17

D. 25, 27

E. 22, 20

331) 14, 14, 26, 26, 38, 38, 50, ?, ?

500 Most Important Number Series Questions

www.ibpsguide.com | estore.ibpsguide.com| www.sscexamguide.com

For Free Online Mock Test for IBPS PO/Clerk 2017 – Visit: estore.ibpsguide.com

124 124

A. 60, 72

B. 50, 62

C. 50, 72

D. 62, 62

E. 62, 80

332) 32, 31, 32, 29, 32, 27, 32, ?, ?

A. 25, 32

B. 31, 32

C. 29, 32

D. 25, 30

E. 29, 30

333) 83, 73, 93, 63, __, 93, 43,

A. 33

B. 53

C. 73

D. 93

334) 15, __, 27, 27, 39, 39

A. 51

B. 39

C. 23

D. 15

335) 72, 76, 73, 77, 74, __, 75

A. 70

B. 71

C. 75

D. 78

336) F2, __, D8, C16, B32

A. A16

B. G4

500 Most Important Number Series Questions

www.ibpsguide.com | estore.ibpsguide.com| www.sscexamguide.com

For Free Online Mock Test for IBPS PO/Clerk 2017 – Visit: estore.ibpsguide.com

125 125

C. E4

D. E3

337) 664, 332, 340, 170, ____, 89

A. 85

B. 97

C. 109

D. 178

338) 70, 71, 76, __, 81, 86, 70, 91

A. 70

B. 71

C. 80

D. 96

339) 8, 43, 11, 41, __, 39, 17

A. 8

B. 14

C. 43

D. 44

340) 10, 6, 12, 35, ?, 591.75

(A) 130

(B) 129.5

(C) 127.25

(D) 133

(E) None of these

326) Option C

Explanation: This is an alternating subtraction series in which 2 is subtracted twice, then 3 is

subtracted once, then 2 is subtracted twice, and so on.

327) Option B

Explanation: This is an alternating addition and subtraction series, in which the addition of 4 is

500 Most Important Number Series Questions

www.ibpsguide.com | estore.ibpsguide.com| www.sscexamguide.com

For Free Online Mock Test for IBPS PO/Clerk 2017 – Visit: estore.ibpsguide.com

126 126

alternated with the subtraction of 3.

328) Option E

Explanation: This is an alternating subtraction series, which subtracts 5, then 2, then 5, and so

on.

329) Option C

Explanation: This is an alternating addition series, with a random number, 35, interpolated as

every third number. The pattern of addition is to add 2, add 5, add 2, and so on. The number 35

comes after each "add 2" step.

330) Option B

Explanation: Here, there are two alternating patterns, with every other number following a

different pattern. The first pattern begins with 13 and adds 2 to each number to arrive at the

next; the alternating pattern begins with 29 and subtracts 3 each time.

331) Option B

Explanation: In this simple addition with repetition series, each number in the series repeats

itself, and then increases by 12 to arrive at the next number.

332)OptionA

Explanation: This is an alternating repetition series. The number 32 alternates with a series in

which each number decreases by 2.

333) Option B

Explanation: This is a simple subtraction series in which a random number, 93, is interpolated as

every third number. In the subtraction series, 10 is subtracted from each number to arrive at the

next.

334) Option D

Explanation: In this simple addition with repetition series, each number in the series repeats

itself, and then increases by 12 to arrive at the next number.

335) Option D

500 Most Important Number Series Questions

www.ibpsguide.com | estore.ibpsguide.com| www.sscexamguide.com

For Free Online Mock Test for IBPS PO/Clerk 2017 – Visit: estore.ibpsguide.com

127 127

Explanation: This series alternates the addition of 4 with the subtraction of 3.

336) Option C

Explanation: The letters decrease by 1; the numbers are multiplied by 2.

337) Option D

Explanation: This is an alternating division and addition series: First, divide by 2, and then add 8.

338) Option A

Explanation: In this series, 5 is added to the previous number; the number 70 is inserted as every

third number.

339) Option B

Explanation: This is a simple alternating addition and subtraction series. The first series begins

with 8 and adds 3; the second begins with 43 and subtracts 2.

340)Option B

Directions (1-10): What will come in place of the question marks (?) in the following Number

series?

341. 0, 6, 24, 60, 120, 210, ?

A. 336

B. 349

C. 312

D. 337

E. None of these

342. 11, 14, 19, 22, 27, 30, ?

A. 39

B. 34

C. 36

D. 35

E. None of these

500 Most Important Number Series Questions

www.ibpsguide.com | estore.ibpsguide.com| www.sscexamguide.com

For Free Online Mock Test for IBPS PO/Clerk 2017 – Visit: estore.ibpsguide.com

128 128

343. 6, 12, 21, ? , 48

A. 33

B. 39

C. 36

D. 31

E. None of these

344. 18, 22, 30, ?,78, 142

A. 44

B. 35

C. 46

D. 48

E. None of these

345. 73205, 6655, 605, 55, ?

A. 9

B. 5

C. 13

D. 11

E. None of these

346. 25, 100, ?, 1600, 6400

A. 400

B. 300

C. 360

D. 420

E. None of these

347. 125, ?, 343, 512, 729, 1000

A. 216

B. 215

C. 256

D. 225

500 Most Important Number Series Questions

www.ibpsguide.com | estore.ibpsguide.com| www.sscexamguide.com

For Free Online Mock Test for IBPS PO/Clerk 2017 – Visit: estore.ibpsguide.com

129 129

E. None of these

348. 1 , 9 , 125 , 343 , ? , 1331

A. 730

B. 729

C. 512

D. 772

E. None of these

349. 121, 144, 169, ?, 225

A. 180

B. 172

C. 186

D. 196

E. None of these

350. ?, 2116, 2209, 2304, 2401, 2500

A. 2124

B. 1972

C. 1521

D. 2025

E. None of these

341. (A)

The given series is : 13 – 1, 23 – 2, 33 – 3, 43 – 4, 53 – 5, 63 – 6,

So the missing term = 73 – 7 = 343 – 7 = 336 .

342. (D)

The pattern is + 3, + 5, + 3, + 5, …………

So the missing term is = 30 + 5 = 35 .

343. (A)

The pattern is + 6, + 9, + 12, +15 ………..

So the missing term is = 21 + 12 = 33 .

500 Most Important Number Series Questions

www.ibpsguide.com | estore.ibpsguide.com| www.sscexamguide.com

For Free Online Mock Test for IBPS PO/Clerk 2017 – Visit: estore.ibpsguide.com

130 130

344. (C)

The pattern is +4, +8, +16, +32, +64

So the missing term is = 30 + 16 = 46 .

345. (B)

5 x 11 = 55, 55 x 11 = 605, 605 x 11 = 6655, 6655 x 11 = 73205

346. (A)

25 x 4 = 100, 100 x 4 = 400, 400 x 4 = 1600, 1600 x 4 = 6400.

347. (A)

125 = 53 , 216 = 63, 343 = 73, 512 = 83, 729 = 93, 1000 = 103.

348. (B)

13 , 33 , 53 , 73 , 93 , 113

349. (D)

121 = 112, 144 = 122, 169 = 132, 196 = 142, 225 = 152.

350. (D)

2025 = 452, 2116 = 462, 2304 = 482, 2401 = 492, 2500 = 502

(351-360) What will come in place of the question mark (?) in the following number series.

351) 100, 100, 119, ?, 299, 562

A) 170

B) 176

C) 174

D) 184

E) 164

352) 3, 4, 12, 45, ?, 1005

A) 196

B) 109

C) 99

500 Most Important Number Series Questions

www.ibpsguide.com | estore.ibpsguide.com| www.sscexamguide.com

For Free Online Mock Test for IBPS PO/Clerk 2017 – Visit: estore.ibpsguide.com

131 131

D) 110

E) 182

353) 1, 3, 9, 31, ?, 651

A) 127

B) 129

C) 131

D) 120

E) 130

354) 5, 12, 26, 54, ?, 222

A) 109

B) 110

C) 112

D) 111

E) None of these

355) 8, 11, 20, ?, 128, 371

A) 55

B) 51

C) 49

D) 48

E) 47

356) 3, 10, 20, 39, 85, ?

A) 105

B) 121

C) 212

D) 127

E) 129

357) 4, 31, 156, 499, 1228, ?

A) 1525

B) 2559

C) 2232

D) 2560

E) 2533

358) 152, 156, 140, 176, 112, ?

500 Most Important Number Series Questions

www.ibpsguide.com | estore.ibpsguide.com| www.sscexamguide.com

For Free Online Mock Test for IBPS PO/Clerk 2017 – Visit: estore.ibpsguide.com

132 132

A) 212

B) 112

C) 152

D) 198

E) 232

359) 7, 9, 21, 67, 273, ?

A) 1171

B) 1272

C) 1225

D) 1371

E) 1470

360) 6, 4, 5, 11,?, 361

A) 55

B) 51

C) 49

D) 48

E) 45

351) C)

The number series pattern is *2-102, *2 – 92, *2 -82…….,

? = 119 *2 – 82 = 174

352) A)

*1 + 12, *2 + 22, *3+32, *4+42……

? = 45*4+16 = 196

353) B)

*1+2, *2+3, *3+4,…..*5+6

? = 31*4+5 = 129

354) B)

+7, +14, +28, +56……

? = 54+56 = 110

500 Most Important Number Series Questions

www.ibpsguide.com | estore.ibpsguide.com| www.sscexamguide.com

For Free Online Mock Test for IBPS PO/Clerk 2017 – Visit: estore.ibpsguide.com

133 133

355) E)

+3, +9, +27, +81, 243

? = 47

356) C)

First difference of numbers i.e.

357) B)

+33 , +53, +73, +93, 113

? = 1228 + 1331 = 2559

358) A)

+22, +42, +62, +82…..

? = 112 + 100 = 212

359) D)

*1+2, *2+3, *3+4…….

? = 273 * 5+6 = 1371

360) E)

*0.5+1, *1+1, *2+1, *4+1….

? = 45 * 8+1 = 361

Directions (361-370): Find the missing number:

361) 11, 13, 17, 19, 23, 29, 31, 37, 41, (…)

(A) 43

(B) 47

(C) 53

500 Most Important Number Series Questions

www.ibpsguide.com | estore.ibpsguide.com| www.sscexamguide.com

For Free Online Mock Test for IBPS PO/Clerk 2017 – Visit: estore.ibpsguide.com

134 134

(D) 51

(E) None of these

362) 15, 31, 63, 127, 255, (…)

(A) 513

(B) 511

(C) 517

(D) 523

(E) None of these

363) 8, 24, 12, 36, 18, 54, (…)

(A) 27

(B) 108

(C) 68

(D) 72

(E) None of these

364) 7, 26, 63, 124, 215, 342, (…)

(A) 481

(B) 511

(C) 391

(D) 421

(E) None of these

365) 2, 4, 12, 48, 240, (…)

(A) 960

(B) 1440

(C) 1080

(D) 1920

(E) None of these

366) 100 100 119 ? 299 562

(A) 184

(B) 220

500 Most Important Number Series Questions

www.ibpsguide.com | estore.ibpsguide.com| www.sscexamguide.com

For Free Online Mock Test for IBPS PO/Clerk 2017 – Visit: estore.ibpsguide.com

135 135

(C) 240

(D) 174

(E) None of these

367) 3 4 12 45 ? 1005

(A) 152

(B) 198

(C) 144

(D) 192

(E) None of these

368) 1 3 9 31 ? 651

(A) 97

(B) 127

(C) 129

(D) 109

(E) None of these

369) 5 12 36 123 ? 2555 15342

(A) 508

(B) 381

(C) 504

(D) 635

(E) None of these

370) 8 11 17 ? 65 165.5 498.5

(A) 27.5

(B) 32

(C) 28

(D) 30.5

(E) None of these

361) A

Sol. Numbers are all primes. The next prime is 43.

500 Most Important Number Series Questions

www.ibpsguide.com | estore.ibpsguide.com| www.sscexamguide.com

For Free Online Mock Test for IBPS PO/Clerk 2017 – Visit: estore.ibpsguide.com

136 136

362) B

Sol. Each number is double the preceding one plus 1.

So, the next number is (255 × 2) + 1 = 511.

363) A

Sol. Numbers are alternately multiplied by 3 and divided by 2.

So, the next number = 54 ÷ 2 = 27.

364) B

365) B

Sol. Go on multiplying the given numbers by 2, 3, 4, 5, 6. So, the correct next number is 1440.

366) D

Sol. The Series is:

× 2 - 102, × 2 - 92, × 2 - 82, × 2 - 72, × 2 - 62

367) E

Sol. The Series is:

× 1 + 12, × 2 + 22, × 3 + 32……

368) C

Sol. The Series is:

× 1 + 2, × 2 + 3, × 3 + 4, × 4 + 5

369) A

Sol. The pattern of the Series is:

5x1 + 1x7 = 12

12x2 + 2x6 = 36

36x3 + 3x5 = 123

123x4 + 4x4 = 508

500 Most Important Number Series Questions

www.ibpsguide.com | estore.ibpsguide.com| www.sscexamguide.com

For Free Online Mock Test for IBPS PO/Clerk 2017 – Visit: estore.ibpsguide.com

137 137

370) D

Sol. The pattern of the Series is:

8x0.5 + 7 = 11

11x1 + 6 = 17

17x1.5 + 5 = 30.5

30.5x2 + 4 = 65

Direction (371-380): What will be the come in place of question mark (?) in the following

number series?

371)5 7.5 15 37.5 112.5 ?

(A) 340

(B) 395.5

(C) 393.75

(D) 397.25

(E) 339

372)66 35 72 38 78 ?

(A) 39

(B) 158

(C) 37

(D) 41

(E) 40

373)9 5 6 10.5 23 ?

(A) 30

(B) 48

(C) 60

(D) 69

(E) 65

374)2 3 6 18 108 ?

(A) 2000

(B) 1953

(C) 1928

500 Most Important Number Series Questions

www.ibpsguide.com | estore.ibpsguide.com| www.sscexamguide.com

For Free Online Mock Test for IBPS PO/Clerk 2017 – Visit: estore.ibpsguide.com

138 138

(D) 1944

(E) 1900

375)6 20 83 419 2519 ?

(A) 18924

(B) 19230

(C) 16510

(D) 17892

(E) 17639

376) 680 655 704 623 744 ?

A) 575

B) 675

C) 600

D) 913

E) None of these

377) 10 10 20 ? 110 300 930

A) 40

B) 35

C) 45

D) 50

E) 60

378)1 ? 22 188 2052 28748

A) 2

B) 3

C) 6

D) 16

E) 10

379) 50327 7169 1215 223 76 ?

A) 1

B) 2

500 Most Important Number Series Questions

www.ibpsguide.com | estore.ibpsguide.com| www.sscexamguide.com

For Free Online Mock Test for IBPS PO/Clerk 2017 – Visit: estore.ibpsguide.com

139 139

C) 3

D) 4

E) 8

380) 14 34 62 ? 142 194

A) 94

B) 98

C) 108

D) 112

E) None of these

371) Ans.(C)

Sol. The series is:

5×1.5, 7.5×2, 15×2.5, 37.5×3, 112.5×3.5

372) Ans.(D)

Sol. The series is:

66, 72, 78….. & 35, 38, 41

373) Ans.(C)

Sol. The series is:

9× ½ + ½ , 5×1 + 1, 6×1.5 + 1.5, 10.5×2 + 2, 23×2.5 + 2.5

374) Ans.(D)

Sol. The series is:

2×3, 3×6, 18×6, 108×18 = 1944

375) Ans.(E)

Sol. The series is:

6×3 + 2, 20×4 + 3, 83×5 + 4, 419×6 + 5, 2519×7 + 6

376) A

680 - 5^2 = 655

655 + 7^2 = 704

500 Most Important Number Series Questions

www.ibpsguide.com | estore.ibpsguide.com| www.sscexamguide.com

For Free Online Mock Test for IBPS PO/Clerk 2017 – Visit: estore.ibpsguide.com

140 140

704 - 9^2 = 623

623 + 11^2 = 744

744 - 13^2 = 575

377) C

10 × 0.5 + 5 = 10

10 × 1 + 10 = 20

20 × 1.5 + 15 = 45

45 × 2 + 20 = 110

110 × 2.5 + 25 = 300

300 × 3 + 30 = 930

378) C

1 × 2 + 4 = 6

6 × 5 - 8 = 22

22 × 8 + 12 = 188

188 × 11 - 16 = 2052

2052 × 14 + 20 = 28748

379) D

(50327 – 144) ÷ 7 = 7169

(7169 + 121) ÷ 6 = 1215

(1215 – 100) ÷ 5 = 223

(223 + 81) ÷ 4 = 76

(76 – 64) ÷ 3 = 4

380) B

2 + 3 × 4 = 14

4 + 5 × 6 = 34

6 + 7 × 8 = 62

8 + 9 × 10 = 98

10 + 11 × 12 = 142

12 + 13 × 14 = 194

500 Most Important Number Series Questions

www.ibpsguide.com | estore.ibpsguide.com| www.sscexamguide.com

For Free Online Mock Test for IBPS PO/Clerk 2017 – Visit: estore.ibpsguide.com

141 141

Direction(381-390): What would come in place of the question mark (?) in the following

number series?

381) 7, 28, 63, ?, 215, 344

a. 121

b. 163

c. 126

d. 134

e. 156

382) 441, 429, 454, 442, 467, ?

a. 421

b. 455

c. 445

d. 492

e. 437

383) 38, 50, 65, 81, 96, 108 ?

a. 38

b. 50

c. 82

d. 96

e. 108

384) 18, 41, 112, 327, 974, ?

a. 2917

b. 1260

c. 1458

d. 1267

e. 1563

385) 0, 1, 5, 41, ?, 3263441

a. 1841

b. 1830

c. 1820

500 Most Important Number Series Questions

www.ibpsguide.com | estore.ibpsguide.com| www.sscexamguide.com

For Free Online Mock Test for IBPS PO/Clerk 2017 – Visit: estore.ibpsguide.com

142 142

d. 1876

e. 1805

386) 7, 9, 21, 67, ?, 1371, 8233

a. 273

b. 268

c. 337

d. 341

e. 409

387) 6, 16, 57 , 244, 1245, ?, 52591

a. 7515

b. 7506

c. 7445

d. 7816

e. 7726

388) 24, 36, 90, 315, ?, 7796.25

a. 1415

b. 1517

c. 1525.5

d. 1335.5

e. 1417.5

389) 2, 3, 8, 30, 144, ?

a. 720

b. 640

c. 240

d. 840

e. 480

390) 408, 120, 312, ?, 72, 120

a. 148

b. 184

500 Most Important Number Series Questions

www.ibpsguide.com | estore.ibpsguide.com| www.sscexamguide.com

For Free Online Mock Test for IBPS PO/Clerk 2017 – Visit: estore.ibpsguide.com

143 143

c. 168

d. 124

e. None of these

381) 23 – 1 = 7

33 + 1 = 28

43– 1 = 63

53 + 1 = 126

63– 1 = 215

73 + 1 = 344

Hence, option c.

382) 441-12 = 429

429 +25= 454

454-12=442

442+25 =467

467-12= 455

Hence, option b

383) 31+7*1 = 38

38+6*2 = 50

50+5*3 = 65

65+4*4 = 81

81+3*5 = 96

96+2*6 = 108

Hence, option c.

384) 6+ 6*2= 18

5 + 18*2 = 41

4 + 54*2 = 112

3 + 162*2 =327

2 + 486*2 = 974

1 + 1458*2 = 2917

Hence, option a.

500 Most Important Number Series Questions

www.ibpsguide.com | estore.ibpsguide.com| www.sscexamguide.com

For Free Online Mock Test for IBPS PO/Clerk 2017 – Visit: estore.ibpsguide.com

144 144

385) 0 +12= 1

1+22 = 5

5+62= 41

41+ 422 = 1805

1805 + 18062 = 3263441

Hence, option e.

386) 7 × 1 + 2 = 9, 9 × 2 + 3 = 21

21 × 3 + 4 = 67, 67 × 4 + 5 = 273

273 × 5 + 6 = 1371, 1371 × 6 + 7 = 8233

Hence, option a

387) 6 × 2 + 22 = 16, 16 × 3 + 32 = 57

57 × 4 + 42 = 244, 244 × 5 + 52 = 1245

1245 × 6 + 62 = 7506, 7506 × 7 + 72 = 52591

Hence, option b

388) 24 × 1.5 = 36, 36 × 2.5 = 90

90 × 3.5 = 315, 315 × 4.5 = 1417.5

1417.5 × 5.5 = 7796.25

Hence, option e.

389) 0! + 1! = 2, 2! + 1! = 3

3! + 2! = 8, 4! + 3! = 30

5! + 4! = 144, 6! + 5! = 840

Hence, option d.

390) 372 – 312 = 408, 312 – 292 = 120

292 – 232 = 312, 232 – 192 = 168

192 – 172 = 72, 172 – 132 = 120

Hence, option c.

500 Most Important Number Series Questions

www.ibpsguide.com | estore.ibpsguide.com| www.sscexamguide.com

For Free Online Mock Test for IBPS PO/Clerk 2017 – Visit: estore.ibpsguide.com

145 145

Direction(391-400). In the given number series, what would come in place of the (?)

question mark?

391) 28, 41, ?, 696, 5816

a. 120

b. 146

c. 129

d. 132

e. 210

392) 195, 167, 141, 117, 95, ?, 57

a. 80

b. 75

c. 82

d. 81

e. 79

393) 114, 39, -18, ?, -41

a. 32

b. 35

c. 38

d. -35

e. -38

394) 343, 441, 512, 576, 729, 729, ?

a. 841

b. 824

c. 1000

d. 1331

e. 900

395) -20, -16, -7, 18, 67, ? , 357

a. 126

b. 144

c. 188

500 Most Important Number Series Questions

www.ibpsguide.com | estore.ibpsguide.com| www.sscexamguide.com

For Free Online Mock Test for IBPS PO/Clerk 2017 – Visit: estore.ibpsguide.com

146 146

d. 208

e. 257

396) 0, 10, 24, 68, ?

a. 90

b. 105

c. 120

d. 130

e. 145

397) 2500, 2275, 2106, ?, 1904

a. 1975

b. 1985

c. 1990

d. 1995

e. None of these

398) 11, 42, 71,?,113,130

a. 93

b. 96

c. 95

d. 94

e. None of these

399) 33, 47, 63, ?, 101, 123

a. 78

b. 76

c. 81

d. 83

e. 87

400) 47, 112, 118,184, ?, 330

a. 265

b. 227

500 Most Important Number Series Questions

www.ibpsguide.com | estore.ibpsguide.com| www.sscexamguide.com

For Free Online Mock Test for IBPS PO/Clerk 2017 – Visit: estore.ibpsguide.com

147 147

c. 189

d. 295

e. None of these

391) The series follows the pattern

28+13*(1)2= 41, 41+11*(2)3 = 129, 129 + 7*(3)4 = 696, 696 + 5*(4)5 = 5816

Hence, option c

392) The series follows the pattern

142 – 1 = 195, 132 – 2 = 167, 122 – 3 = 141, 112 – 4 = 117, 102 – 5 = 95, 92 – 6 = 75

82 – 7 = 57

Hence, option b

393) The series follows the pattern

5! – (1+2+3) = 114, 4! + (4+5+6) = 39, 3! – (7+8+9) = -18, 2! + (10+11+12) = 35, 1! –

(13+14+15) = -41

Hence, option b.

394) The series follows the pattern

343 = 73, 441 = (7*3)2, 512 = 83, 576 = (8*3)2, 729 = 93, 729 = (9*3)2, 1000 = 103

Hence, option c.

395) The series follows the pattern

-20 + 22 = -16, -16 + 32 = -7, -7 + 52 =18, 18 + 72 = 18 + 49 = 67, 67 + 112 = 67 + 121 = 188, 188

+132 = 188 +169 = 357

Hence, option c

396) 0 = 13-1,

10 = 23+2,

24 = 33– 3

and so on

So missing no. will be 53-5 = 120

Hence, option c

500 Most Important Number Series Questions

www.ibpsguide.com | estore.ibpsguide.com| www.sscexamguide.com

For Free Online Mock Test for IBPS PO/Clerk 2017 – Visit: estore.ibpsguide.com

148 148

397) The series follows the pattern:

2500-152 = 2275, 2275-132 = 2106, 2106-112 = 1985, 1985-92 = 1904

Hence, option b

398) The series follows the pattern

11+31 = 42, 42+29 = 71, 71+23 = 94, 94+19 = 113, 113+17 = 130

Hence, option d

399)The series follows the pattern:

33 + 14 = 47, 47 + 16 = 63, 63 + 18 = 81, 81 + 20 = 101, 101 + 22 = 123

Hence, option c.

400) The series follows the pattern:

47 + 42 + 72 = 112, 112 + 12 + 12 + 22 = 118, 118 + 12 + 12 + 82 = 184, 184 + 12 + 82 + 42 = 265,

265 +22 + 62 + 52 = 330

Hence, option a

What would come in place of the question mark (?) in the following number series?

401) 13, 64, 248, 717, 1370, ?

a. 1394

b. 1379

c. 1245

d. 1296

e. 1398

402) 9, 15, 26, 46, 85, ?

a. 151

b. 155

c. 163

d. 174

e. 180

403) 5, 1, 5, 25, 157, ?

a. 1516

500 Most Important Number Series Questions

www.ibpsguide.com | estore.ibpsguide.com| www.sscexamguide.com

For Free Online Mock Test for IBPS PO/Clerk 2017 – Visit: estore.ibpsguide.com

149 149

b. 1558

c. 1432

d. 1265

e. 1213

404) 42, 78, 303, 339, ?, 789

a. 564

b. 432

c. 386

d. 271

e. 621

405) 68, 69, 73, 100, ?

a. 156

b. 107

c. 132

d. 116

e. 124

406) 111, 138, 238, 302, 423, ?

a. 532

b. 523

c. 548

d. 623

e. 638

407) 31, 29, 54, 154, 600, ?

a. 1696

b. 2024

c. 2025

d. 2116

e. 2968

500 Most Important Number Series Questions

www.ibpsguide.com | estore.ibpsguide.com| www.sscexamguide.com

For Free Online Mock Test for IBPS PO/Clerk 2017 – Visit: estore.ibpsguide.com

150 150

408) 4, 13, 49, 130, ?

a. 245

b. 264

c. 274

d. 215

e. 166

409) 16, 48, -3, 69, ?

a. -5

b. 12

c. -14

d. -26

e. None of these

410) 23, 26, 44, 64, ?

a. 92

b. 84

c. 79

d. 95

e. 67

401) The series follows the pattern

13*5 -13 = 64, 64*4 -23 = 248, 248*3 -33 = 717, 717*2 – 43 = 1370, hence 1370*1 -53 = 1245

Hence, option c.

402) The next term is obtained by adding the immediate prime number smaller than the given

number and subtracting consecutive natural numbers from them.

9+7-1=15, 15+13-2=26, 26+23-3=46, 46+43-5=85, hence 85+83-5= 163

Hence, option c.

403) The series follows the pattern

5*0 + 1 = 1, 1*2 + 3 = 5, 5*4 + 5 = 25, 25*6 + 7 = 157, 157*8 + 9 = 1265

Hence, option d.

500 Most Important Number Series Questions

www.ibpsguide.com | estore.ibpsguide.com| www.sscexamguide.com

For Free Online Mock Test for IBPS PO/Clerk 2017 – Visit: estore.ibpsguide.com

151 151

404) The series follows the pattern

42 + (4 + 2)2 = 78, 78 + (7 + 8)2 = 303, 303 + (3 + 0 + 3)2 = 339, 339 + (3 + 3 + 9)2 = 564, 564 +

(5 + 6 + 4)2 = 789

Hence, option a

405) The series follows the pattern

68 + 13 = 69, 69 + 22 = 73, 73 + 33 = 100, 100 + 42 = 116

Hence, option d.

406) The series follows the pattern

111 + 33 = 138, 138 + 102 = 238, 238 + 43 = 302, 302 + 112 = 423, 423 + 53 = 548

Hence, option c.

407) The series follows the pattern

(31*1) – 2 = 29, (29*2) – 4 = 54, (54*3) – 8 = 154, (154*4) – 16 = 600, (600*5) – 32 = 2968

Hence, option e.

408) The series follows the pattern

4 + 32 = 13, 13 + 62 = 49, 49 + 92 = 130, 130 + 122 = 274

Hence, option c.

409) The series follows the pattern

16 + 16*2 = 48, 48 – 17*3 = -3, -3 + 18*4 = 69, 69 – 19*5 = -26

Hence, option d

410) The last digit of the previous term is multiplied by consecutive odd numbers to get the next

term

23 + 3 * 1 = 26, 26 + 6 * 3 = 44, 44 + 4 * 5 = 64, 64 + 4 * 7 = 92

Hence, option a.

Direction (411 – 42): What will come in place of (?) in the following number series.

411) 3, 11, 54, 339, 2732,?

a) 27356

b) 22852

c) 21650

d) 27375

500 Most Important Number Series Questions

www.ibpsguide.com | estore.ibpsguide.com| www.sscexamguide.com

For Free Online Mock Test for IBPS PO/Clerk 2017 – Visit: estore.ibpsguide.com

152 152

e) 27345

412) 358, 320, 284, 250,?

a) 218

b) 258

c) 236

d) 264

e) 278

413) 8, 22, 80, 364, ?

a) 4805

b) 2010

c) 2007

d) 2560

e) 2700

414) 7, 22, 118, 1107, ?

a) 17756

b) 24334

c) 17779

d) 17772

e) 28986

415) 8, 37, 104, 233, 454, ?

a) 338

b) 823

c) 837

d) 803

e) 934

416) 12, 19, 35, 59, 90, ?

a) 132

b) 126

c) 127

d) None of these

e) 146

417). 4, 10, 27, 112, 555, ?

a) 3786

b) 4756

500 Most Important Number Series Questions

www.ibpsguide.com | estore.ibpsguide.com| www.sscexamguide.com

For Free Online Mock Test for IBPS PO/Clerk 2017 – Visit: estore.ibpsguide.com

153 153

c) None of these

d) 3336

e) 3686

418) 6,14,45,184,925 ?

a) 4596

b) 5556

c) 6756

d) 6556

e) None of these

419) 11,20,38,74,?

a) 164

b) 146

c) 182

d) 150

e) None of these

420) 15,21,38,65,101, ?

a) 150

b) 130

c) 155

d) 145

e) None of these

411). The Series is x2+5, x4+10, x6+15, x8+20, x10+25

412).The Series is 19^2-3,18^2-4, 17^2-5, 16^2-6, 15^2-7

413). Series is x2.5+2, x3.5+3, x4.5+4, x5.5+5

414).Series is x1^2+15, x2^2+30, x3^2+45, x4^2+60

415).Difference between no. is, 3^3+2, 4^3+3, 5^3+4, 6^3+5, 7^3+6

146

416). The difference of difference of the series is, 9, 8, 7,….(i.e. 16 – 7 = 9; 24 – 16 = 8…..)

19 – 12 = 7

35 – 19 = 16

59 – 35 = 24…

417).

The series is,

4 * 2 + 2 = 10

500 Most Important Number Series Questions

www.ibpsguide.com | estore.ibpsguide.com| www.sscexamguide.com

For Free Online Mock Test for IBPS PO/Clerk 2017 – Visit: estore.ibpsguide.com

154 154

10 * 3 – 3 = 27

27 * 4 + 4 = 112…

418).

The series is,

6 * 2 + 2 = 14

14 * 3 + 3 = 45

45 * 4 + 4 = 184…..

419)The series is

20 – 11 = 9

38 – 20 = 18

74 – 38 = 36….

420) The difference of difference of the series is, 11, 10, 9,….(i.e. 17-6 = 11; 27-17 = 10….)

21 – 15 = 6

38 – 21 = 17

65 – 38 = 27…

Directions(421-430): Find the missing number in the given series:

421) 13, 7, 6, 14, 52, _

A. 482

B. 476

C. 466

D. 424

E. 460

422)139, 142, 133, 160, 79, _

A. 326

B. 322

C. 331

D. 340

E. 355

423) 10, 16, 27, 48, 84, _

A. 138

B. 153

500 Most Important Number Series Questions

www.ibpsguide.com | estore.ibpsguide.com| www.sscexamguide.com

For Free Online Mock Test for IBPS PO/Clerk 2017 – Visit: estore.ibpsguide.com

155 155

C. 166

D. 140

E. 135

424) 66, 77, 63, 73, 60, _

A. 66

B. 69

C. 61

D. 64

E. 63

425) 16, 9, 8, 13, 25, _

A. 64.5

B. 59.5

C. 63.5

D. 68.5

E. 61.5

426) 11, 12, 22, 69, 272, _

A. 1365

B. 1368

C. 1324

D. 1325

E. 1329

427) 11, 13, 16, 33, 96, _

A. 358

B. 354

C. 386

D. 366

E. 353

428) 4, 21, 120, 595, 2376, _

A. 7125

500 Most Important Number Series Questions

www.ibpsguide.com | estore.ibpsguide.com| www.sscexamguide.com

For Free Online Mock Test for IBPS PO/Clerk 2017 – Visit: estore.ibpsguide.com

156 156

B. 7035

C. 7085

D. 7055

E. 7115

429) 64, 66, 75, 103, 168, _

A. 282

B. 294

C. 286

D. 295

E. 288

430) 13, 25, 39, 57, 83, _

A. 122

B. 124

C. 126

D. 120

E. 125

421) Sol. D.424

13 * 0.5 + 0.5 = 7

7 * 1 – 1 = 6

6 * 2 + 2 = 14

422) Sol. B.322

139 + 3 = 142

142 – 9 = 133

133 + 27 = 160

423) Sol. D

16 – 10 = 6

27 – 16 = 11

48 – 27 = 21; 11 – 6 = 5, 21 – 11 = 10

500 Most Important Number Series Questions

www.ibpsguide.com | estore.ibpsguide.com| www.sscexamguide.com

For Free Online Mock Test for IBPS PO/Clerk 2017 – Visit: estore.ibpsguide.com

157 157

424) Sol. B

66 – 63 = 3

77 – 73 = 4

63 – 60 = 3

425) Sol. C

16 * 0.5 + 1 = 9

9 * 1 – 1 = 8

8 * 1.5 + 1 = 13

426) Sol. A

11 * 1 + 1 = 12

12 * 2 – 2 = 22

22 * 3 + 3 = 69

427) Sol. E

11 + 1² + 1= 13

13 + 2² – 1= 16

16 + 4² + 1= 33

428) Sol. A

4*7 -7 = 21

21*6 -6 = 120

120*5 -5 = 595

429) Sol. B

64 + (1³ + 1) = 66

66 + (2³ + 1) = 75

75 + (3³ + 1) = 103

430) Sol. E

13*2 – 1 = 25

25*2 – 11 = 39

39*2 – 21 = 57

500 Most Important Number Series Questions

www.ibpsguide.com | estore.ibpsguide.com| www.sscexamguide.com

For Free Online Mock Test for IBPS PO/Clerk 2017 – Visit: estore.ibpsguide.com

158 158

Directions (431-440) : What should come in place of question mark (?) in the following

number series ?

431)13 14 30 93 376 1885 ?

(A) 10818

(B) 10316

(C) 11316

(D) 11318

(E) None of these

432)4 6 9 13.5 20.25 30.375 ?

(A) 40.25

(B) 45.5625

(C) 42.7525

(D) 48.5625

(E) None of these

433)400 240 144 86.4 51.84 31.104 ?

(A) 19.2466

(B) 17.2244

(C) 16.8824

(D) 18.6624

(E) None of these

434)9 4.5 4.5 6.75 13.5 33.75 ?

(A) 101.25

(B) 103.75

(C) 99.75

(D) 105.50

(E) None of these

435)705 728 774 843 935 1050 ?

(A) 1190

(B) 1180

500 Most Important Number Series Questions

www.ibpsguide.com | estore.ibpsguide.com| www.sscexamguide.com

For Free Online Mock Test for IBPS PO/Clerk 2017 – Visit: estore.ibpsguide.com

159 159

(C) 1185

(D) 1187

(E) None of these

436) 2916, 972, ?,108, 36, 12

a) 324

b) 248

c) 234

d) 391

e) None of these

437). 8, 27, 125, ?, 1331

a) 81

b) 216

c) 512

d) 343

e) 169

438) 3, 22, 7, 45, 15,?, 31

a) 91

b) 90

c) 151

d) 121

e) 5

439) . 3, 15, ?, 51, 53, 159, 161

a) 27

b) 35

c) 17

d) 24

e) None of these

440). 5963, ?, 5938, 5913, 5877, 5828

a) 5932

b) 5940

c) 5951

d) 5954

e) None of these

500 Most Important Number Series Questions

www.ibpsguide.com | estore.ibpsguide.com| www.sscexamguide.com

For Free Online Mock Test for IBPS PO/Clerk 2017 – Visit: estore.ibpsguide.com

160 160

SOLUTION:

431) Ans.(C)

Sol. The given number series is based on the following pattern :

13 × 1 + 1 = 14

14 × 2 + 2 = 30

30 × 3 + 3 = 93

93 × 4 + 4 = 376

376 × 5 + 5 = 1885

? = 1885 × 6 + 6 = 11316

Hence, number 11316 will replace the question mark.

432) Ans.(B) )

Sol. The given series is based on the following pattern :×1.5,×1.5,……..

433) Ans.(D)

Sol. The given series is based on the following pattern : ×0.6,×0.6……..

434) Ans.(A)

Sol. The given series is based on the following pattern : ×0.5,×1,×1.5,×2,×2.5…….

435) Ans.(E)

Sol.

705 + 1 × 23 = 728

728 + 2 × 23 = 774

774 + 3 × 23 = 843

843 + 4 × 23 = 935

935 + 5 × 23 = 1050

? = 1050 + 6 × 23 = 1050 + 138 = 1188

436) Explanation:

2916 ÷ 3 = 972

972 ÷ 3 = 324

324 ÷ 3 = 108

108 ÷ 3 = 36

500 Most Important Number Series Questions

www.ibpsguide.com | estore.ibpsguide.com| www.sscexamguide.com

For Free Online Mock Test for IBPS PO/Clerk 2017 – Visit: estore.ibpsguide.com

161 161

36 ÷ 3 = 12

437) Explanation:

The pattern of the given series is:

⇒ Cube of prime numbers: ⇒

2^3 = 8

3^3 = 27

5^3 = 125

7^3 = 343

11^3 = 1331

438)Here, two alternate series are going together with the pattern given below

7 = 3 × 2 + 1

45 = 22 × 2 + 1

15 = 7 × 2 + 1

31 = 15 × 2 + 1

∴The number in the place of question mark = 45 × 2 + 1 = 91

439)We have the terms of the given series formed as

15 = 5 × 3

? = 15 + 2 = 17

51 = 17 × 3

53 = 51 + 2

159 = 53 × 3

161 = 159 + 2

440)

Explanation:

5963 – 3^2 = 5954

5954 – 4^2 = 5938

5938 – 5^2 = 5913

5913 – 6^2 = 5877

5877 – 7^2 = 5828

500 Most Important Number Series Questions

www.ibpsguide.com | estore.ibpsguide.com| www.sscexamguide.com

For Free Online Mock Test for IBPS PO/Clerk 2017 – Visit: estore.ibpsguide.com

162 162

Directions (441-450) What will come in place of question mark (?) in the given number

series ?

441)18 96 161 213 252 ?

A) 278

B) 291

C) 382

D) 362

E) 560

442)512 128 32 8 2 ?

A) 1/32

B) 1/16

C) 1/4

D) 1/2

E) 1/8

443)178 537.5 ? 18858.5 169736 1867107.5

A) 2694

B) 2693

C) 3892

D) 2574

E) 2486

444)68921 79507 91125 103823 117649 ?

A) 267752

B) 267254

C) 267755

D) 132651

E) 267782

445) 5 12.5 54.5 333.5 2676.5 ? ?

A) 26775

B) 26725.5

C) 26775.5

500 Most Important Number Series Questions

www.ibpsguide.com | estore.ibpsguide.com| www.sscexamguide.com

For Free Online Mock Test for IBPS PO/Clerk 2017 – Visit: estore.ibpsguide.com

163 163

D) 26785.5

E) 26778

446)19 22 28 ? 49 64

A) 42

B) 37

C) 36

D) 45

E) 40

447)48 47 92 273 ? 5435

A) 1020

B) 1050

C) 1100

D) 1088

E) 1540

448)576 599 ? 645 484 691

A) 530

B) 315

C) 360

D) 560

E) 358

449)64 25 216 49 512 ?

A) 100

B) 81

C) 136

D) 140

E) 135

450)776 ? 772 785 768 787

A) 760

B) 750

500 Most Important Number Series Questions

www.ibpsguide.com | estore.ibpsguide.com| www.sscexamguide.com

For Free Online Mock Test for IBPS PO/Clerk 2017 – Visit: estore.ibpsguide.com

164 164

C) 985

D) 783

E) 776

441) A-278)

The series is +78/+65/+52…..

442)D-1/2)

The pattern is divided by 4

443) B-2693)

The series is *3+3.5/*5+5.5/…..

444) D-132651)

The series is cube of 41/43/45/47

445) C-26775.5)

The pattern is *2+2.5/*4+4.5/*6+6.5/……

446) B-37)

The series is +3/+6/+9/……

447) D-1088)

The series is *1-1/*2-2/*3-3/…….

448) A-530)

The series is +23*1/-23*3/+23*5/

449) B-81)

The series is cube & square 4/5/6 /…….

450) D-783)

The series is +7/-11/+13/-17/+19

500 Most Important Number Series Questions

www.ibpsguide.com | estore.ibpsguide.com| www.sscexamguide.com

For Free Online Mock Test for IBPS PO/Clerk 2017 – Visit: estore.ibpsguide.com

165 165

Directions (451-460) What will come in place of question mark (?) in the given number

series ?

451) 6 3 4.5 11.25 39.375 ?

A) 589

B) 534

C) 634

D) 530

E) None of these

452) 45 5678 ? 155 210

A) 110

B) 112

C) 90

D) 111

E) 115

453)292 144 74 35 19.5 ? 5.875

A) 11.25

B) 9.5

C) 7.75

D) 8.25

E) 6.135

454)740 ? 181 86.5 38.25 13.125

A) 364

B) 368

C) 366

D) 378

E) 374

455)13 10.8 15.4 8.2 17.8 ?

A) 5.8

B) 5.6

C) 6.2

500 Most Important Number Series Questions

www.ibpsguide.com | estore.ibpsguide.com| www.sscexamguide.com

For Free Online Mock Test for IBPS PO/Clerk 2017 – Visit: estore.ibpsguide.com

166 166

D) 7.2

E) 6.8

456)14 12.4 15.6 ? 17.2 9.2

A) 10.8

B) 12

C) 12.8

D) 14.6

E) 18.6

457) 7 10.5 21 ? 157.5 551.25

A) 42.5

B) 52

C) 63

D) 52.5

E) 62.5

458)93 95.8 99.8 105 ? 119

A) 110.4

B) 111.4

C) 110.8

D) 112.4

E) 111.8

459) 2 3 7.5 26.25 118.125 ?

A) 648.6875

B) 649.6875

C) 650.6875

D) 675.6875

E) None of these

460) 5 7.25 13.5 25.75 46 ?

A) 70.25

B) 71.25

500 Most Important Number Series Questions

www.ibpsguide.com | estore.ibpsguide.com| www.sscexamguide.com

For Free Online Mock Test for IBPS PO/Clerk 2017 – Visit: estore.ibpsguide.com

167 167

C) 73.25

D) 75.25

E) 76.25

451) E- 117.1875)

The series is * .5/*1.5/*2.5/……..

452) D-111.9)

The pattern is +11/+22/+33/+44/…..

453) C-7.75)

The series is -4÷2/+4÷2/……

454) B-368)

The series is (740*.5)-2=368/(368 *.5)-3=181

455) B-5.6)

The pattern is combination of two series 13 + 2.4/10.8-2.6/

456) A-10.8)

The series is 14 +1.6*-1=12.4/12.4 +1.6 *2=15.6/15.6 +1.6*-3=10.8/

457) D-52.5)

The series is *1.5 /*2/*2.5/*3/…..

458) B-111.4)

The series is 93 +2.8=95.8/95.8 +4=99.8/99.8 +5.2=105/…….

459) B-649.6875)

The series is (2 *1.5)=3/(3*2.5=7.5/……

460) E-76.25)

The series is +square of 1.5/+square of 2.5/+ square of 3.5/…..

500 Most Important Number Series Questions

www.ibpsguide.com | estore.ibpsguide.com| www.sscexamguide.com

For Free Online Mock Test for IBPS PO/Clerk 2017 – Visit: estore.ibpsguide.com

168 168

Directions (461-470) What will come in place of question mark (?) in the given number

series ?

461)8835 9023 9213 9405 ? 9795

A) 9278

B) 9599

C) 9382

D) 9362

E) 9560

462) 1716 2184 2730 3360 ? 4896

A) 4080

B) 4216

C) 4400

D) 4378

E) 4234

463) 2197 3375 4913 6859 ? 12167

A) 9361

B) 10648

C) 9216

D) 9261

E) None of these

464) 68921 79507 91125 103823 117649 ?

A) 267752

B) 267254

C) 267755

D) 132651

E) 267782

465)3249 3969 4225 4761 5625 ?

A) 5929

B) 5939

C) 5949

500 Most Important Number Series Questions

www.ibpsguide.com | estore.ibpsguide.com| www.sscexamguide.com

For Free Online Mock Test for IBPS PO/Clerk 2017 – Visit: estore.ibpsguide.com

169 169

D) 5959

E) 4929

466) 5477 5626 5777 ? 6085 6242

A) 5842

B) 5937

C) 5736

D) 5645

E) 5930

467) 46656 1156 32768 ? 21952 676

A) 861

B) 900

C) 2700

D) 961

E) 841

468)4032 4290 4556 4830 5112 ?

A) 5329

B) 5402

C) 5302

D) 5529

E) 5482

469)1546 1731 1924 2125 2334 ?

A) 2521

B) 2531

C) 2541

D) 2551

E) 2561

470)1543 1440 1337 ? 1131

A) 1233

B) 1234

500 Most Important Number Series Questions

www.ibpsguide.com | estore.ibpsguide.com| www.sscexamguide.com

For Free Online Mock Test for IBPS PO/Clerk 2017 – Visit: estore.ibpsguide.com

170 170

C) 1235

D) 1238

E) 1239

461) B-9599)

The series is Sq of 94 -1/sq of 95-2/……..

462) A-4080)

The pattern is cube of 12-12/cube of 13-13/…..

463) D-9261)

The series is cube of 13/cube of 15/cube of 17/……

464) D-132651)

The series is cube of 41/43/45/47

465) A-5929)

The pattern is Square of consecutive odd numbers but not prime numbers

Sq of 57/sq of 63/sq of 65/sq of 69/sq of 75/sq of 77

466) E-5930)

The series is +149/+151/+153/…….

467) B-900)

The series is mixed .square &cube .cube of 36/sq of 34/cube of 32/sq of 30/

468) B-5402)

The series is Sq of 63 + 63/sq of 65 + 65/sq of 67 + 67/…..

469) D-2551)

The series is sq of 39 +25/sq of 41 +50/sq of 43 +75/…..

470) B-1234)

The series is subtract 103 from all the numbers

500 Most Important Number Series Questions

www.ibpsguide.com | estore.ibpsguide.com| www.sscexamguide.com

For Free Online Mock Test for IBPS PO/Clerk 2017 – Visit: estore.ibpsguide.com

171 171

Directions (471-780) What will come in place of question mark (?) in the given number

series ?

471) 3 5 13 49 241 ?

A) 1430

B) 1348

C) 1228

D) 1441

E) 1334

472) 7 13 31 85 247 ?

A) 365

B) 335

C) 733

D) 285

E) 295

473) 5 7 17 47 115 ?

A) 285

B) 245

C) 195

D) 299

E) 88

474) 508 256 130 67 35.5 ?

A) 18.72

B) 19.75

C) 16.66

D) 15.54

E) 14.84

475) 17 9 15 40 143.5 ?

A) 435

B) 700

C) 650.25

500 Most Important Number Series Questions

www.ibpsguide.com | estore.ibpsguide.com| www.sscexamguide.com

For Free Online Mock Test for IBPS PO/Clerk 2017 – Visit: estore.ibpsguide.com

172 172

D) 578

E) 678.5

476) 5120 1280 320 80 ?

A) 25

B) 20

C) 28

D) 40

E) 57

477) 8 17 42 91 ?

A) 122

B) 167

C) 142

D) 172

E) 187

478) 8 7 12 33 128 ?

A) 635

B) 528

C) 372

D) 402

E) 692

479) 982 977 952 827 822 ?

A) 632

B) 698

C) 625

D) 538

E) 797

480)11 23 47 95 191 ?

A) 351

B) 329

500 Most Important Number Series Questions

www.ibpsguide.com | estore.ibpsguide.com| www.sscexamguide.com

For Free Online Mock Test for IBPS PO/Clerk 2017 – Visit: estore.ibpsguide.com

173 173

C) 213

D) 383

E) 221

471).D)

The series is * 2-1/&3-2/…..

472).C)

The series is *3-8 /*3-8/ ….

473).B)

The series is difference is -Cube of any number + same number

7-5=2=cube of 1 + 1/17-7=10=cube of 2+2/……

474).B)

The series is ÷2 +2/ …….

475).C)

The series is *.5 + .5/*1.5 + 1.5/*2.5 +2.5/ ……….

476).B)

The series is ÷ 4/÷4/….

477).D)

The series is + square of number + square of 3/+ square of 5/…..

478).A)

The series is*1-1/*2-2/*3-3/….

479).E)

The series is -5/-25/-125/-5/-25/…..

480).D)

The series is *2+1/*2+1/*2+1/……..

500 Most Important Number Series Questions

www.ibpsguide.com | estore.ibpsguide.com| www.sscexamguide.com

For Free Online Mock Test for IBPS PO/Clerk 2017 – Visit: estore.ibpsguide.com

174 174

Directions (481-490) What will come in place of question mark (?) in the given number

series ?

481) 2 4 10 ? 82 244

A) 30

B) 48

C) 28

D) 46

E) 34

482) 33 16.5 ? 24.75 49.5 123.75

A) 16.5

B) 13.5

C) 22.5

D) 20.5

E) 12.5

483) 20 23 30 43 64 ?

A) 85

B) 92

C) 95

D) 99

E) 88

484) 44 ? 99 148.5 222.75 334.125

A) 72

B) 77

C) 66

D) 54

E) 84

485). 7 8 4 13 -3 22 ?

A) -7

B) -10

500 Most Important Number Series Questions

www.ibpsguide.com | estore.ibpsguide.com| www.sscexamguide.com

For Free Online Mock Test for IBPS PO/Clerk 2017 – Visit: estore.ibpsguide.com

175 175

C) -12

D) -14

E) -9

486). 124 215 342 511 ? 999

A) 695

B) 625

C) 728

D) 806

E) 573

487). 516 256 126 61 28.5 ?

A) 12.25

B) 16.75

C) 14.25

D) 20.20

E) 18.75

488). 7 21 50 109 ? 467

A) 198

B) 228

C) 372

D) 402

E) 275

489). 18 82 118 134 138 ?

A) 132

B) 198

C) 125

D) 138

E) 92

490). 5 22 103 406 ? 2422

A) 1516

500 Most Important Number Series Questions

www.ibpsguide.com | estore.ibpsguide.com| www.sscexamguide.com

For Free Online Mock Test for IBPS PO/Clerk 2017 – Visit: estore.ibpsguide.com

176 176

B) 1298

C) 1136

D) 1602

E) 1213

481). C)

The series is *3-2 , *3-2 ,……..

482). A)

The series is *5 , *1 , *1.5 , ….

483). C)

+ 3 , + 7, +13 , +21, + 31

+4 +6 +8 +10

484). C)

The series is *1.5 , *1.5 , …….

485). D)

The series is + square of 1 , – square of 2 , ……….

486). C)

The series is cube of number -1

Cube of 5 – 1 = 124 , cube of 6-1= 215 , …..

487). A)

The series is 516-4 ÷2=256 , 256-4 ÷2=126 , …….

488). B)

The series is*2 + 7 , *2 +8 ,…….

489). D)

500 Most Important Number Series Questions

www.ibpsguide.com | estore.ibpsguide.com| www.sscexamguide.com

For Free Online Mock Test for IBPS PO/Clerk 2017 – Visit: estore.ibpsguide.com

177 177

The series is + square of number

+ square of 8 , + square of 6 , …………..

490). E)

The series is

*6-8/*5-7/*4-6/……………..

491). 150 102 70 46 26 ?

A) 7

B) 13

C) 8

E) 2

E) 12

492). 10 14 24 52 134

A) 351

B) 302

C) 368

D) 341

E) 378

493). 24 11 10 14 27 ?

A) 67.5

B) 60.5

C) 66.5

D) 61.5

E) 62.55

494). 4500 900 90 6 ? 0.012

A) 0.3

B) 0.09

C) 0.9

D) 0.015

E) 0.03

500 Most Important Number Series Questions

www.ibpsguide.com | estore.ibpsguide.com| www.sscexamguide.com

For Free Online Mock Test for IBPS PO/Clerk 2017 – Visit: estore.ibpsguide.com

178 178

495). 8 7 12 33 128 ?

A) 672

B) 684

C) 635

D) 620

E) 692

496). 16 17 21 30 46 ?

A) 82

B) 104

C) 71

D) 92

E) 84

497). 2 2 4 12 48 ?

A) 180

B) 220

C) 240

D) 160

E) 210

498). 10 14 23 37 56 ?

A) 74

B) 80

C) 118

D) 120

E) 94

499). 4 7 13 25 49 ?

A) 118

B) 136

C) 86

500 Most Important Number Series Questions

www.ibpsguide.com | estore.ibpsguide.com| www.sscexamguide.com

For Free Online Mock Test for IBPS PO/Clerk 2017 – Visit: estore.ibpsguide.com

179 179

D) 97

E) 124

500). 50 59 41 68 32 ?

a) 77

b) 45

c) 20

d) 85

e) 60

491) C

150-102=48

102-70=32

70-46=24

46-26=20

26-8=18

492) E

10+(4)=14

14 + (10)=24

24 +(28)=52

52+(82)=134

134 + (244)=378

493) C

24*(1/2)-1=11

11*1-1=10

10*(3/2)-1=14

So on

494) A

4500/5=900

900/10=90

90/15=6

500 Most Important Number Series Questions

www.ibpsguide.com | estore.ibpsguide.com| www.sscexamguide.com

For Free Online Mock Test for IBPS PO/Clerk 2017 – Visit: estore.ibpsguide.com

180 180

6/20=.3

.3/.25=.012

495) C

8*1-1=7

7*2-2=12

So on

496) C

16 + (12)=17

17 + (22)=21

So on

497) C

2 *1=2

3*2=6

4*3=12

So on

498) B

10 + (4)=14

14+(9)=23

23+(14)=37

So on

499) D

4 *2-1=7

7*2-1=13

So on

500) A

50 + (9)=59

59 -(18)=41

41+ (27)=68

500 Most Important Number Series Questions

www.ibpsguide.com | estore.ibpsguide.com| www.sscexamguide.com

For Free Online Mock Test for IBPS PO/Clerk 2017 – Visit: estore.ibpsguide.com

181 181

68-(36)=32

So on.

Number Series

1) 5, 25, 7,______, 9, 19

a) 23

b) 22

c) 25

d) 32

e) None of The Above

Solution (Option B)

2) 62, 64,______, 32, 14,16

a) 26

b) 28

c) 30

d) 32

e) None of The Above

Solution (Option)

3) 100, 50, 52, 26, 28,______

a) 30

b) 32

c) 14

d) 16

e) None of The Above

Solution (Option C)

4) 980, 392, 156.8,______, 25.088, 10.0352

a) 62.72

b) 63.85

c) 65.04

d) 60.28

e) None of The Above

Solution (Option A)

5) 113, 225, 449,_____, 1793

a) 789

b) 786

c) 897

d) 987

e) None of The Above

Solution (Option C)

6) 5, 9, 21, 37, 81,_____

a) 163

b) 153

c) 181

www.BankExamsToday.com

Number Series

d) 203

e) None of The Above

Solution (Option B)

7) 2, 10, 30, 68,______

a) 126

b) 130

c) 140

d) 150

e) None of The Above

Solution (Option B)

8) 45, 46, 70, 141,_______, 1061.5

a) 353

b) 353.5

c) 352.5

d) 352

e) None of The Above

Solution (Option B)

9) 33, 321, 465, 537, 573_______

a) 600

b) 591

c) 585

d) 498

e) None of The Above

Solution (Option B)

10) 2, 9,_______, 105, 436, 2195

a) 25

b) 27

c) 30

d) 33

e) None of The Above

Solution (Option C)

11) 4 6 14 44 ? 892

www.BankExamsToday.com

Number Series

a) 176

b) 172

c) 178

d) 1821

e) None of The Above

Solution (Option C)

12) 126 64 34 20 14 ?

a) 12

b) 14

c) 16

d) 18

e) None of The Above

Solution (Option A)

13) 16 8 12 30 ? 472.5

a) 100

b) 105

c) 205

d) 300

e) None of The Above

Solution (Option B)

14) 25 34 30 39 35 ?

a) 45

b) 44

c) 46

d) 50

e) None of The Above

Solution (Option B)

www.BankExamsToday.com

Number Series

15) 10, 11, 13, 21, 69, ?

a) 384

b) 490

c) 453

d) 390

e) None of The Above

Solution (Option C)

16) 5, ........., 8, 56, 7, 63, 9?

a) 45

b) 35

c) 40

d) 42

e) None of The Above

Solution (Option C)

17) 32, ........, 92, 134, 184?

a) 55

b) 38

c) 45

d) 58

e) None of The Above

Solution (Option D)

18) 11, 16, 26, ..........., 86?

a) 46

b) 56

c) 76

d) 86

e) None of The Above

Solution (Option A)

19) 198, 194, 185, 169, .......?

a) 154

b) 165

c) 144

d) 134

e) None of The Above

Solution (Option C)

www.BankExamsToday.com

Number Series

20) 9050, 5675, 3478, 2147, .......?

a) 3478

b) 1418

c) 2428

d) 3678

e) None of The Above

Solution (Option B)

21) 5, 17, 37, 65, ........ , 145.

a) 95

b) 99

c) 97

d) 101

e) None of The Above

Solution (Option D)

22) 95, 115.5, 138, ......., 189.

a) 154.5

b) 164.5

c) 162.5

d) 166.5

e) None of The Above

Solution (Option C)

23) 3, 4.5, 9, 22.5, 67.5, ........., 945

a) 265.25

b) 236.25

c) 225.36

d) 150

e) None of The Above

Solution (Option B)

24) 8544, 1420, 280, ..........., 18, 5

a) 33

b) 44

c) 56

d) 66

e) None of The Above

Solution (Option D)

25) 812, 398, ..........., 90, 40, 16

www.BankExamsToday.com

Number Series

a) 192

b) 182

c) 172

d) 162

e) None of The Above

Solution (Option A)

26) 6072, ..........., 200, 48, 14, 5, 3

a) 1008

b) 1010

c) 1001

d) 1005

e) None of The Above

Solution (Option B)

27) 200, 165, 148, .........., 104, 77, 68

a) 115

b) 116

c) 117

d) 118

e) None of The Above

Solution (Option C)

28) 445, 221, 109, ............, 25, 11, 4

a) 46

b) 57

c) 53

d) 59

e) None of The Above

Solution (Option C)

29) -1/2, 0, 1/2 , .........., 3/2, 2

a) 1/2

b) 2/3

www.BankExamsToday.com

Number Series

c) 3/4

d) 1

e) None of The Above

Solution (Option D)

30) 3, 9, 41, 113, ........., 577

a) 260

b) 265

c) 270

d) 275

e) None of The Above

Solution (Option B)

31) 7, 15, ..........., 106, 231

a) 32

b) 45

c) 42

d) 64

e) None of The Above

Solution (Option C)

32) 8, .............., 41, 74, 118

a) 29

b) 15

c) 22

d) 19

e) None of The Above

Solution (Option D)

33) 13, ........., 15, 21, 33

a) 12

b) 11

c) 13

d) 14

e) None of The Above

Solution (Option C)

34) 7, ..........., 17, 42, 91

a) 8

www.BankExamsToday.com

Number Series

b) 7

c) 9

d) 12

e) None of The Above

Solution (Option A)

35) .........., 11, 24, 75, 304

a) 8

b) 5

c) 10

d) 9

e) None of The Above

Solution (Option C)

36) 2, 9, .........., 65, 126

a) 25

b) 22

c) 28

d) 30

e) None of The Above

Solution (Option C)

37) 0, .............., 68, 222, 520

a) 3

b) 6

c) 9

d) 10

e) None of The Above

Solution (Option D)

38) 3, ............, 12, 38, 154

a) 9

b) 12

c) 4

d) 5

e) None of The Above

Solution (Option D)

39) 9, 22, 24, 37, ..............,

52

a) 39

b) 42

c) 45

d) 50

e) None of The Above

Solution (Option A)

www.BankExamsToday.com

Number Series

40) 4, 4, .............., 12, 30

a) 5

b) 6

c) 8

d) 10

e) None of The Above

Solution (Option B)

41) 2348, 3437, 4346, ............., 6344, 7433

a) 5425

b) 5436

c) 5456

d) 5435

e) None of The Above

Solution (Option D)

42) 87, 86, 82, 73, 57, 32, ...........

a) -5

b) -4

c) 6

d) 10

e) None of The Above

Solution (Option B)

43) 12, 20, 19, 26, ........, 31, 30

a) 24

b) 26

c) 25

d) 35

e) None of The Above

Solution (Option C)

44) 4, 8, 11, 22, 18, .........., 25, 50

a) 36

b) 35

c) 46

d) 56

e) None of The Above

Solution (Option A)

www.BankExamsToday.com

Number Series

45) 9, 7, 64, 6, 3, .............., 1,

8, 9

a) 28

b) 16

c) 18

d) 19

e) None of The Above

Solution (Option D)

46) 51975, 9450, 2100, ........, 240, 160,

320

a) 600

b) 800

c) 900

d) 1000

e) None of The Above

Solution (Option A)

47) 2, 9, 30, 105, 436, 2195,

...........

a) 13184

b) 13186

c) 13175

d) 13182

e) None of The Above

Solution (Option D)

48) 4, 2.5, 3.5, 6.75, 15.5, 41.25, ..........

a) 126.75

b) 136.75

c) 166.75

d) 175.75

e) None of The Above

Solution (Option A)

www.BankExamsToday.com

Number Series

49) 1, 5, 9, 17, 25, ........., 49

a) 35

b) 45

c) 37

d) 42

e) None of The Above

Solution (Option C)

50) 2, 10, 30, ......., 130

a) 64

b) 65

c) 68

d) 69

e) None of The Above

Solution (Option C)

51) 3, 12,______,6912

a) 192

b) 194

c) 196

d) 212

e) None of The Above

Solution (Option A)

52) ______, 108, 18, 6

a) 960

b) 900

c) 918

d) 972

e) None of The Above

Solution (Option D)

www.BankExamsToday.com

Number Series

53) 6, _____ 18, 56, 226

a) 10

b) 12

c) 8

d) 15

e) None of The Above

Solution (Option C)

54) 8, _______,218 , 515

a) 64

b) 66

c) 65

d) 68

e) None of The Above

Solution (Option C)

55) 37.5, ______, 450, 900

a) 125

b) 150

c) 200

d) 250

e) None of The Above

Solution (Option B)

56) ______, 16, 24, 48

a) 9

b) 8

c) 16

d) 10

e) None of The Above

Solution (Option C)

www.BankExamsToday.com

Number Series

57) 50, 40, _______, 14

a) 30

b) 28

c) 20

d) 60

e) None of The Above

Solution (Option B)

58) ______, 7, 16, 41

a)3

b)5

c) 6

d) 8

e) None of The Above

Solution (Option C)

59) 2, 10, -6, _______

a) 12

b) 10

c) 28

d) 18

e) None of The Above

Solution (Option D)

60) ______, 4, 1.6, 0.64

a) 5

b) 10

c) 15

d) 20

e) None of The Above

Solution (Option B)

5, _______, 15, 37.5, 112.5

a) 5.5

b) 7.5

c) 3.5

d) 9.5

www.BankExamsToday.com

Number Series

e) None of The Above

Solution (Option B)

61) 3, 5, 14, ______, 107

a) 42

b) 28

c) 30

d) 16

e) None of The Above

Solution (Option A)

62) 5, ______, 10, 30, 120

a) 4

b) 5

c) 6

d) 10

e) None of The Above

Solution (Option B)

63) ______, 11, 20, 45, 94

a) 5

b) 10

c) 8

d) 6

e) None of The Above

Solution (Option B)

64) ______, 85, 185, 235, 435

a) 25

b) 20

c) 35

d) 10

e) None of The Above

Solution (Option C)

65) 10, _____, 64, 129, 388

www.BankExamsToday.com

Number Series

a) 18

b) 21

c) 15

d) 30

e) None of The Above

Solution (Option B)

66) 1,______, 24, 528, 16896

a) 2

b) 8

c) 5

d) 10

e) None of The Above

Solution (Option A)

67) 5, ______, 6, 15, 56

a) 2

b) 3

c) 4

d) 1

e) None of The Above

Solution (Option C)

68) 5, _______, 21, 37, 81

a) 7

b) 9

c) 10

d) 15

e) None of The Above

Solution (Option B)

69) 6072, _______, 200, 48, 14

a) 1010

b) 1000

c) 1072

d) 985

e) None of The Above

Solution (Option A)

70) 3, 3, 4.5, .........., 22.5, 67.5 ?

www.BankExamsToday.com

Number Series

a) 13.5

b) 8.5

c) 9.0

d) 10.5

e) None of The Above

Answer (Option C)

71) 7200, 3600, 1200, ........., 60, 10 ?

a) 200

b) 250

c) 350

d) 300

e) None of The Above

Answer (Option D)

72) .........., 21, 43, 87, 175?

a) 8

b) 5

c) 9

d)10

e) None of The Above

Answer (Option D)

73) 5, 8, 19, ........, 243?

a) 50

b) 60

c) 70

d) 90

e) None of The Above

Answer (Option B)

74) 45, ..........., 184, 371, 746?

a) 75

b) 60

c) 91

d) 124

e) None of The Above

www.BankExamsToday.com

Number Series

Answer (Option C)

75) 15, 27, 51, ........, 195?

a) 99

b) 100

c) 97

d) 95

e) None of The Above

Answer (Option A)

76) ........, 23, 44, 85, 166?

a) 8

b) 12

c) 11

d) 10

e) None of The Above

Answer (Option B)

77) ........., 98, 292, 1166, 5828?

a) 45

b) 50

c) 55

d) 10

e) None of The Above

Answer (Option B)

78) 16, .........., 24, 12, 36?

a) 20

b) 18

c) 19

d) 8

e) None of The Above

Answer (Option D)

79) ........., 9, 25, 49, 81?

a) 2

b) 4

www.BankExamsToday.com

Number Series

c) 1

d) 8

e) None of The Above

Answer (Option C)

80) 15, 30, 60, 120, 240, ?

Answer

81) 360, 180, 90, 45, 22.5, ?

Answer

82) 12, 12, 24, 72, 288, ?

Answer:

83) 3, 6, 18, 72, ?

Sol:

84) 51975, 9450, 2100, 600, 240, 160, ?

Sol:

85) 3, 4.5, ? , 6.75, 10.125, 15.1875

Sol:

86) 45, 46, 70, 141, ? , 1061.5

Sol:

87) 50, 60, 75, 97.5, ? , 184.275, 267.19875

Sol:

88) 6, 9, 12, 15, 18, ?

Sol:

89) 4, 5, 7, 10, 14, 19, ?

Answer:

www.BankExamsToday.com

Number Series

90) 3.5, 7, 10.5, 14, ?

Answer

91) 32, 58, 92, 134, ?

Answer

92) 24, 60, 120, 210, ?

Answer

93) 2, 3.5, 5, 6.5, 8, ?

Answer

94) 8, 16, 28, 44, ?

Answer

95) 4, 11, 30, 67, 128, ?

Answer

96) 3, 128, 6, 64, 9, ? , 12, 16, 15, 8

Answer

97) 13, 41, 85, 145, ?

Answer

98) 0, 5, 18, 43, 84, 145, ?

Answer

www.BankExamsToday.com

Number Series Workbook V-1

www.BankExamsToday.Com Page 1

Direction (1-5) : What value should come in place of

question mark (?) in the following number series?

1. 22, 42, 64, 88, ?

1. 112

2. 118

3. 116

4. 114

5. 115

2. 11, 61, 299, 1189, ?

1. 3559

2. 3659

3. 3569

4. 3549

5. 3459

3. 215, 19, 163, 63, ?

1. 117

2. 127

3. 125

4. 126

5. 109

4. 160, 80, 120, 300, ?

1. 1050

2. 1000

3. 1040

4. 1020

5. 1060

5. 4, 5, 8, 15, ?

1. 25

2. 26

3. 28

4. 31

5. 24

6. 19, 27, 0 , 64, ? , 155

1. 61

2. 83

3. -23

4. 47

5. Other than the given options

7. 122, 62, 32, ?, 9.5, 5.75

1. 19

2. 24

3. 17

4. 20.25

5. Other than the given options

8. 49, 216 , 625 , 1024 , 729 , ?

1. 128

2. 512

3. 256

4. 324

5. Other than the given options

9. 71 , ? , 868, 4345, 26076

1. 322

2. 264

3. 198

4. 216

5. Other than the given options

10. 1 , 12 , 144 , 1728 , ?

1. 18024

Number Series Workbook V-1

www.BankExamsToday.Com Page 2

2. 9962

3. 20736

4. 21302

5. Other than the given options

Directions (11-15): What should come in place of the

question mark (?) in the following number series?

11. 8, 4.5 , 5.5 , 13 , 56 , ?

a) 566

b) 496

c) 596

d) 450

e) 456

12. 19 , 16 , 44 , 107 , ?

a) 108

b) 156

c) 215

d) 151

e) 251

13. 11 , 14 , 23 , 50, ?

a) 111

b) 121

c) 151

d) 131

e) 141

14. 19 , 25 , 42 ,71 , 113 , ?

a) 169

b) 153

c) 186

d) 196

e) 269

15. 21 , 35 , 30 , 44 , 39 ,?

a) 59

b) 53

c) 55

d) 45

e) 46

Directions (16-20): What should come in place of the

question mark (?) in the following number series?

16. 7 , 14 , 30, 56 , 93 , ?

a) 142

b) 403

c) 124

d) 96

e) 124

17. 23 , 39 , 32, 48 , 41 , ?

a) 58

b) 57

c) 59

d) 48

e) 84

18. 11, 13 , 20 , 48 , 111 , ?

a) 237

b) 125

c) 273

d) 255

e) 555

19. 13 , 17 , 33 , 97 , ? , 1377

Number Series Workbook V-1

www.BankExamsToday.Com Page 3

a) 259

b) 563

c) 535

d) 455

e) 353

20. 6 , 3.5 , 4.5 , 11 , 48 , ?

a) 96

b) 56

c) 392

d) 192

e) 292

Directions (21-25): What should come in place of the

question mark (?) in the following number series?

21. 6, 16, 45 ,184 , 917 , ?

a) 5056

b) 5506

c) 5006

d) 5060

e) 6050

22. 11, 20, 38, 74, ?

a) 85

b) 96

c) 100

d) 136

e) 146

23. 15, 21 , 38 , 65 , 101 , ?

a) 150

b) 120

c) 125

d) 145

e) 154

24. 24, 28 , 19 , 35 , 10 , ?

a) 45

b) 44

c) 46

d) 42

e) 47

25. 14 , 6 , 4 , 4, 8 , ?

a) 32

b) 8

c) 4

d) 16

e) 14

Directions (26-30): What should come in place of the

question mark (?) in the following number series?

26. 14, 25, 47, 91, ?, 355

a) 100

b) 197

c) 179

d) 335

e) 155

27. 11, 24, 44, 70, 101, ?

a) 136

b) 102

c) 80

d) 102

e) 163

Number Series Workbook V-1

www.BankExamsToday.Com Page 4

28. 18, 8, 6, 8, 24, ?

a) 6

b) 48

c) 24

d) 176

e) 167

29. 28, 32, 23, 39, 14, ?

a) 30

b) 50

c) 55

d) 6

e) 14

30. 5, 12, 33, 136, 675, ?

a) 5569

b) 4426

c) 5046

d) 4065

e) 4056

Directions (31 - 33): Complete the following series.

31. 2, 4, 12, 4, 240, ?

a) 960

b) 1440

c) 1080

d) 1920

e) None of these

32. 2, 5, 9, 19, 37, ?

a) 76

b) 74

c) 75

d) 73

e) None of these

33. 4, -8, 16, -32, 64, ?

a) 128

b) -128

c) 192

d) -192

e) None of these

Directions (34 -3 5): Find the wrong term in the

following given series.

34. 2, 9, 28, 65, 126, 216, 344

a) 2

b) 28

c) 65

d) 126

e) 216

35. 10, 26, 74, 218, 654, 1946, 5834

a) 26

b) 74

c) 218

d) 654

e) 1946

Directions (36-40): What will come in place of

question mark (?) in the following number series?

36. 59.76, 58.66, 56.46, 52.06, ?, 25.66

a) 48.08

b) 46.53

c) 43.46

d) 43.26

Number Series Workbook V-1

www.BankExamsToday.Com Page 5

e) None of these

37. 36, 157, 301, 470, ?, 891

a) 646

b) 695

c) 639

d) 669

e) None of these

38. 14, 70, 350, ?, 8750, 43750

a) 1570

b) 875

c) 1750

d) 785

e) None of these

39. 13, 13, 65, 585, 7605, 129285,?

a) 2456415

b) 2235675

c) 2980565

d) 2714985

e) 2197845

40. 1, 16, 81, 256, 625, 1296, ?

a) 4096

b) 2401

c) 1764

d) 3136

e) 6561

Directions (41-45): What will come in place of

question mark (?) in the following number series?

41. 1, 7, 49, 343, (?)

a) 16807

b) 1227

c) 2058

d) 2401

e) None of these

42. 13, 20, 39, 78, 145, (?)

a) 234

b) 244

c) 236

d) 248

e) None of these

43. 12, 35 , 81 , 173 , 357 , (?)

a) 725

b) 715

c) 726

d) 736

e) None of these

44. 3, 100, 297, 594, 991, (?)

a) 1489

b) 1479

c) 1478

d) 1498

e) None of these

45. 112, 119, 140, 175, 224, (?)

a) 277

b) 276

c) 287

d) 266

e) None of these

Number Series Workbook V-1

www.BankExamsToday.Com Page 6

Directions (46 -50): In the following number series

only one number is wrong. Find out the wrong

number.

46. 18.3, 20.6 , 16, 22.9 , 13.7, 2.2, 11.4

a) 25.2

b) 18.3

c) 13.7

d) 22.9

e) 20.6

47. 9 , 5 , 6 , 10.5 , 23 , 61, 183

a) 183

b) 10.5

c) 61

d) 5

e) 9

48. 188, 154, 140, 132 ,128, 126 , 125

a) 125

b) 154

c) 132

d) 126

e) 188

49. 2, 4 , 11 , 37 , 151 , 771 , 4633

a) 11

b) 4633

c) 771

d) 151

e) 2

50. 391 , 394 , 399 , 411 , 431 , 461, 503

a) 503

b) 394

c) 399

d) 431

e) 391

Directions (51-55): What should come in place of the

question mark (?) in the following number series?

51. 150, 102, 70 , 46 , 26 , ?

a) 16

b) 8

c) 10

d) 2

e) 4

52. 10 , 14 , 28 , 52 , 134 , ?

a) 302

b) 268

c) 300

d) 304

e) 208

53. 4500 , 900 , 90 , 6 , ? , 0.012

a) 0.3

b) 3

c) 3.33

d) 0.33

e) 3.3

54. 24 , 11 , 10 , 14 , 27 , ?

a) 66

b) 70.5

c) 68

d) 66.5

Number Series Workbook V-1

www.BankExamsToday.Com Page 7

e) 68.5

55. 8 , 7 , 12 , 33 , 128 , ?

a) 528

b) 365

c) 653

d) 825

e) 635

Directions (56-60): What should come in place of the

question mark (?) in the following number series?

56. 7 , 5 , 7 ,17 , 63 ?

a) 308

b) 302

c) 309

d) 409

e) 390

57. 50 , ? , 61 , 89, 154 , 280

a) 52

b) 51

c) 60

d) 62

e) 60

58. 17 , 19 , 25 , 37 , ? , 87

a) 47

b) 37

c) 57

d) 67

e) 75

59. 11 , 14 ,19 , 28, 43, ?

a) 55

b) 44

c) 77

d) 88

e) 66

60. 26 , 144 , 590 , 1164 , ?

a) 1296

b) 1182

c) 2059

d) 1182

e) 1181

Direction ( 61-65) Find the wrong number in the

following series.

61. 40, ,326, 2946 ,29418, 323607

1.326

2.40

3.2946

4.323607

5.29418

62. 560 , 1089 , 1725 , 2443, 3284 , 4245

1.2443

2.1725

3.4245

4.3284

5.560

63. 3252 , 3080 , 2958 , 2876 , 2826

1.3080

2.2876

3.2826

Number Series Workbook V-1

www.BankExamsToday.Com Page 8

4.3252

5.2958

64. 2442 , 1222 , 614 , 312 ,163 , 90 , 55.75

1.1222

2.312

3.90

4.614

5.163

65. 1250, 1322 , 1452, 1674, 2024 , 2544

1.1322

2.1674

3.2544

4.2024

5.1250

Direction (66-70) Find the missing number in the

given series.

66. 1953.125 781.25 312.5 ? 20

1.48

2.49

3.50

4.51

5.52

67. 81 87 162 504 1992 9990 ?

1.59903

2.59906

3.56895

4.59904

5.59861

68. 49 72 118 ? 394 762 1498

1.234

2.239

3.210

4.219

5.243

69. 142 143 156 193 272 417 ?

1.653

2.658

3.659

4.657

5.656

70. 118 122 158 206 356 536 ?

1.956

2.953

3.928

4.965

5.963

71. 2, 8, 14, 24, 34, 48, ?

a. 66

b. 62

c. 58

d. 64

72. 1, 2, 3, 2, 4, 5, 4, 5, ?

Number Series Workbook V-1

www.BankExamsToday.Com Page 9

a. 9

b. 6

c. 10

d. 7

73. 3, 8, 13, 24, 41, ?

a. 65

b. 75

c. 70

d. 80

74. 2, 4, 4, 8, 16, 16, 256,?

a. 64

b. 36

c. 180

d. 32

75. 1, 2, 3, 5, 8, ?

a. 11

b. 13

c. 15

d. 9

76. 138, 115, 92, 69, 46, ?

a. 9

b. 18

c. 21

d. 23

77. 4, 2, 8, 16, 128, ?

a. 562

b. 1018

c. 2024

d. 2048

78. 186, 115, 71, 44, 27, ?

a. 19

b. 17

c. 13

d. 21

79. 128, 64, 32, 16, ?, 4

a. 10

b. 8

c. 6

d. 12

Number Series Workbook V-1

www.BankExamsToday.Com Page 10

80. 1, 1, 2, 8, 64, ?, 65536

a. 1024

b. 2556

c. 4096

d. 1088

81. 1, 2, 4, 8, 16, 32, ?

a. 48

b. 56

c. 64

d. 70

82. 5, 50, 45, 450, 445, ?, 4445

a. 4450

b. 4600

c. 4550

d. 4500

83. 243, 5, 81, 15, 27, 45, 9, ?

a. 5

b. 15

c. 135

d. 27

84. 1, 2, 2, 5, 3, 10, ?

a. 8

b. 4

c. 7

d. 11

85. 4, -8, 14, -22, 32, ?

a. -44

b. 42

c. -42

d. 44

Answer:

1. 4;

The series is 3² + 13 = 22,4² + 26 = 42,

5² + 39= 64,6² +52 = 88

? = 7² + 65 = 114

2. 1;

The series is

Number Series Workbook V-1

www.BankExamsToday.Com Page 11

3. 2;

The series is

4. 1;

The series is

5. 3;

The series is

6. 5;

The series is

7. 3;

The series is

Number Series Workbook V-1

www.BankExamsToday.Com Page 12

8. 1;

The series is

9. 4;

The series is

10. 3;

The series is

11. e) 456

Solution:

8 x 0.5 + 0.5 = 4.5

4.5 x 1 + 1 = 5.5

5.5 x 2 + 2 = 13

Number Series Workbook V-1

www.BankExamsToday.Com Page 13

13 x 4 + 4 = 56

56 x 8 + 8 = 456

12. c) 215

Solution:

13. d) 131

Solution:

11 + 31 = 11 + 3 = 14

14 + 32 = 14 + 9 = 23

23 + 33 = 23 + 27 = 50

50 + 34 = 50 + 81 = 131

14. a) 169

Solution:

15. b) 53

Solution:

21 + 14 = 35

Number Series Workbook V-1

www.BankExamsToday.Com Page 14

35 – 5 = 30

30 + 14 = 44

44 – 5 = 39

39 + 14 = 53

16. a) 142

Solution:

17. b) 57

Solution:

18. a) 237

Solution:

11 + (13 + 1) = 11 + 3 = 13

13 + (23 - 1) = 13 + 7 = 20

20 + (33 + 1) = 20 + 28 = 48

48 + (43 - 1) = 48 + 63 = 111

111 + (53 + 1) = 111 + 126 = 237

Number Series Workbook V-1

www.BankExamsToday.Com Page 15

19. e) 353

Solution:

20. c) 392

Solution:

6 x 0.5 + 0.5 = 3.5

3.5 x 1 + 1 = 4.5

4.5 x 2 + 2 = 11

11 x 4 + 4 = 48

48 x 8 + 8 = 392

21. b) 5506

Solution:

6 x 2 + 4 = 16

16 x 3 - 3 = 45

45 x 4 + 4 = 184

184 x 5 - 5 = 917

917 x 6 - 6 = 5506

22. e) 146

Solution:

11 + 9 = 20

20 + 18 = 38

38 + 36 = 74

74 + 72 = 146

23. d) 145

Solution:

15 + 6 = 21

21 + 17 = 38 (17 = 6+11)

38 + 27 = 65 (27 = 17+10)

65 + 36 = 101 (36 = 27 + 9)

101 + 44 = 145 (44 = 36 + 8)

24. c) 46

Solution:

24 + 22 = 24 + 4 = 28

28 - 32 = 28 - 9 = 19

19 + 42 = 19 + 16 = 35

35 - 52 = 35 - 25 = 10

10 + 62 = 10 + 36 = 46

25. a) 32

Solution:

14 x 1 - 8 = 6

Number Series Workbook V-1

www.BankExamsToday.Com Page 16

6 x 2 - 8 = 4

4 x 3 - 8 = 4

4 x 4 - 8 = 8

8 x 5 - 8 = 32

26. c) 179

Solution:

14, 25, 47, 91, ?, 355

14 x 2 - 3 = 25

25 x 2 - 3 = 47

47 x 2 - 3 = 91

91 x 2 - 3 = 179

27. a) 136

Solution:

11, 24, 44, 70, 101, ?

28. d) 176

Solution:

18, 8, 6, 8, 24,?

18 x 0.5- 1 = 8

8 x 1- 2 = 6

6 x 2 - 4 = 8

8 x 4 - 8 = 24

24 x 8 - 16 = 176

29. b) 50

Solution:

28, 32, 23, 39, 14, ?

30. e) 4056

Solution:

5, 12, 33, 136, 675, ?

5 x 2 + 2 = 12

12 x 3 - 3 = 33

33 x 4 + 4 = 136

136 x 5 - 5 = 675

675 x 6 + 6 = 4056

31. a) 2, 4, 12, 48, 240, ….

The pattern is: to arrive at a term, the previous term is

being multiplied by (n+1) where ‘n’ keeps on increasing

by 1 for every term.

32.

4 = 2 × (2 + 0)

12 = 4 × (2 + 1)

48 = 12 × (2 + 2)

240 = 48 × (2 + 3)

⇒ Next term = 240 × (2 + 4) = 240 × 6 = 1440

Number Series Workbook V-1

www.BankExamsToday.Com Page 17

2. c) 2, 5, 9, 19, 37, …..

The pattern is: every number is arrived at previous

number multiplied by 2 and then alternate addition and

subtraction by 1 i.e.

2

5=2×2+1

9=5×2-1

19=9×2+1

37=19×2-1

the next term 37×2+1 = 75

33. b) 4, -8, 16, -32, 64, ….

The pattern is: Every number is arrived at by multiplying

previous alternate number with ‘4’ as shown below:

4 x 4 = 16

-8 x 4 = -32

16 x 4 = 64

-32 x 4 = -128

Hence, ‘-128’ is the correct answer.

34. e) 2, 9, 28, 65, 126, 216, 344.

The pattern in the series is that the series is triangular as

shown below:

In the triangular series, the difference between

consecutive terms is written below the numbers and

then, difference between consecutive differences is

written below & this process carries on until all the

difference become equal. In the figure above there was

an error & we have corrected it.

35. d) 10, 26, 74, 218, 654, 1946, 5834

The pattern is: to arrive at next term, the previous is

multiplied by 3 and subtracted by 4:

10

10 × 3 – 4 = 26

26 × 3 – 4 = 74

74 × 3 – 4 = 218

218 × 3 – 4 = 650 ≠ 654

650 × 3 – 4 = 1946

1946 × 3 – 4 = 5834

Here, ‘654’ was wrong.

36. d) The series is - 1.1, - 2.2, - 4.4, - 8.8, - 17.6

37. e) The series is: + 112, + 12

2, + 13

2, + 14

2, + 15

2

38. c) The series is: x5, x5, x5 ……

39. d) The series is: x1, x5, x9, x13, x17, x21

40. b) The series is: 14, 2

4, 3

2, 4

2, 5

4, 6

4, 7

4 (= 2401)

41. d.) 2401

Solution:

1 x 7 = 7

7x 7 = 49

49 x 7 = 343

343 x 7 = 2401

Number Series Workbook V-1

www.BankExamsToday.Com Page 18

42. d) 248

Solution:

13+122+3, 20+ 4

2+3, 39+6

2+3, 78+8

2+3, 145+ 10

2+3

43. a) 725

Solution:

difference x 2

44. e) none

Solution:

+97,+197+297,+397,+497

991+497 = 1488

45. c) 287

Solution:

+ 7x1, +7x3, +7x5, +7x7, +7x9

46. a) 25.2

18.3 + 2.3 = 20.6

20.6 - 4.6 = 16

16 + 6.9 = 22.9

22.9 - 9.2 = 13.7

13.7 +11.5 = 25.2

25.2 - 13.8 = 11.4

47. c) 61

9 x 0.5 + 0.5 = 5

5 x 1 + 1 = 6

6 x 1.5 + 1.5 = 10.5

10.5 x 2 + 2 = 23

23 x 2.5 + 2.5 = 60

60 x 3 + 3 = 183

48. e) 188

186 - 36 = 154

154 - 16 = 140

140 - 8 = 132

132 - 4 = 128

128 - 2 = 126

126 - 1 = 125

49. d) 151

2 x 1 + 2 = 4

4 x 2 + 3 = 11

11 x 3 + 4 = 37

37 x 4 + 5 = 153

153 x 5 + 6 = 771

771 x 6 + 7 = 4633

50. b) 394

391 + 2 = 393

393 + 6 = 399

399 + 12 = 441

441+ 20 = 431

431 + 30 = 461

461 +40 = 503

51. b) 8

Solution:

52. d) 304

Solution:

Number Series Workbook V-1

www.BankExamsToday.Com Page 19

10 x 3 - 2= 28

14 x 4 - 4= 52

28 x 5 - 6= 134

52 x 6- 8= 304

53. a) 0.3

Solution:

4500/5 = 900

900/10 = 90

90/15 = 6

6/20 = 0.3

0.3/25 = 0.012

54. d) 66.5

Solution:

24 x (1/2) – 1 = 11

11 x (2/2) – 1 = 10

10 x (3/2) – 1 = 14

14 x (4/2) – 1 = 27

27 x (5/2) – 1 = 66.5

55. e) 635

Solution:

8 x 1 – 1 = 7

7 x 2 – 2 = 12

12 x 3 – 3 = 33

33 x 4 – 4 = 128

128 x 5 – 5 = 635

56. c) 309

Solution:

7 x 1 – 2 = 5

5 x 2 – 3 = 7

7 x 3 – 4 = 17

17 x 4 – 5 = 63

63 x 5 – 6 = 309

57. a) 52

Solution:

50..50+(1³+1) = 52

52+(2³+1)=61.

61+(3³+1)=89

89+(4³+1)=154

154 + (5³+1) = 280

58. c) 57

Solution:

17 + 1 x 2 = 19

19 + 2 x 3 = 25

25 + 3 x 4 = 37

37 + 4 x 5 = 57

57 + 5 x 6 = 87

59. e) 66

Solution:

3….5...9...15...23

…2....4....6....8…

Answer 43+23= 66

60. b) 1182

Solution:

26 x 6 – 12 = 144

144 x 4 + 14 = 590

590 x 2 – 16 = 1164

1164 x 1 + 18 = 1182

Number Series Workbook V-1

www.BankExamsToday.Com Page 20

61.2946

40 x 8+6=326

326 x 9+7=294

2946 x 10+8=29418

29418 x 11+9=323607

62.1725

the difference between numbers is +23² ,+25² , +27² ,

+29² .......

63.3252

the difference between numbers is -(13²+1), -(11²+1), -

(9²+1) , -(7²+1), -(5²+1)

64. 90

2442/2 + 1 = 1222

1222/2 + 3= 614

614/2 + 5 = 312

312/2 + 7 =163

163/2 + 9 = 89.5

89.5/2 + 11 = 55.75

65. 1250

the difference between numbers is +(3³+3) , +(4³+4)

, +(5³+5), +(6³+6) , +(7³+7),

+(8³+8)

66.50

1953.125/2.5=781.25

781.25/2.5=312.5

312.5/2.5=125

125/2.5=50

50/2.5=20

67.59904

the difference between numbers is x 1+6 , x 2-12, x

3+18 ,x 4-24 ,x 5+30 , x 6-36

68.210

the difference between numbers is +23 , +46 , +92 ,

+184 , +368 ,+736

69.658

the difference between numbers is +1³+0 , +2³+5 ,+3³+10 , +4³+15 , +5³+20 ,+6³+25

70. 928

the difference between numbers is

+2³-2²

+3³-3²

+4³-4²

+5³-5²

+6³-6²

+7³-7²

71. (b): The sequence in the series is:

Number Series Workbook V-1

www.BankExamsToday.Com Page 21

The difference increases by 4 at alternate step.

72. (a): In this series three numbers from a set. The first two numbers of each set are in natural order and the third number

is the sum of first and second numbers. The first number of the next set begins with double the first number of the

previous set.

73. (c): The sequence in the series is (number + next number ) + addition of natural number increasing by 1 at each step,

after beginning from 2.

Number Series Workbook V-1

www.BankExamsToday.Com Page 22

74. (d): There are two alternate series:

Series I : 2,4,16,256 (next number is the square of precious numbers)

Series II : 4,8,16,32 (number is multiplied by 2 to get the next number)

75. (b): The numbers in the series are sum of two numbers preceding them.

76. (d): The numbers in the series are decreasing by 23.

77. (d): The number in the series is product of two numbers preceding it.

Number Series Workbook V-1

www.BankExamsToday.Com Page 23

78. (b): The sequence in the series is that the number subtracted from its previous number gives the next number.

79. (b): The numbers in the series are divided by 2 at each step.

80. (a): The sequence in the series is ×1, ×2, ×4, ×8, ×16, ×64.

Number Series Workbook V-1

www.BankExamsToday.Com Page 24

81. (c): The numbers in the series are double the previous number.

82. (a): The sequence in the series is ×10,-5, which is repeated.

83. (c): There are two alternate series:

84. (b): There are two alternate series:

Series I: 1,2,3,4 (numbers are increasing in natural order)

Series II: 2,5,10 (third number is product of previous two numbers)

85. (a) Alternative numbers are marked positive and negative, but the difference between their magnitude increases by 2

at each step.

Chennai: #1, South Usman Road, T Nagar. | Madurai: #24/21, Near Mapillai Vinayagar Theatre, Kalavasal. | Trichy: opp BSNL office, Juman Center, 43 Promenade Road, Cantonment. | Salem: #209, Sonia Plaza / Muthu Complex, Junction Main Rd, State Bank Colony, Salem. | Coimbatore #545, 1st floor, Adjacent to SBI (DB Road Branch), Diwan Bahadur

Road, RS Puram, Coimbatore (Kovai) – 641002 | Chandigarh: SCO 131-132 Sector 17C. | Bangalore.

H.O: 7601808080 / 9043303030 | www.raceinstitute.in

Is Now In CHENNAI | MADURAI | TRICHY | SALEM | COIMBATORE | CHANDIGARH |

BANGALORE|NAMAKKAL|ERODE|PUDUCHERRY

www.raceinstitute.in | www.bankersdaily.in

Number Series – 100 Q

R.A.C.E Rapid Academy of Competitive Exams

Chennai: #1, South Usman Road, T Nagar. Chennai. Mob: 9043303030 / 7601808080 Madurai: #24/21, Near Mapillai Vinayagar Theatre, Kalavasal. M: 7695814311 | www.RACEInstitute.in

2

MISSING NUMBER

1. 4500, 5400, 5488, 4608, ___?

2. 3, 6, 17, 48, ___, 248

3. 13, 24, 42, 79, 153, ___?

4. 2, 4, 20, 74, ___, 452

5. 122, 131, 203, 446, ___?

6. 6, 1338, 3074, 5298, ___?

7. 2199, 868, 139, 14, ___?

8. 3, 36, 153, 408, ___?

9. 10, 36, 92, 190, 342, ___?

10. 2, 65, 730, 4097, ___?

11. 1730, 1242, 856, 560, ___?

12. 10648, 9261, 15625, 9261, ___?

13. 999, 973, 875, 657, ___?

14. 157, 284, 375, ___, 473

15. 1, 9, 35, 91, ___, 341

16. 8, 125, 1331, 4913, ___?

17. 2, 28, 730, 2198, ___?

18. 1000, 1458, 1331, 1536, ___, 1372?

19. 54, 8, 500, 192, 2058, ___, 1080?

20. 625, 864, 1029, 1024, ___?

21. 2000, 24, 2048, 320, 1296, ___?

22. 1331, 56, 135, 192, ___?

23. 88, 189, 625, 1029, ___?

24. 3, 18, 57, 132, 255, ___?

25. 125, 128, 162, ___, 120

26. 124, 65, 53, 49, ___?

27. 121, 194, 165, 132, ___?

28. 725, 970, 663, 404, ___?

29. 720, 960, 3000, 2058, 1458, ___?

30. 0, 128, 162, 96, ___?

31. 3, 16, 81, 128, 375, ___?

32. 3, 4, 81, 32, 375, ___?

33. 3, 33, 73, 172, ___?

34. 0.5, 16, 3, 96, 187.5, ___?

35. 500, 768, 540, 224, ___?

36. 121.5, 437.5, ___, 1093.5, 665.5

37. 170, 34, 13.6, 8.16, ___?

38. 40, 80, 53.3, 21.32, ___?

39. 750, 377, 191.5, 100.75, ___?

40.

?

41.

?

42. 1, 1, 2, 6, ___?

43. 1, 3, 8, 27, ___?

44. 0, 2, 12, 72, ___?

45. -1, -1, 2, 19, ___?

R.A.C.E Rapid Academy of Competitive Exams

Trichy: opp BSNL office, Juman Center, 43 Promenade Road, Cantonment. Mob: 9360703030 Salem: #209, Muthu Plaza, Junction Main Rd, State Bank Colony, Salem. Mob: 7305958080

Coimbatore: #545, First Floor, Diwan Bahadur Road, RS Puram. Mob: 7667673030 / 7667678080

3

46. 4, 18, 96, 600, ___?

47. 5, 14, 54, 264, ___?

48. 2, 6, 15, 40, ___?

49. 0, -2, -3, 8, ___?

50. 2, 6, 21, 100, ___?

51. 8, 13, 33, 123, ___?

52. 2, 6, 42, 600, ___?

53. 2, 0, 4, 0, 9, 3, 28, 20, ___?, ___?

54. 2, 8, 72, 1152, ___?

55. 4, 3, 9, 25, ___?

56. 4, 7, 24, 29, ___?

57. 2, 1, 12, 12, ___?

58. 1, 1, 36, 1, ___?

59. 1, 1, 18, 6, ___?

60. 1, 8, 54, 384, ___?

61. 1, 8, 162, 384, ___?

62. 6, 15, 31, 65, ___?

63. -1, 1, 6, 25, ___?

64. 4991, 95, -3, ___?

65. 1, 8, 216, ___?

66. 2 3 18 115 854, ___?

67. 17, 26, 113, 1026, ___? 410634

68. 9, 54.9, 356.85, ___?

69. 2431, 221, 17, ___?

70. 6, 15, 35, 77, ___ ?

71. 97, 267, 415, 553, ___ ?

72. 106, 236, 366, 536, ___ ?

73. 5, 15, 35, 63, ___ ?

74. 3, 5, 19, 41, 89, 131, ___?

75. 7, 10, 16, 22, 28, ___?

76. 3, 7, 14, 23, ___, 49.

77. 2, 6, 11, 18, 25, ___ ?

78. 5, 8, 12, 16, 22, ___?

79 . 3, 11, 32, 71, ___?, 229

80. 10, 69, 227, 529, ___?

81. 11, 56, 135, 192, ___?

82. 3, 23, 136, 447, ___?

83. 17, 194, ___?, 3588, 8024

84. 122, 27, 11, 9, 12, ___?

85. 2, 12, 36, 80, ___?

86. 2, 13, 34, 65, ___?

87. 4, 144, 900, ___?

88. 201, 243, ___?, 339

89. 3, 14, 35, 66, ___?

90. 6, 21, ___?, 81, 126

91. 2, 16, ___?, 128

R.A.C.E Rapid Academy of Competitive Exams

Chennai: #1, South Usman Road, T Nagar. Chennai. Mob: 9043303030 / 7601808080 Madurai: #24/21, Near Mapillai Vinayagar Theatre, Kalavasal. M: 7695814311 | www.RACEInstitute.in

4

92. 221, 265, ___?, 365

93. 20, 4, 52, 144, ___?, ___?

94. 99, 101, 96, ___?, ___?, 109

95. 450, 441, 500, ___?, ___?, 529

96. 231, 276, ___?, 378

97. 1640, 1806, 2162, ___?

98.. 2, 0, 5, 3, ___?, 8

99. 240, 288, ___?, 396, 456

100. 101, 98, ___?, 86

1. 4500, 5400, 5488, 4608, ____?

Sol:

4500 5400 5488 4608 2916

53x62 63x52 73x42 83x32 93x22

Q.2) 3, 6, 17, 48, ____, 248

Sol:

3 6 17 48 117 248

13+2 23+3 33+4 43+5 53+6

Q.3) 13. 13, 24, 42, 79, 153, ____?

Sol:

13 24 42 79 153 288

13+10 23+10 33+10 43+10 53+10

Q.4) 2, 4, 20, 74, ____, 452

Sol:

2 4 20 74 202 452

13+13 23+23 33+33 43+43 53+53

5. 122, 131, 203, 446, ____?

Sol:

122 131 203 443 1022

13+23 23+43 33+63 43+83

6. 6, 1338, 3074, 5298, ____?

Sol:

6 1338 3074 5298 8106

13+113 23+123 33+133 43+143

7. 2199, 868, 139, 14, ___?

R.A.C.E Rapid Academy of Competitive Exams

Trichy: opp BSNL office, Juman Center, 43 Promenade Road, Cantonment. Mob: 9360703030 Salem: #209, Muthu Plaza, Junction Main Rd, State Bank Colony, Salem. Mob: 7305958080

Coimbatore: #545, First Floor, Diwan Bahadur Road, RS Puram. Mob: 7667673030 / 7667678080

5

Sol:

2199 868 139 14 15

113 93 53 -13

-2 -4 -6

8. 3, 36, 153, 408, ___?

Sol:

3 = (13 +13) + 1 408 = (43 +73)

+ 1

34 = (23 +33) + 1 855 = (53 +93) + 1

153 = (33 +53) + 1

9. 10, 36, 92, 190, 342, ___?

Sol:

10 = (13 +23) + 1 190 = (43 +53)

+ 1

36 = (23 +33) + 1 342 = (53 +63) + 1

92 = (33 +43) + 1 560 = (63 +73) + 1

10. . 2, 65, 730, 4097, ____?

Sol:

2 = (13 x 13) + 1 4097 = (43 x

43) + 1

65 = (23 x 23) + 1 15626 = (53 x 53)

+1

730 = (33 x 33) + 1

11. . 1730, 1242, 856, 560, ____?

Sol:

1730 = (103 + 93) + 1 560 = (73 + 63)

+ 1

1242 = (93 + 83) + 1 342 = (63 + 53)

+1

856 = (83 +73) + 1

12. 10648, 9261, 15625, 9261, ____?

Sol:

10648 9261 15625 9261

10648

23 x 113 33 x 73 53 x 53 73 x 33 113

x 23

13. 999, 973, 875, 657, ____?

999 973 875 657 271

1000-13 1000-33 1000-53 1000-73

1000-93

14. 157, 284, 375, ____, 473

Sol:

157 284 375 436 473

500-73 500-63 500-53 500-43 500-

33

15. 1, 9, 35, 91, ___, 341

Sol:

R.A.C.E Rapid Academy of Competitive Exams

Chennai: #1, South Usman Road, T Nagar. Chennai. Mob: 9043303030 / 7601808080 Madurai: #24/21, Near Mapillai Vinayagar Theatre, Kalavasal. M: 7695814311 | www.RACEInstitute.in

6

1 9 35 91 189

341

03+13 13+23 23+33 33+43 43+53

53+63

16. 8, 125, 1331, 4913, ____?

Sol:

Alternate prime number cube

8 125 1331 4913

12167

23 53 113 173 233

17. 2, 28, 344, 2198, ____?

Sol:

2 28 344 2198 9262

13+1 33+1 73+1 133+1 213+1

+2 +4 +6 +8

18. 1000, 1458, 1331, 1536, ____?

Sol:

1000 1458 1331 1536 1728

1372

103 93x2 113 83x3 123

73x4

+1 +1

-1 -1

19. 54, 8, 500, 192, 2058, ____?

Sol:

54 8 500 192 2058

1080

33x2 23x1 53x4 43x3 73x6

63x5

+2 +2

+2 +2

20. 625, 864, 1029, 1024, ___?

Sol:

625 864 1029 1024 729

53x5 63x4 73x3 83x2 93x1

21. 2000, 24, 2048, 320, 1296, ____?

Sol:

2000 24 2048 320 1296

1512

103x2 23x3 83x4 43x5 63x6 63x7

-2 -2

+2 +2

22. 1331, 56, 135, 192, ___?

Sol:

R.A.C.E Rapid Academy of Competitive Exams

Trichy: opp BSNL office, Juman Center, 43 Promenade Road, Cantonment. Mob: 9360703030 Salem: #209, Muthu Plaza, Junction Main Rd, State Bank Colony, Salem. Mob: 7305958080

Coimbatore: #545, First Floor, Diwan Bahadur Road, RS Puram. Mob: 7667673030 / 7667678080

7

1331 56 135 192 250

13x11 23x7 33x5 43x3 53x2

Note: 11, 7, 5, 3, 2 are prime numbers

23. 88, 189, 625, 1029, ____?

Sol:

88 189 625 1029

2662

23x11 33x7 53x5 73x3 113x2

Multiplication of cube of first 5 prime

numbers with prime numbers in reverse

order

24. 3, 18, 57, 132, 255, ___?

Sol:

3 = (13 x2) + 1 132 = (43 x2) + 4

18= (23 x2) + 2 255 = (53 x2) + 5

57= (33 x2) + 3 438 = (63 x2) + 6

25. 125, 128, 162, ___, 120

Sol:

125 128 162 192 120

53x1! 43x2! 33x3! 23x4!

13x5!

26. 124, 65, 53, 49, ___?

Sol:

124 = 53x0!-1 49 =

23x3!+1

65 = 43x1!+1 23 = 13x4!-1

53 = 33x2!-1

27. 121, 194, 165, 132, ___?

Sol:

121 = 13x5!+1 132 = 43x2!+4

194 = 23x4!+2 130 = 53x1!+5

165 = 33x3!+3

28. 725, 970, 663, 404, ___?

Sol:

725 = 13x6!+5 404 = 43x3!+20

970 = 23x5!+10 275 = 53x2!+25

648 = 33x4!+15

29. 720, 960, 3000, 1296, 686, ___?

Sol:

720 = 13x6! 1296 = 63x3!

960 = 33x5! 686 = 73x2!

3000 = 53x4! 512 = 83x1!

30. 0, 128, 162, 96, ___?

Sol:

0 = 53x1x0 96 = 23x4x3

128 = 43x2x1 25 = 13x5x5

162 = 33x3x2

31. 3, 16, 81, 128, 375, ___?

Sol:

3 16 81 128 375 432

13x3 23x2 33x3 43x2 53x3 63x2

R.A.C.E Rapid Academy of Competitive Exams

Chennai: #1, South Usman Road, T Nagar. Chennai. Mob: 9043303030 / 7601808080 Madurai: #24/21, Near Mapillai Vinayagar Theatre, Kalavasal. M: 7695814311 | www.RACEInstitute.in

8

32. 3, 4, 81, 32, 375, ___?

Sol:

3 4 81 32 375 108

13x3 23 2 33x3 43 2 53x3 63 2

33. 3, 33, 73, 172, ___?

Sol:

3 = (23+1) 3 172 = (83+4) 3

33 = (43+2) 3 335 = (103+5) 3

73 = (63+3) 3

34. 0.5, 16, 3, 96, 187.5, ___?

Sol:

0.5 16 3 96 187.5 216

13x0.5 23x2 33x1 43x1.5 53x1.5

63x1

+0.5 +0.5

-0.5 -0.5

35. 500, 768, 540, 224, ____?

Sol:

500 768 540 224 36

103x0.5 83x1.5 63x2.5 43x3.5

23x4.5

36. 121.5, 437.5, ____, 1093.5, 665.5

Sol:

121.5 437.5 857.5 1093.5

665.5

33x4.5 53x3.5 73x2.5 93x1.5

113x0.5

37. 170 34 13.6 8.16 ?

Sol:

170 34 13.6 8.16 6.528

X0.2 x0.4 x0.6 x0.8

38. 40 80 53.3 21.32

Sol:

40 80 53.3 21.32 6.091

0.5 1.5 2.5 3.5

39. 750 377 191.5 100.75 ?

Sol:

750 377 191.5 100.75 57.375

0.5+2 0.5+3 0.5+5 0.5+7

40. 1/2 1/3 5/6 7/6 ?

R.A.C.E Rapid Academy of Competitive Exams

Trichy: opp BSNL office, Juman Center, 43 Promenade Road, Cantonment. Mob: 9360703030 Salem: #209, Muthu Plaza, Junction Main Rd, State Bank Colony, Salem. Mob: 7305958080

Coimbatore: #545, First Floor, Diwan Bahadur Road, RS Puram. Mob: 7667673030 / 7667678080

9

Sol:

1/2 1/3 5/6 7/6 2

+

+

41. 2 7/3 8/3 3 ?

Sol:

2 7/3 8/3 3 10/3

+0.33 +0.33 +0.33 +0.33

42. Ans: 24

Soln: Factorial of ‘n’ numbers with each term

n

(Eg: Upto 5! = 1, 2, 6, 24, 120)

, , , ,

43. Ans: 124

Soln: Factorial of ‘n’ numbers with each term

+ previous number

(Eg: 5! = 1, 2, 6, 24, 120)

+0,+1,+2, +3, +4

44. Ans: 480

Soln: Factorial of ‘n’ numbers with each term

x previous number

(Eg: Upto 5! = 1, 2, 6, 24, 120)

x0,x1,x2, x3, x4

45. Ans: 114

Soln: Factorial of ‘n’ numbers with each term

- next number

(Eg: Upto 5! = 1, 2, 6, 24, 120)

-2,-3, -4, -5, -6

46. Ans: 4320

Soln: 3!-2!, 4!-3!, 5!-4!, 6!-5!,……

47. Ans: 1560

Soln: Upto 5!

= 1, 2, 6, 24, 120

x(2+3), x(3+4),x(4+5),x(5+6),x(6+7)

48. Ans: 145

Soln: Upto 5!

= 1, 2, 6, 24, 120

+12, +22, +32 +42 +52

49 . Ans: 95

Soln: Upto 5!

= 1, 2, 6, 24, 120

-12, -22, -32 -42 -52

50. Ans: 725

Soln: Upto 5!

= 1, 2, 6, 24, 120

x1+1, x2+2, x3+3, x4+4, x5+5

R.A.C.E Rapid Academy of Competitive Exams

Chennai: #1, South Usman Road, T Nagar. Chennai. Mob: 9043303030 / 7601808080 Madurai: #24/21, Near Mapillai Vinayagar Theatre, Kalavasal. M: 7695814311 | www.RACEInstitute.in

10

51. Ans: 723

Soln: Upto 5!

= 1, 2, 6, 24, 120

X5+3, x5+3, x5+3, x5+3, x5+3

52. . Ans: 14520

Soln: Upto 5!

= 1, 2, 6, 24, 120

12+1, 22+2, 62+6, 242+24,1202+120

53. Ans: 125, 115……….

Soln: n!+n, n!-n

54. Ans: 28800

Soln: Upto 5!

= 1, 2, 6, 24, 120

12+12, 22+22, 62+62,242+242,1202+1202

55. Ans: 123

Soln: Upto 5!

= 1, 2, 6, 24, 120

+3, -1, +3, -1, +3

56. Ans: 480

Soln: Upto 5!

= 1, 2, 6, 24, 120

x4, +5, x4, +5, x4

57. Ans: 240

Soln: Upto 5!

= 1, 2, 6, 24, 120

x2, 2, x2, 2, x2

58. Ans: 14400

Soln: Upto 5!

= 1, 2, 6, 24, 120

x1, 2, x6, 24, x120

59. Ans: 720

Soln: Upto 5!

= 1, 2, 6, 24, 120

x1, 2, x3, , x5

60. Ans: 3000

Soln: Upto 5!

= 1, 2, 6, 24, 120

x12, x22, x32, x42, x52

61. Ans: 15000

Soln: Upto 5!

= 1, 2, 6, 24, 120

x13, x22, x33, x42, x53

62. Ans: 181

Soln: Upto 5!

= 1, 2, 6, 24, 120

+12+22, 22+32, 32+42, 42+52, 52+62

R.A.C.E Rapid Academy of Competitive Exams

Trichy: opp BSNL office, Juman Center, 43 Promenade Road, Cantonment. Mob: 9360703030 Salem: #209, Muthu Plaza, Junction Main Rd, State Bank Colony, Salem. Mob: 7305958080

Coimbatore: #545, First Floor, Diwan Bahadur Road, RS Puram. Mob: 7667673030 / 7667678080

11

63. Ans: 122

Soln: Upto 5!

= 1, 2, 6, 24, 120)

-2, -1, 0, +1, +2

64. Ans: 0

Soln: 7!-72, 5!-52, 3!-32, 1!-12

65. Ans: 13824

Soln: 1! 3, 2! 3, 3!3, 4! 3

66. . 2 3 18 115 854 ?

Sol :

2 3 18 115 854 7767

x 1+ x3+ x5+ x7+ x9+

Ans :7767

67. 17 26 113 1026 ? 410634

Sol : 17 26 113 1026 ? 410634

x 1+9 x4+9 x9+9 x16+9 x25+9

Ans : 16425

68. . 9 54.9 356.85 ?

Sol :

9 54.9 356.85 ?

x6.1 x6.5 x6.9

Ans : 2462.265

69. 2431, 221 , 17 , ?

Sol :

2431 221 17 ?

Ans: 1

70. 6, 15, 35, 77, ___ ?

Soln: 3x2, 5x3, 7x5, 11x7, 3x11 (Prime no.

x previous prime no.)

Ans: 143

71. 97, 267, 415, 553, ___ ?

Soln: .

Prime numbers in reverse from 97

97x1 89x3 83x5 79x7 73x9

Odd numbers

Ans: 657

72.

106, 236, 366, 536, ___ ?

Soln:

Prime numbers in from 53

53x2 59x4 61x6 67x8 71x10

R.A.C.E Rapid Academy of Competitive Exams

Chennai: #1, South Usman Road, T Nagar. Chennai. Mob: 9043303030 / 7601808080 Madurai: #24/21, Near Mapillai Vinayagar Theatre, Kalavasal. M: 7695814311 | www.RACEInstitute.in

12

Even numbers

73. 5, 15, 35, 63, ___ ?

Soln: 2x2+1, 3x4+3, 5x6+5, 7x87, 11x10+9

Ans: 119

74. . 3, 5, 19, 41, 89, 131, ___?

Soln: 2x2-1, 3x3-4, 5x5-6, 7x7-8, 11x9-10,

13x11-12

Ans: 131

75. 7, 10, 16, 22, 34, ___?

Soln: Alternate prime numbers (2, 5, 11,. .

.) +5

Ans: 42

76. 3, 7, 14, 23, ___, 49.

Soln: 2+12, 3+22, 5+32, 7+42, 11+52, 13+62

Ans: 36

77. 2, 6, 11, 18, 25, ___ ?

Soln: 22-2, 32-3, 42-5, 52-7, 62-11, 72-13

Ans: 36

78. 5, 8, 12, 16, 22, ?

Soln: 2+22-12, 3+32-22, 5+42-32, 7+52-42,

11+62-52, 13+72-62

Ans: 26

79. 3, 11, 32, 71, ?, 229

Soln: 2+13, 3+23, 5+33, 7+43, 11+53

Ans: 136

80. 10, 69, 227, 529, ?

Soln: 2+23, 5+43, 11+63, 17+83, 29+103

Ans: 1029

81. 11, 56, 135, 192, ?

Soln: 11x13, 7x23, 5x33, 3x43, 2x53

Ans: 250

82. . 3, 23, 136, 447, ?

Soln: 2x13+1, 3x23-1, 5x33+1, 7x43-1,

11x63+1

Ans: 2377

83. 17, 194, ?, 3588, 8024

Soln: 2x23+1, 3x43+2, 5x63+3, 7x83+4,

11x93+5

Ans: 1083

84. 122, 27, 11, 9, 11, ?

Soln: 2+5!, 3+4!, 5+3!, 7+2!, 11+1!, 13+0!

Ans: 14

85. 2, 12, 36, 80, ?

Ans: 150

Soln: 12(1+1), 22 (1+2), 32 (1+3),

42(1+4),……

R.A.C.E Rapid Academy of Competitive Exams

Trichy: opp BSNL office, Juman Center, 43 Promenade Road, Cantonment. Mob: 9360703030 Salem: #209, Muthu Plaza, Junction Main Rd, State Bank Colony, Salem. Mob: 7305958080

Coimbatore: #545, First Floor, Diwan Bahadur Road, RS Puram. Mob: 7667673030 / 7667678080

13

86. . 2, 13, 34, 65, ?, ……

Ans: 106

Soln: 12+12, 22+32, 32+52, 42+72, ……..

87. 4, 144, 900, ?

Ans: 1764

Soln: 12x22, 32x42, 52x62, 62x72,……

88. 201, 243, ?, 339

Ans: 289

Soln: (102x2)+1, (112x2)+1, (122x2)+1,

(132x2)+1,…….

89. . 3, 13, 35, 65, ?

Ans: 107

Soln: 12+12+1, 22+32+1, 32+52+1,

42+72+1, ……..

90. 6, 21, ?, 81, 126

Ans: 46

Soln: 12+22+1, 22+42+1, 32+62+1,

42+82+1, ……..

91. 2, 16, ?, 128

Ans: 54

Soln: (12+12)x1,( 22+22)x2, (32+32)x3,

(42+42)x4, ……..

92. 221, 265, ?, 365

Ans: 313

Soln: (102+112), (112+122), (122+132),

(132+142),…….

93. 20, 4, 52, 144, ?, ?

Ans: 22+42, 12x22, 42+62, 32x42, 62+82,

42x52

94. 99, 101, 96, ?, ?, 109

Ans: 104, 91

Soln: 100-12, 100+12, 100-22, 100+22, 100-

32, 100+32,….

95. 450, 441, 500, ?, ?, 529

Ans: 484, 550

Soln: 202+(50x1), 212, 202+(50x2), 222,

202+(50x3), 232

96. 231, 276, ?, 378

Ans: 325

Soln: (102+112)+10, (112+122) +11,

(122+132) +12, (132+142) +13,…….

97. 1640, 1806, 2162, ?

Ans: 2756

Soln: 412-41, 432-43, 472-47, 532-53,….

98. 2, 0, 5, 3, ?, 8

Ans: 10

R.A.C.E Rapid Academy of Competitive Exams

Chennai: #1, South Usman Road, T Nagar. Chennai. Mob: 9043303030 / 7601808080 Madurai: #24/21, Near Mapillai Vinayagar Theatre, Kalavasal. M: 7695814311 | www.RACEInstitute.in

14

Soln: 12+1, 12-1, 22+1, 22-1, 32+1, 32-1,….

99. 240, 288, ?, 396, 456

Ans: 340

Soln: 152+(15x1), 162+(16x2), 172+(17x3),

182+(18x4),….

100. 101, 98, ?, 86

Ans: 93

Soln: 102-(1x2)+3, 102-(2x3)+4, 102-

(3x4)+5, 102-(4x5)+6

Branches

Chennai, Madurai, Trichy,

Salem, Coimbatore,

Chandigarh & Bangalore

Official Website:

www.raceinstitute.in

Official Blog:

www.bankersdaily.in

Number Series Questions Asked in Previous Exams

www.BankExamsToday.com Page 1

1) 7, 10, 16, 28, ?, 100

Ans. 52

2) 9, 11, 16, 26, ?, 69

3) 3 , 4, 10, 33, 136, ?

3 4 10 33 136 685

3 x 1 + 1 = 4

4 x 2 + 2 = 10

10 x 3 + 3 = 33

136 x 5 + 5 685

4) 21, 24, 32, 45, 63, ?

5) 6, 3, 3, 6, 24, ?

6 ✕ 0.5 = 3

3 ✕ 1 = 3

3 ✕ 2 = 6

6 ✕ 4 = 24

24 ✕ 8 = 112

6) 7, 10, 16, 28, 52, 100

7) 9, 9, 12, 20, 35, ?

Ans. 59

8) 4, 13, 23, 35, 50, ?

Ans. 69

9) 9, 17, 81, 297, ?, 1809

Ans. 809

10) 7, 20, 13, 26, 19, ?

Ans. 32

11) 7, 20, 46, 85, ? , 202

Ans : 137

Number Series Questions Asked in Previous Exams

www.BankExamsToday.com Page 2

12) 4, 13, 40, 121, ? , 1093

Ans: 364

13) 17, 18, 14, 23, 7, ?

Ans: 32

14) 2180 , 2179, 2152, 2027, ?

Ans: 1684

15) 6, 10, 19, 32, 48, ?

Ans : 66

Directions (16-20): What should come in place

of the question mark (?) in the following

number series?

16) 14, 25, 47, 91, ?, 355

a) 100

b) 197

c) 179

d) 335

e) 155

Ans. c) 179

14, 25, 47, 91, ?, 355

14 * 2 - 3 = 25

25 * 2 - 3 = 47

47 * 2 - 3 = 91

91 * 2 - 3 = 179

17) 11, 24, 44, 70, 101, ?

a) 136

b) 102

c) 80

d) 102

e) 163

Ans. a) 136

11, 24, 44, 70, 101, ?

Number Series Questions Asked in Previous Exams

www.BankExamsToday.com Page 3

18) 18, 8, 6, 8, 24, ?

a) 6

b) 48

c) 24

d) 176

e) 167

Ans. d) 176

18, 8, 6, 8, 24,?

18 * 0.5- 1 = 8

8 * 1- 2 = 6

6* 2 - 4 = 8

8 * 4 - 8 = 24

24*8 - 16 = 176

19) 28, 32, 23, 39, 14, ?

a) 30

b) 50

c) 55

d) 6

e) 14

Ans. b) 50

28, 32, 23, 39, 14, ?

20) 5, 12, 33, 136, 675, ?

a) 5569

b) 4426

c) 5046

d) 4065

e) 4056

Ans. e) 4056

5, 12, 33, 136, 675, ?

5 * 2 + 2 = 12

12 * 3 - 3 = 33

33 * 4 + 4 = 136

136 * 5 - 5 = 675

675 * 6 + 6 = 4056

Ques 21) 5, 4, 7, 20, 79,?

Ans.

⇒ 5 ✕ 1-1=4

4 ✕ 2-1=7

7 ✕ 3-1=20

20 ✕ 4-1=79

79 ✕ 5-1=394

Ans 394

Ques 22) 18, 9, 9, 13.5, 27,?

Ans.

Number Series Questions Asked in Previous Exams

www.BankExamsToday.com Page 4

⇒ 18 ✕ 0.5 = 9

⇒ 9 ✕ 1 = 9

⇒ 9 ✕ 1.5 = 13.5

⇒ 15.5 ✕ 2= 27

⇒ 27 ✕ 2.5 = 67.5 answers

Ques 23) 3, 83, 152, 209, 253,?

Ans.

There is pattern between the differences of the

numbers given in the series.

83 - 3 = 80

152 - 83 = 69

209 - 152 = 57

253 -209 = 44

Now

⇒ 80 - 69 = -11

⇒ 69 - 57 = -12

⇒ 57 - 44 = -13

⇒ 44 - 30 = -14

30 is the answer.

Ques 24) 318, 160, 82, 44,?

Ans.

This questions was very easy, there was a clear

pattern in series, no need to find the differences.

⇒ x0.5+1

⇒ x0.5+2

⇒ x0.5+3

⇒ x0.5+4

Ques 25) 500, 500, 493, 467, 404,?

Ans.

Again we need to find the differences between

the numbers and then there was a pattern in the

differences.

⇒ 0....-7....-26....-63....-124

⇒ 7....19....37....61

⇒12....18....24

Ques 26) 22, 11, 11, 16.5, 33,?

Ans.

⇒ x0.5....x1....x1.5....x2....x2.5

⇒ 33 ✕ 2.5 = 82.5

Ques 27) 6, 5, 9, 26, 103,?

Ans.

⇒ x1-1….x2-1…x3-1….x4-1

Ques 28) 401, 401, 394, 368, 305, ?

Ans.

There is pattern between the differences of the

numbers given in the series.

⇒ 0....-7....-26....-63....-124

⇒ 7....19....37....61

⇒ 12....18....24

Ques 29) 4, 75, 136, 186, 224,?

Ans.

+71....+61....+50....+38....+25

-10....-11....-12....-13

Ques 30)158, 80, 42, 24,?

Number Series Questions Asked in Previous Exams

www.BankExamsToday.com Page 5

Ans.

x0.5+1….x0.5+2….x0.5+3….x0.5+4

24 ✕ 1/2 + 3= 15 Answer

Directions (31-35): What should come in place

of the question mark (?) in the following

number series?

31) 6, 4, 5, 11, 39, ?

a) 188

b) 198

c) 189

d) 199

e) 200

Ans. c) 189

6 x 1 – 2 = 4

4 x 2 – 3 = 5

5 x 3 – 4 = 11

11 x 4 – 5 = 39

39 x 5 – 6 = 189

32) 14, 7, 7, 14, 56, ?

a) 158

b) 184

c) 484

d) 884

e) 448

Ans. e) 448

14 x 0.5 = 7

7 x 1 = 7

7 x 2 = 14

14 x 4 = 56

56 x 8 = 448

33) 2, 3, 7, 16, 32, ?

a) 57

b) 59

c) 56

d) 60

e) 75

Ans. a) 57

2 + 12 = 3

3 + 22 = 7

7 + 32 = 16

16 + 42 = 32

32 + 52 = 57

34) 8, 14, 24, 38, 56, ?

a) 55

b) 65

c) 96

d) 78

e) 87

Ans. (d) 78

8……....14….....24….......38….....56

…..+6…...+10…....+14…......+18

…….+4……...+4…......…+4…….+4

Answer: 56 + 18 + 4 = 78

35) 5, 7, 13, 25, 45, ?

a) 57

b) 75

c) 55

d) 77

e) 54

Ans. b) 75

5….....…7……...13…......25….......45

…....+2….....+6…......+12..….+20

…......…+4…...…+6….......+8……+10

Number Series Questions Asked in Previous Exams

www.BankExamsToday.com Page 6

Answer: 45 + 20 + 10 = 75

Directions (36-40): What should come in place

of the question mark (?) in the following

number series?

36) 4, 16, 26, 34, 40, ?

a) 45

b) 46

c) 43

d) 55

e) 44

Ans. e) 44

4………16……….26………..34…...….40………

…….+12…..+10…..…..+8…..…...+6….…...+4

Answer: 40 + 4 = 44

37) 825, 582, 501, 474, 465,?

a) 484

b) 642

c) 246

d) 462

e) 532

Ans. d) 462

825…….…..582….…..501….…..474……..465………

……..-243…...…-81………-27…….…-9…….…-3…

Answer: 465 – 3 = 462

38) 8, 12, 24, 60, 180, ?

a) 360

b) 300

c) 630

d) 600

e) 720

Ans. c) 630

8 x 1.5 = 12

12 x 2 = 24

24 x 2.5 = 60

60 x 3 = 180

180 x 3.5 = 630

39) 3, 4, 10, 33, 136, ?

a) 586

b) 685

c) 569

d) 690

e) 265

Ans. b) 685

3 x 1 + 1 = 4

4 x 2 + 2 = 10

10 x 3 + 3 = 33

33 x 4 + 4 = 136

136 x 5 + 5 = 685

40) 16, 17, 13, 22, 6, ?

a) 31

b) 2

c) 1

d) 12

e) 13

Ans. a) 31

16………..17……..13……....22……..6……

…...+1…........-4……....+9…….-16……(+25)

….(1 x 1)….(2 x 2)….(3x 3)….(4x4)....(5 x 5)

Answer: 6+25= 31

41) 1, 4, 18, 44, 83......?

Ans. 136

Number Series Questions Asked in Previous Exams

www.BankExamsToday.com Page 7

42) 12, 13, 17, 26, 42.....?

Ans. 67

43) 7, 14, 21, 28......?

Ans. 35

44) 157, 150, 136, 115, .....?

Ans. 87

45) 8, 4, 4, 6, 12, .....?

Ans. 30

46) 1, 2, 8, 48, 384, ......?

Ans. 3840 (Numbers are multiplied by 2,4,6,8,10)

Directions (47-51): What should come in place

of the question mark (?) in the following

number series?

47) 7, 6, 8, 15, 44, ?

a) 88

b) 120

c) 159

d) 150

e) 195

Ans. e) 195

7 x 1 – 12 = 6

6 x 2 – 22 = 8

8 x 3 – 32 = 15

15 x 4 – 42 = 44

44 x 5 – 52 = 195

48) 16, 25, 42, 66, 96, ?

a) 120

b) 160

c) 118

d) 131

e) 130

Ans. d) 131

+9….…..+17…..…+24……...+30….…..+35

+8…….…+7….…...+6…...…...+5

96 + 35 = 131

49) 4, 6, 9, 34, 68, ?

a) 56

b) 64

c) 161

d) 68

e) 116

Ans. c) 161

4 x 2 – 2 = 6

6 x 3 – 9 = 9

9 x 4 - 2 = 34

34 x 5 – 9 = 161

50) 7.6, 9.9, 14.5, 23.7, ?

a) 42.2

b) 42.1

c) 42.3

d) 42.4

e) 42.5

Ans. 42.1

+2.3 ….. +4.6 ….. +9.2 .…. +18.4

7.6 + 2.3 = 9.9

9.9 + 4.6 = 14.5

14.5 + 9.2 = 23.7

23.7 + 18.4 = 42.1

51) 9, 5, 4, 7.5, 13, ?

a) 35

b) 26

Number Series Questions Asked in Previous Exams

www.BankExamsToday.com Page 8

c) 34

d) 48

e) 62

Ans. a) 35

9 x 0.5 + 0.5 = 5

5 x 1 – 1 = 4

4 x 1.5 + 1.5 = 7.5

7.5 x 2 – 2 = 13

13 x 2.5 + 2.5 = 35

Directions (52-56): What should come in place

of the question mark (?) in the following

number series?

52) 7, 14, 30, 56, 93, ?

a) 142

b) 403

c) 124

d) 96

e) 124

Ans. a) 142

53) 23, 39, 32, 48, 41, ?

a) 58

b) 57

c) 59

d) 48

e) 84

Ans. b) 57

54) 11, 13, 20, 48, 111, ?

a) 237

b) 125

c) 273

d) 255

e) 555

Ans. a) 237

11 + (13 + 1) = 11 + 3 = 13

13 + (23 - 1) = 13 + 7 = 20

20 + (33 + 1) = 20 + 28 = 48

48 + (43 - 1) = 48 + 63 = 111

111 + (53 + 1) = 111 + 126 = 237

55) 13, 17, 33, 97, ?, 1377.

a) 259

b) 563

c) 535

d) 455

e) 353

Ans. e) 353

Number Series Questions Asked in Previous Exams

www.BankExamsToday.com Page 9

56) 6, 3.5, 4.5, 11, 48, ?

a) 96

b) 56

c) 392

d) 192

e) 292

Ans. c) 392

6 x 0.5 + 0.5 = 3.5

3.5 x 1 + 1 = 4.5

4.5 x 2 + 2 = 11

11 x 4 + 4 = 48

48 x 8 + 8 = 392

Directions (57-61) : What should come in place

of the question mark (?) in the following

number series?

57) 8, 4.5, 5.5, 13, 56, ?

a) 566

b) 496

c) 596

d) 450

e) 456

Ans. e) 456

8 x 0.5 + 0.5 = 4.5

4.5 x 1 + 1 = 5.5

5.5 x 2 + 2 = 13

13 x 4 + 4 = 56

56 x 8 + 8 = 456

58) 19, 16, 44, 107, ?

a) 108

b) 156

c) 215

d) 151

e) 251

Ans. c) 215

59) 11, 14, 23, 50, ?

a) 111

b) 121

c) 151

d) 131

e) 141

Ans. d) 131

11 + 31 = 11 + 3 = 14

14 + 32 = 14 + 9 = 23

23 + 33 = 23 + 27 = 50

50 + 34 = 50 + 81 = 131

60) 19, 25, 42, 71, 113, ?

a) 169

b) 153

c) 186

d) 196

e) 269

Ans. a) 169

Number Series Questions Asked in Previous Exams

www.BankExamsToday.com Page 10

61) 21, 35, 30, 44, 39, ?

a) 59

b) 53

c) 55

d) 45

e) 46

Ans. b) 53

21 + 14 = 35

35 – 5 = 30

30 + 14 = 44

44 – 5 = 39

39 + 14 = 53

Directions (62-66): What should come in place

of the question mark (?) in the following

number series?

62) 12, 6.5, 7.5, 17, 72, ?

a) 356

b) 82

c) 185

d) 292

e) 584

Ans. e) 584

12 x 0.5 + 0.5 = 6.5

6.5 x 1 + 1 = 7.5

7.5 x 2 + 2 = 17

17 x 4 + 4 = 72

72 x 8 + 8 = 584

63) 463, 220, 139, 112, ?, 100

a) 101

b) 100

c) 104

d) 103

e) 102

Ans. d) 103

463 – 220 = 243

220 – 139 = 81

139 – 112 = 27

112 – ? = 9

? – 100 = 3

So, ? = 112 – 9 = 103

64) 17, 30, 21, 34, 25, ?

a) 38

b) 30

c) 27

d) 36

e) 40

Ans. a) 38

17 +13 = 30

30 – 9 = 21

21 +13 = 34

34 – 9 = 25

25 + 13 = 38

65) 31, 33, 40, 68, 131, ?

a) 232

b) 257

c) 132

Number Series Questions Asked in Previous Exams

www.BankExamsToday.com Page 11

d) 250

e) 275

Ans. b) 257

31 + (13 + 1) = 31 + 2 = 33

33 + (23 - 1) = 33 + 7 = 40

40 + (33 + 1) = 40 + 28 = 68

68 + (43 - 1) = 68 + 63 = 131

131 + (53 + 1) = 131 + 126 = 257

66) 11, 16, 34, 66, 113, ?

a) 130

b) 160

c) 176

d) 175

e) 177

Ans. c) 176

Directions (67-71): What should come in place

of the question mark (?) in the following

number series?

67) 150, 102, 70, 46, 26, ?

a) 16

b) 8

c) 10

d) 2

e) 4

Ans. b) 8

68) 10, 14, 28, 52, 134, ?

a) 302

b) 268

c) 300

d) 304

e) 208

Ans. d) 304

10*3 - 2= 28

14*4 - 4= 52

28*5 - 6= 134

52*6- 8= 304

69) 4500, 900, 90, 6, ?, 0.012.

a) 0.3

b) 3

c) 3.33

d) 0.33

e) 3.3

Ans. a) 0.3

4500/5 = 900

900/10 = 90

90/15 = 6

6/20 = 0.3

0.3/25 = 0.012

70) 24, 11, 10, 14, 27, ?

a) 66

Number Series Questions Asked in Previous Exams

www.BankExamsToday.com Page 12

b) 70.5

c) 68

d) 66.5

e) 68.5

Ans. d) 66.5

24*(1/2) – 1 = 11

11*(2/2) – 1 = 10

10*(3/2) – 1 = 14

14*(4/2) – 1 = 27

27*(5/2) – 1 = 66.5

71) 8, 7, 12, 33, 128, ?

a) 528

b) 365

c) 653

d) 825

e) 635

Ans. e) 635

8*1 – 1 = 7

7*2 – 2 = 12

12*3 – 3 = 33

33*4 – 4 = 128

128*5 – 5 = 635

Directions (72 -76): In the following number

series only one number is wrong. Find out the

wrong number.

72) 18.3, 20.6, 16, 22.9, 13.7, 2.2, 11.4

a) 25.2

b) 18.3

c) 13.7

d) 22.9

e) 20.6

Ans. a) 25.2

18.3 + 2.3 = 20.6

20.6 - 4.6 = 16

16 + 6.9 = 22.9

22.9 - 9.2 = 13.7

13.7 +11.5 = 25.2

25.2 - 13.8 = 11.4

73) 9, 5, 6, 10.5, 23, 61, 183

a) 183

b) 10.5

c) 61

d) 5

e) 9

Ans. c) 61

9*0.5 + 0.5 = 5

5*1 + 1 = 6

6*1.5 + 1.5 = 10.5

10.5*2 + 2 = 23

23*2.5 + 2.5 = 60

60*3 + 3 = 183

74) 188, 154, 140, 132, 128 , 126, 125

a) 125

b) 154

c) 132

d) 126

e) 188

Ans. e) 188

186 - 36 = 154

154 - 16 = 140

140 - 8 = 132

132 - 4 = 128

128 - 2 = 126

126 - 1 = 125

Number Series Questions Asked in Previous Exams

www.BankExamsToday.com Page 13

75) 2, 4, 11, 37, 151, 771, 4633

a) 11

b) 4633

c) 771

d) 151

e) 2

Ans. d) 151

2*1 + 2 = 4

4*2 + 3 = 11

11*3 + 4 = 37

37*4 + 5 = 153

153*5 + 6 = 771

771*6 + 7 = 4633

76) 391, 394, 399, 411, 431, 461, 503

a) 503

b) 394

c) 399

d) 431

e) 391

Ans. b) 394

391 + 2 = 393

393 + 6 = 399

399 + 12 = 441

441+ 20 = 431

431 + 30 = 461

461 +40 = 503

Directions (77-81): What should come in place

of the question mark (?) in the following

number series?

77) 14, 25, 47, 91, ?, 355

a) 100

b) 197

c) 179

d) 335

e) 155

Ans. c) 179

14, 25, 47, 91, ?, 355

14 * 2 - 3 = 25

25 * 2 - 3 = 47

47 * 2 - 3 = 91

91 * 2 - 3 = 179

78) 11, 24, 44, 70, 101, ?

a) 136

b) 102

c) 80

d) 102

e) 163

Ans. a) 136

11, 24, 44, 70, 101, ?

79) 18, 8, 6, 8, 24, ?

a) 6

b) 48

c) 24

d) 176

e) 167

Number Series Questions Asked in Previous Exams

www.BankExamsToday.com Page 14

Ans. d) 176

18, 8, 6, 8, 24,?

18 * 0.5- 1 = 8

8 * 1- 2 = 6

6* 2 - 4 = 8

8 * 4 - 8 = 24

24*8 - 16 = 176

80) 28, 32, 23, 39, 14, ?

a) 30

b) 50

c) 55

d) 6

e) 14

Ans. b) 50

28, 32, 23, 39, 14, ?

81) 5, 12, 33, 136, 675, ?

a) 5569

b) 4426

c) 5046

d) 4065

e) 4056

Ans. e) 4056

5, 12, 33, 136, 675, ?

5 * 2 + 2 = 12

12 * 3 - 3 = 33

33 * 4 + 4 = 136

136 * 5 - 5 = 675

675 * 6 + 6 = 4056

Directions (82-86): What should come in place

of the question mark (?) in the following

number series?

82) 22, 10, 8, 12, 40, ?

a) 50

b) 48

c) 304

d) 636

e) 512

Ans. c) 304

22 x 0.5 – 1 = 10

10 x 1 (double of 0.5) – 2 (double of 1) = 8

8 x 2 – 4 = 12

12 x 4 – 8 = 40

40 x 8 – 16 = 304

83) 6, 14, 39, 160, 795, ?

a) 1005

b) 3776

c) 2569

d) 56

e) 4776

Ans. e) 4776

6 x 2 + 2 = 14

14 x 3 – 3 = 39

39 x 4 + 4 = 160

160 x 5 – 5 = 795

795 x 6 + 6 = 4776

84) 25, 29, 20, 36, 11, ?

a) 47

Number Series Questions Asked in Previous Exams

www.BankExamsToday.com Page 15

b) 37

c) 30

d) 74

e) 35

Ans. a) 47

25 + 22 = 29

29 - 32 = 20

20 + 42 = 36

36 - 52 = 11

11 + 62 = 11 + 36 = 47

85) 18, 23, 36, 56, 82, ?

a) 103

b) 113

c) 163

d) 96

e) 88

Ans. b) 113

86) 13, 21, 37, 69, ? ,261

a)103

b) 86

c) 134

d) 133

e) 163

Ans. d) 133

13 x 2 - 5 = 21

21 x 2 - 5 = 37

37 x 2 - 5 = 69

69 x 2 - 5 = 133

133 x 2 - 5 = 261

21 x 2 - 5 = 37

37 x 2 - 5 = 69

69 x 2 - 5 = 133

133 x 2 - 5 = 261

Exclusively prepared by Red Team

Chennai: #1, South Usman Road, T Nagar. | Madurai: #24/21, Near Mapillai Vinayagar Theatre, Kalavasal. | Trichy: opp BSNL office, Juman Center, 43 Promenade Road, Cantonment. | Salem: #209, Sonia Plaza / Muthu Complex,

Junction Main Rd, State Bank Colony, Salem. | Coimbatore.

7601808080 / 9043303030 | www.RaceInstitute.in

an ISO 9001: 2008 Certified Institution

Is Now In CHENNAI | MADURAI | TRICHY | SALEM | COIMBATORE

www.RACEInstitute.in

51 Buffet Quantitative Aptitude

Mi?sing Numbers

R.A.C.E Rapid Academy of Competitive Exams

Chennai: #1, South Usman Road, T Nagar. Chennai. Mob: 9043303030 / 7601808080 Madurai: #24/21, Near Mapillai Vinayagar Theatre, Kalavasal. M: 7695814311 | www.RACEInstitute.in

2

Number Series

1. 17 19 23 29 37 ?

(a) 46 (b) 49 (c) 47

(d) 48 (e) 45

2. 900 899 891 864 800 ?

(a) 695 (b) 685 (c) 665

(d) 675 (e) 655

3. 4 32 224 1344 6720 ?

(a) 26885 (b) 26880 (c) 26882

(d) 26888 (e) 26883

4. 56 54 58 50 66 ?

(a) 34 (b) 98 (c) 38

(d) 94 (e) 44

5. 655 637 622 610 601 ?

(a) 598 (b) 593 (c) 595

(d) 597 (e) 594

6. 7 33 137 553 ? 8873

(a) 1217 (b) 2217 (c) 2117

(d) 2127 (e) None of these

7. 8 56 336 2352 14112 ?

(a) 98874 (b) 98784 (c) 87894

(d) 84672 (e) None of these

8. 5963 ? 5938 5913 5877 5828

(a) 5932 (b) 5940 (c) 5951

(d) 5954 (e) None of these

9. 2916 972 ? 108 36 12

(a) 324 (b) 234 (c) 248

(d) 391 (e) None of these

10. 9 15 27 45 69 ?

(a) 109 (b) 99 (c) 91

(d) 81 (e) None of these

11. 5 15 35 75 155 ?

(a) 295 (b) 315 (c) 275

(d) 305 (e) None of these

12. 3 6 18 72 360 ?

(a) 2160 (b) 1800 (c) 2520

(d) 1440 (e) none of these

13. 688 472 347 283 256 ?

(a) 236 (b) 229 (c) 255

(d) 248 (e) None of these

14. 16 8 12 30 105 ?

(a) 387.5 (b) 470.5 (c) 367.5

(d) 472.5 (e) None of these

15. 3 7 19 55 163 ?

(a) 467 (b) 487 (c) 475

(d) 485 (e) None of these

16. 11 16 23 32 43 ?

(a) 55 (b) 57 (c) 58

(d) 59 (e) None of these

17. 1 6 36 240 1960 ?

(a) 19660 (b) 3680 (c) 36800

(d) 19600 (e) None of these

18. 12 14 17 13 8 14 21

13 4 ?

(a) 14 (b) 13 (c) 15

(d) 2 (e) None of these

19. 13 17 25 37 53 73 ?

(a) 101 (b) 105 (c) 94

(d) 96 (e) None of these

20. 15 16 18 21 25 30 ?

(a) 36 (b) 25 (c) 37

(d) 38 (e) None of these

21. 23 33 46 62 81 103 ?

(a) 126 (b) 130 (c) 133

(d) 128 (e) none of these

22. 7 14 25 38 55 74 ?

(a) 97 (b) 99 (c) 107

(d) 113 (e) None of these

23. 9 73 105 121 129 ? 135

(a) 13 (b) 131 (c) 133

(d) 134 (e) None of these

24. What should come in the place of

question – mark (?) in the following series?

3 7 15 ? 63

(a) 19 (b) 27 (c) 31

(d) 35 (e) None of these

25. What should come in the place of

question – mark (?) in the following series?

5 11 23 41 65 95 ?

(a) 131 (b) 125 (c) 137

R.A.C.E Rapid Academy of Competitive Exams

Trichy: opp BSNL office, Juman Center, 43 Promenade Road, Cantonment. Mob: 9360703030 Salem: #209, Muthu Plaza, Junction Main Rd, State Bank Colony, Salem. Mob: 7305958080

3

3

(d) 121 (e) none of these

26. 8 9 12 17 ?

(a) 26 (b) 22 (c) 28

(d) 24 (e) None of these

27. 7 13 25 49 ?

(a) 99 (b) 98 (c) 97

(d) 96 (e) None of these

28. 32 37 47 62 ?

(a) 77 (b) 82 (c) 72

(d) 87 (e) None of these

29. 11 20 38 74 ?

(a) 141 (b) 121 (c) 151

(d) 154 (e) None of these

30. 16 24 36 54 ?

(a) 81 (b) 72 (c) 74

(d) 84 (e) None of these

31. What will come in place of the questions

mark (?) in the following number series?

8 12 24 ? 180 630

(a) 48 (b) 60 (c) 36

(d) 64 (e) None of these

32. What should come in place of the

question mark (?) in the following number

series?

5690 5121 4552 3983 3414 2845 ?

(a) 2276 (b) 2516 (c) 2746

(d) 2356 (e) none of these

33. 7 11 19 35 67 ?

(a) 121 (b) 153 (c) 141

(d) 133 (e) None of these

34. 5 6 10 19 35 ?

(a) 55 (b) 65 (c) 60

(d) 70 (e) None of these

35. 1 3 8 18 35 ?

(a) 61 (b) 72 (c) 67

(d) 52 (e) 71

36. 3 19 115 691 ? 24883

(a) 6923 (b) 4147 (c) 2719

(d) 1463 (e) None of these

37. 5 10 20 ? 80 160

(a) 30 (b) 60 (c) 40

(d) 50 (e) None of these

38. 10 11 14 19 26 ?

(a) 40 (b) 25 (c) 39

(d) 27 (e) none of these

39. 1598 798 398 198 ? 48

(a) 56 (b) 74 (c) 68

(d) 98 (e) None of these

40. 5 10 15 20 25 ?

(a) 35 (b) 40 (c) 25

(d) 30 (e) None of these

41. What should come in place of question

mark (?) in the following series?

16 8 12 30 ?

(a) 95 (b) 96 (c) 84

(d) 87 (e) none of these

42. 7 11 20 36 61 ?

(a) 97 (b) 86 (c) 99

(d) 85 (e) None of these

43. 6 13 22 33 46 ?

(a) 62 (b) 61 (c) 65

(d) 60 (e) None of these

44. 620 428 332 284 260 ?

(a) 246 (b) 258 (c) 262

(d) 240 (e) None of these

45. What should come in place of the

question mark (?) in the following number

series?

56 84 ? 189 283.5 425.25

(a) 168 (b) 132 (c) 136

(d) 126 (e) none of these

46. 9 21 45 81 129 ?

(a) 187 (b) 199 (c) 177

(d) 189 (e) None of these

47. 652 428 316 260 232 ?

(a) 218 (b) 225 (c) 204

(d) 228 (e) None of these

48. 12 16 32 68 132 ?

(a) 196 (b) 232 (c) 276

(d) 213 (e) None of these

R.A.C.E Rapid Academy of Competitive Exams

Chennai: #1, South Usman Road, T Nagar. Chennai. Mob: 9043303030 / 7601808080 Madurai: #24/21, Near Mapillai Vinayagar Theatre, Kalavasal. M: 7695814311 | www.RACEInstitute.in

4

49. A series of numbers is given below.

Which number should come in the place of

the question mark (?)?

6 36 216 1296 ?

(a)1596 (b) 7016 (c) 7676

(d) 8776 (e) None of these

50. What should come in place of the

question mark (?) in the following number

series?

888 888 444 148 ? 7.4

(a) 32 (b) 37 (c) 35

(d) 36 (e) none of these

51. 28 37 64 109 172 ?

(a) 253 (b) 265 (c) 234

(d) 246 (e) None of these

52. 23 439 647 751 803 ?

(a) 864 (b) 819 (c) 855

(d) 825 (e) None of these

53. 13 52 29 68 45 ?

(a) 97 (b) 74 (c) 84

(d) 76 (e) None of these

54. 17 98 147 172 181 ?

(a) 180 (b) 192 (c) 184

(d) 182 (e) None of these

55. 11 19 31 47 67 ?

(a) 80 (b) 81 (c) 86

(d) 96 (e) None of these

56. 748 460 316 244 208 ?

(a) 180 (b) 190 (c) 172

(d) 182 (e) None of these

57. 8 52 ? 1287 4504.5

11261.25 16891.875

(a) 464 (b) 286 (c) 196

(d) 324 (e) none of these

58. 3 42 504 ? 40320

241920 967680

(a) 5848 (b) 3344 (c) 4736

(d) 5040 (e) None of these

59. 403 400 394 382 358 310 ?

(a) 234 (b) 120 (c) 214

(d) 264 (e) None of these

60. 7 8 4 13 -3 22 ?

(a) -7 (b) -10 (c) -12

(d) -14 (e) None of these

61. 250000 62500 12500 3125

625 ? 31.25

(a) 156.25 (b) 172.25 (c) 125

(d) 150 (e) none of these

62.800 400 200 100 50 ?

(a) 20 (b) 30 (c) 25

(d) 35 (e) None of these

63. 2 13 35 68 112 ?

(a) 173 (b) 178 (c) 163

(d) 167 (e) None of these

64. 650 601 565 540 524 ?

(a) 512 (b) 514 (c) 511

(d) 516 (e) None of these

65. 27 28 32 41 57 82 ?

(a) 116 (b) 112 (c) 118

(d) 119 (e) 117

66. 35 34.5 35.5 34 36 33.5 ?

(a) 31 (b) 39 (c) 31.5

(d) 33 (e) None of these

67. 33 38 ? 63 83 108

(a) 47 (b) 46 (c) 43

(d) 48 (e) 49

68. 343 350 336 357 329 ? 322

(a) 354 (b) 364 (c) 368

(d) 362 (e) None of these

69. 1 2 5 10 17 26 ?

(a) 35 (b) 37 (c) 33

(d) 39 (e) None of these

70. 1 2 6 24 120 ?

(a) 740 (b) 724 (c) 720

(d) 728 (e) 720

71. 323 312 299 284 267 ?

(a) 248 (b) 244 (c) 242

(d) 246 (e) 238

72. 18 23 33 48 68 ?

(a) 98 (b) 93 (c) 83

R.A.C.E Rapid Academy of Competitive Exams

Trichy: opp BSNL office, Juman Center, 43 Promenade Road, Cantonment. Mob: 9360703030 Salem: #209, Muthu Plaza, Junction Main Rd, State Bank Colony, Salem. Mob: 7305958080

5

5

(d) 88 (e) 103

73. 783 747 722 706 697 ?

(a) 695 (b) 691 (c) 693

(d) 696 (e) 692

74. 28 27.5 28.5 27 29 ?

(a) 31.5 (b) 28 (c) 27.5

(d) 26 (e) 26.5

75. 5 6 16 57 ? 1245

(a) 244 (b) 148 (c) 296

(d) 271 (e) None of these

76. 12 ? 168 504 1260 2520

(a) 96 (b) 59 (c) 61

(d) 48 (e) None of these

77. 4 9 29 ? 599 3599

(a) 117 (b) 347 (c) 258

(d) 174 (e) None of these

78. 177 170 159 146 ? 110

(a) 132 (b) 106 (c) 129

(d) 127 (e) None of these

79. 2 3 11 38 102 ?

(a) 402 (b) 182 (c) 227

(d) 168 (e) None of these

80. 17 18 22 31 47 ?

(a) 72 (b) 77 (c) 83

(d) 68 (e) None of these

81. 13 14 17 22 29 ?

(a) 33 (b) 35 (c) 36

(d) 38 (e) 37

82. 6 8 4 12 -4 ?

(a) 28 (b) -26 (c) 24

(d) -22 (e) 32

83. 222 198 178 162 150 ?

(a) 148 (b) 142 (c) 144

(d) 150 (e) 146

84. 53 318 1590 6360 19080 ?

(a) 38165 (b) 38170 (c) 38160

(d) 38175 (e) 38106

85. 600 564 539 523 514 ?

(a) 500 (b) 520 (c) 510

(d) 540 (e) 515

86. 55000 11000 22000 440 88 ?

(a)16.4 (b) 18.3 (c) 17.6

(d) 14.5 (e) 12.7

87. 2 12 60 240 720 ?

(a) 1445 (b) 1440 (c) 1420

(d) 1444 (e) 1448

88. 80 48 64 56 60 ?

(a) 57 (b) 68 (c) 54

(d) 62 (e) 58

89. 32 38 50 68 92 ?

(a) 118 (b) 128 (c) 124

(d) 122 (e) 116

90. 2 16 112 672 3360

13440 ?

(a) 3430 (b) 3340 (c) 40320

(d) 43240 (e) None of these

91. 4 9 19 ? 79 159 319

(a) 59 (b) 39 (c) 49

(d) 29 (e) None of these

92. 4000 2000 1000 500 250 125

?

(a) 80 (b) 65 (c) 62.5

(d) 83.5 (e) None of these

93. 588 563 540 519 ? 483

468

(a) 500 (b) 496 (c) 494

(d) 490 (e) None of these

94. 121 ? 81 64 49 36

25

(a) 92 (b) 114 (c) 98

(d) 100 (e) None of these

95. 11 13 16 20 ?

(a) 24 (b) 26 (c) 28

(d) 27 (e) None of these

96. 7 13 25 49 ?

(a) 99 (b) 97 (c) 89

(d) 87 (e) None of these

97. 608 304 152 76 ?

(a) 39 (b) 36 (c) 38

(d) 37 (e) None of these

R.A.C.E Rapid Academy of Competitive Exams

Chennai: #1, South Usman Road, T Nagar. Chennai. Mob: 9043303030 / 7601808080 Madurai: #24/21, Near Mapillai Vinayagar Theatre, Kalavasal. M: 7695814311 | www.RACEInstitute.in

6

98. 8 9 20 63 ?

(a) 256 (b) 252 (c) 246

(d) 242 (e) none of these

99. 5 6 10 19 ?

(a) 28 (b) 37 (c) 36

(d) 35 (e) None of these

100. 2 14 84 420 1680 5040 ?

(a) 8940 (b) 8680 (c) 10080

(d) 5030 (e) None of these

101. 3 5 8 13 21 34 ?

(a) 62 (b) 57 (c) 55

(d) 74 (e) None of these

102. 1 2 6 21 ? 445 2676

(a) 88 (b) 77 (c) 52

(d) 64 (e) None of these

103. 27 125 ? 729 1331 2197

3375

(a) 612 (b) 347 (c) 216

(d) 343 (e) None of these

104. 10400 2600 650 ? 40.625

10.15625 2.5390625

(a) 175.5 (b) 162.5 (c) 154.75

(d) 156.25 (e) None of these

105. 104 109 99 114 94 ?

(a) 69 (b) 124 (c) 120

(d) 78 (e) None of these

106. 980 392 156.8 ? 25.088

10.0352

(a) 65.04 (b) 60.28 (c) 62.72

(d) 63.85 (e) None of these

107. 14 16 35 109 441 ?

(a) 2651 (b) 2205 (c) 2315

(d) 2211 (e) None of these

108. 1 ? 27 64 125

(a) 8 (b) 4 (c) 6

(d) 9 (e) None of these

109. 25 16 ? 4 1

(a) 3 (b) 6 (c) 12

(d) 18 (e) None of these

110. 12 6.5 7.5 12.75 27.5 71.25

?

(a) 225.75 (b) 216.75 (c) 209.75

(d) 236.75 (e) 249.75

111. 16 24 36 54 81

121.5 ?

(a) 182.25 (b) 174.85 (c) 190.65

(d) 166.5 (e) 158.95

112. 12 12 18 45 180 1170 ?

(a) 13485 (b) 14675 (c) 15890

(d) 16756 (e) 12285

113. 22 23 27 36 52 77 ?

(a) 111 (b) 109 (c) 113

(d) 117 (e) 115

114. 16 14 24 66 256 1270 ?

(a) 8564 (b) 5672 (c) 4561

(d) 7608 (e) 6340

115. 7 17 54 ? 1098 6591

(a) 204 (b) 212 (c) 223

(d) 219 (e) None of these

116. 55 109 ? 433 865 1729

(a) 279 (b) 372 (c) 217

(d) 318 (e) None of these

117. 19 38 ? 228 684 1368

(a) 106 (b) 114 (c) 121

(d) 98 (e) None of these

118. 87 89 95 107 ? 157

(a) 127 (b) 122 (c) 139

(d) 140 (e) None of these

119. 474 459 429 369 249 ?

(a) 9 (b) 0 (c) 4

(d) 11 (e) None of these

120. 121 117 108 92 67 ?

(a) 31 (b) 29 (c) 41

(d) 37 (e) None of these

121. 50 26 14 ? 5 3.5

(a) 6 (b) 8 (c) 10

(d) 12 (e) None of these

122. 3 23 43 ? 83 103

(a) 33 (b) 53 (c) 63

(d) 73 (e) None of these

R.A.C.E Rapid Academy of Competitive Exams

Trichy: opp BSNL office, Juman Center, 43 Promenade Road, Cantonment. Mob: 9360703030 Salem: #209, Muthu Plaza, Junction Main Rd, State Bank Colony, Salem. Mob: 7305958080

7

7

123. 748 737 715 682 638 ?

(a) 594 (b) 572 (c) 581

(d) 563 (e) None of these

124. 1 9 25 49 81 ? 169

(a) 100 (b) 64 (c) 81

(d) 121 (e) None of these

125. 1 1 2 6 ? 120

(a) 24 (b) 60 (c) 100

(d) 30 (e) None of these

126. 7 8 16 43 ? 232

(a) 204 (b) 107 (c) 119

(d) 89 (e) None of these

127. 4 13 17 ? 30 39

(a) 29 (b) 21 (c) 26

(d) 19 (e) None of these

128. 982 977 952 827 822 ?

(a) 779 (b) 817 (c) 789

(d) 697 (e) None of these

129. 41472 5184 576 72 8 ?

(a) 0 (b) 9 (c) 1

(d) 8 (e) None of these

130. 7 9 12 16 ?

(a) 22 (b) 19 (c) 20

(d) 21 (e) None of these

131. 384 192 96 48 ?

(a) 36 (b) 28 (c) 34

(d) 32 (e) None of these

132. 5 6 14 45 ?

(a) 183 (b) 185 (c) 138

(d) 139 (e) None of these

133. 8 9 13 22 ?

(a) 30 (b) 31 (c) 34

(d) 36 (e) None of these

134. 6 11 21 41 ?

(a) 81 (b) 61 (c) 71

(d) 91 (e) None of these

135. 4 19 49 94 154 ?

(a) 223 (b) 225 (c) 229

(d) 239 (e) None o these

136. ½ 1 1 ½ 2 2 ½ 3 ?

(a) 3 ½ (b)

(c) 4

(d) 3 ¼ (e) None of these

137. 101 103 99 105 97 ? 95

(a) 93 (b) 104 (c) 108

(d) 107 (e) none of these

138. 3 219 344 408 ? 443 444

(a) 416 (b) 435 (c) 423

(d) 428 (e) None of these

139. 7 10 16 28 52 ? 196

(a) 100 (b) 90 (c) 160

(d) 150 (e) none of these

140. 12 16 24 40 ?

(a) 76 (b) 72 (c) 84

(d) 88 (e) None of these

141. 9 19 39 79 ?

(a) 139 (b) 129 (c) 159

(d) 149 (e) None of these

142. 8 17 42 91 ?

(a) 170 (b) 142 (c) 140

(d) 172 (e) None of these

143. 7 8 18 57 ?

(a) 244 (b) 174 (c) 186

(d) 226 (e) None of these

144. 3840 960 240 60 ?

(a) 20 (b) 18 (c) 12

(d) 22 (e) None of these

145. 353 354 351 356 349 ?

(a) 348 (b) 358 (c) 338

(d) 385 (e) 340

146. 1 5 13 29 ? 125 253

(a) 83 (b) 69 (c) 61

(d) 65 (e) 81

147. 45 57 81 117 165 ?

(a) 235 (b) 215 (c) 205

(d) 245 (e) 225

148. 17 18 26 53 117 ?

458

(a) 342 (b) 142 (c) 257

(d) 262 (e) 242

R.A.C.E Rapid Academy of Competitive Exams

Chennai: #1, South Usman Road, T Nagar. Chennai. Mob: 9043303030 / 7601808080 Madurai: #24/21, Near Mapillai Vinayagar Theatre, Kalavasal. M: 7695814311 | www.RACEInstitute.in

8

149. ¼ ½ ¾ 1 1

1 1¾ ?

(a) 2 (b) 4 (c) 6

(d)

(e)

150. 11 12 26 81 ?

(a) 324 (b) 328 (c) 320

(d) 280 (e) none of these

151. 5120 1280 320 80 ?

(a) 16 (b) 24 (c) 30

(d) 40 (e) None of these

152. 7 11 27 63 ?

(a) 96 (b) 118 (c) 99

(d) 127 (e) None of these

153. 6 10 18 34 ?

(a) 62 (b) 64 (c) 66

(d) 50 (e) None of these

154. 5 11 23 47 ?

(a) 95 (b) 93 (c) 96

(d) 97 (e) None of these

155. 8 39 155 464 ?

(a) 231 (b) 463 (c) 1391

(d) 927 (e) None of these

156. 4 5 14 51 ?

(a) 158 (b) 156 (c) 260

(d) 208 (e) None of these

157. 7 8 17 42 ?

(a) 67 (b) 78 (c) 91

(d) 106 (e) None of these

158. 5 6 15 50 ?

(a) 207 (b) 157 (c) 155

(d) 205 (e) None of these

159. 729 243 81 27 ?

(a) 18 (b) 9 (c) 3

(d) 15 (e) None of these

160. In the following number series, a wrong

number is given. Find out that wrong

number.

2 11 38 197 1172 8227 65806

(a) 11 (b) 38 (c) 197

(d) 1172 (e) 8227

161. 16 19 21 30 46 71

107

(a) 19 (b) 21 (c) 30

(d) 46 (e) 71

162. 7 9 16 25 41 68

107 173

(a) 107 (b) 16 (c) 41

(d) 68 (e) 25

163. 4 2 3.5 7.5 26.25

118.125

(a) 118.125 (b) 26.25 (c) 3.5

(d) 2 (e) 7.5

164. 16 4 2 1.5 1.75 1.875

(a) 1.875 (b) 1.75 (c) 1.5

(d) 2 (e) 4

165. A wrong number is given in the

following series. Find the wrong number.

1 12 65 264 795 1590 1593

(a) 12 (b) 65 (c) 264

(d) 795 (e) 1590

166. A wrong number is given in the

following series. Find the wrong number.

217 216 212 203 187 151 126

(a) 216 (b) 212 (c) 203

(d) 187 (e) 151

167. A wrong number is given in the

following series. Find the wrong number.

11 16 23 29 37 46 56

(a) 16 (b) 23 (c) 29

(d) 37 (e) 46

168. In the following series a wrong number

is given. Find out the one.

15 31 46 63 79 95

(a) 15 (b) 31 (c) 46

(d) 63 (e) 79

169. 0 6 24 60 120 210 ?

(a). 336 (b).349 (c). 312

(d). 337 (e).None of these

170.11 14 19 22 27 30 ?

(a). 39 (b).34 (c).36

R.A.C.E Rapid Academy of Competitive Exams

Trichy: opp BSNL office, Juman Center, 43 Promenade Road, Cantonment. Mob: 9360703030 Salem: #209, Muthu Plaza, Junction Main Rd, State Bank Colony, Salem. Mob: 7305958080

9

9

(d). 35 (e). None of these

171.6 12 21 ? 48

(a). 33 (b).39 (c).36

(d).31 (e). None of these

172. 18 22 30 ? 78 142

(a). 44 (b).35 (c).46

(d). 48 (e). None of these

173.73205 6655 605 55 ?

(a). 9 (b).5 (c).13

(d).11 (e). None of these

174. 25 100 ? 1600 6400

(a).400 (b). 300 (c).360

(d).420 (e). None of these

175. 125 ? 343 512 729 1000

(a). 216 (b).215 (c).256

(d).225 (e). None of these

176. 1 9 125 343 ? 1331

(a). 730 (b).729 (c).512

(d).772 (e). None of these

177. 121 144 169 ? 225

(a). 180 (b).172 (c).186

(d).196 (e). None of these

178. ? 2116 2209

2304 2401 2500

(a). 2124 (b).1972 (c).1521

(d).2025 (e). None of these

179. Look at this series:

22. 1 2 512 27 58 121 ?

What number should come next?

(a). 248 (b).244 (c).198

(d).190 (e). None of these

180. Complete the series,

3 8 13 24 41 ?

(a). 70 (b).66 (c).64 (d).62

(e). None of these

181. Look at this series:.

24. 11 12 17 18 23 24 ?

Which term will come next ?

(a). 27 (b).29 (c).31

(d).33 (e). None of these

182. Insert the missing number.

12 25 49 99 197 395 ?

(a). 789 (b).1579 (c).722

(d).812 (e). None of these

183. Look at this series:.

7 26 63 124 215 342 ?

(a). 481 (b).391 (c).511

(d).421 (e). None of these

184. Look at this series:

1 1 3 9 6 36 10

100 ? 225

(a). 12 (b).13 (c).14

(d).15 (e). None of these

185. Look at this series: 36 34 30

28 24 ? What number should come

next?

(a). 20 (b).22 (c).23

(d).26 (e). None of these

186. Look at this series:

2 -6 18 -54 162 ?

(a). 486 (b).-486 (c).422

(d).-428 (e). None of these

187. Look at this series:

196 144 100 64 36 ?

(a). 16 (b).8 (c).18

(d).12 (e). None of these

188. Look at this series:.

279936 46656 7776 1296 216, ?

(a). 60 (b).46 (c).36

(d).66 (e). None of these

189. 2 7 10 22 18 37 26 ?

(a). 42 (b).52 (c).46

(d).62 (e). None of these

190. 279936 46656 7776 1296

216 ?

(a). 60 (b).46 (c).36

(d).66 (e). None of these

191. 12 38 116 350 1052 ?

(a). 1800 (b). 2200 (c). 2800

(d).3158 (e). None of these

R.A.C.E Rapid Academy of Competitive Exams

Chennai: #1, South Usman Road, T Nagar. Chennai. Mob: 9043303030 / 7601808080 Madurai: #24/21, Near Mapillai Vinayagar Theatre, Kalavasal. M: 7695814311 | www.RACEInstitute.in

10

192. 46080 3840 384 48 8 2

?

(a). 1 (b).1/64 (c).1/8

(d).2 (e). None of these

193. 5 28 57 88 125

(a). 156 (b).147 (c).166

(d).186 (e). None of these

194. 13 18 25 30 37 40

(a). 25 (b).30 (c). 37

(d).40 (e). None of these

195. 15 46 63 71 75 77 78

(a). 15 (b).46 (c).63

(d).71 (e). 75

196. 2 3 12 37 86 166 288

(a). 37 (b).86

(c).166 (d).288 (e). 12

197.15 16 34 105 424

2124 12756

(a). 16 (b).34 (c).105

(d).424 (e). 2124

198. 3 7 15 27 63 127 255

(a). 7 (b).15 (c).27

(d).63 (e).127

199. 42 40 38 35 33 31,

28 ? ?

(a). 25, 22 (b).26, 23 (c).26, 24

(d).25, 23 (e). 26, 22

200. 8 12 9 13 10 14 11 ? ?

(a). 14, 11 (b).15, 12 (c).8, 15

(d).15, 19 (e). 8, 5

ANSWER KEY:

1. (c) 2. (d) 3. (b) 4. (a) 5. (c) 6. (b) 7. (b)

8. (d) 9. (a) 10. (b) 11. (b) 12. (a) 13. (d)

14. (d) 15. (b) 16. (e) 17. (a) 18. (a) 19. (e)

20. (a) 21. (d) 22. (a) 23. (c) 24. (c) 25. (a)

26. (d) 27. (c) 28. (b) 29. (e) 30. (a) 31. (b)

32. (a) 33. (e) 34. (c) 35. (a) 36. (b) 37. (c)

38. (e) 39. (d) 40. (d) 41. (e) 42. (a) 43. (b)

44. (e) 45. (d) 46. (d) 47. (a) 48. (b) 49. (e)

50. (b) 51. (a) 52. (e) 53. (c) 54. (d) 55. (e)

56. (b) 57. (b) 58. (d) 59. (c) 60. (d) 61. (a)

62. (c) 63. (d) 64. (d) 65. (c) 66. (e) 67. (d)

68. (b) 69. (b) 70. (e) 71. (a) 72. (b) 73. (c)

74. (e) 75. (a) 76. (d) 77. (e) 78. (c) 79. (c)

80. (a) 81. (d) 82. (a) 83. (b) 84. (c) 85. (c)

86. (c) 87. (b) 88. (e) 89. (d) 90. (c) 91. (b)

92. (c) 93. (a) 94. (d) 95. (e) 96. (b) 97. (c)

98. (a) 99. (d) 100. (c) 101. (c) 102. (a)

103. (d) 104. (b) 105. (e) 106. (c) 107. (d)

108. (a) 109. (e) 110. (b) 111. (a) 112. (e)

113. (c) 114. (d) 115. (d) 116. (c) 117. (b)

118. (a) 119. (a) 120. (a) 121. (b) 122. (c)

123. (e) 124. (d) 125. (a) 126. (b) 127. (c)

128. (e) 129. (c) 130. (d) 131. (e) 132. (e)

133. (e) 134. (a) 135. (c) 136. (a) 137. (d)

138. (b) 139. (a) 140. (b) 141. (c) 142. (d)

143. (e) 144. (e) 145. (b) 146. (c) 147. (e)

148. (e) 149. (a) 150. (b) 151. (e) 152. (d)

153. (c) 154. (a) 155. (d) 156. (e) 157. (c)

158. (a) 159. (b) 160. (d) 161. (a) 162. (d)

163. (c) 164. (b) 165. (e) 166. (e) 167. (b)

168. (c) 169. (a) 170. (d) 171. (a) 172. (c)

173. (b) 174. (a) 175. (a) 176. (b) 177. (a)

178. (d) 179. (a) 180. (a) 181. (b) 182. (a)

183. (c) 184. (d) 185. (b) 186. (b) 187. (a)

188. (c) 189. (b) 190. (c) 191. (d) 192. (a)

193. (c) 194. (d) 195. (b) 196. (c) 197. (e)

198. (c) 199. (c) 200. (b)

Number SeriesDirections (1-10): In each of these quesitons a number seriesis given, In each series only one number is wrong, Find out thewrong number.1. 5531 5506 5425 5304 5135 4910 4621

(1) 5531 (2) 5425(3) 4621 (4) 5135(5) 5506

2. 6 7 9 13 26 37 69(1) 7 (2) 26(3) 69 (4) 37(5) 9

3. 1 3 10 36 152 760 4632(1) 3 (2) 36(3) 4632 (4) 760(5) 152

4. 4 5 13 40 105 229 445(1) 4 (2) 13(3) 105 (4) 445(5) 229

5. 157.5 45 15 6 3 2 1(1) 1 (2) 2(3) 6 (4) 157.5(5) 45

6. 7 12 40 222 1742 17390 28608(1) 222 (2) 12(3) 40 (4) 1742(5) 2508608

7. 6 91 584 2935 11756 35277 70558(1) 6 (2) 70558(3) 548 (4) 2935(5) 35277

8. 9050 5675 3478 2147 1418 1077 950(1) 950 (2) 1418(3) 5675 (4) 2147(5) 1077

9. 1 4 25 256 3125 46656 823543(1) 4 (2) 823543(3) 46656 (4) 25(5) 256

10. 8424 4212 2106 1051 526.5 263.25 131.625(1) 526.5 (2) 1051

(3) 4212 (4) 8424(5) 263.25

Directions (11-25): What will come in place of the questionmark (?) in the following number series ?11. 8 10 18 44 124 (?)

(1) 344 (2) 366(3) 354 (4) 356(5) None of these

12. 13 25 61 121 205 (?)(1) 323 (2) 326(3) 324 (4) 313(5) None of these

13. 656 352 200 124 86 (?)(1) 67 (2) 59(3) 62 (4) 57(5) None of these

14. 454 472 445 463 436 (?)(1) 436 (2) 456(3) 454 (4) 434(5) None of these

15. 12 18 36 102 360 (?)(1) 1364 (2) 1386(3) 1384 (4) 1376(5) None of these

16. 1 7 49 343 (?)(1) 16807 (2) 1227(3) 2058 (4) 2401(5) None of these

17. 13 20 39 78 145 (?)(1) 234 (2) 244(3) 236 (4) 248(5) None of these

18. 12 35 81 173 357 (?)(1) 725 (2) 715(3) 726 (4) 736(5) None of these

19. 3 100 297 594 991 (?)(1) 1489 (2) 1479(3) 1478 (4) 1498(5) None of these

2

CAREER POWER

CAREER POWER, G-7, Roots Tower, Laxmi Nagar, District Centre, Delhi-92, www.careerpower.in

Chapterwise Bank PO Quantitative Aptitude

20. 112 119 140 175 224 (?)(1) 277 (2) 276(3) 287 (4) 266(5) None of these

21. 28 39 63 102 158 (?)(1) 232 (2) 242(3) 233 (4) 244(5) None of these

22. 7 16 141 190 919 (?)(1) 1029 (2) 1019(3) 1020 (4) 1030(5) None of these

23. 12 17 32 57 92 (?)(1) 198 (2) 195(3) 137 (4) 205(5) None of these

24. 19 25 45 87 159 (?)(1) 254 (2) 279(3) 284 (4) 269(5) None of these

25. 83 124 206 370 698 (?)(1) 1344 (2) 1324(3) 1364 (4) 1334(5) None of these

Directions (26-40): In each of these quesitons a number seriesis given, In each series only one number is wrong, Find out thewrong number.26. 3601 3602 1803 604 154 36 12

(1) 3602 (2) 1803(3) 604 (4) 154(5) 36

27. 4 12 42 196 1005 6066 42511(1) 12 (2) 42(3) 1005 (4) 196(5) 6066

28. 2 8 12 20 30 42 56(1) 8 (2) 42(3) 30 (4) 20(5) 12

29. 32 16 24 65 210 945 5197.5(1) 945 (2) 16(3) 24 (4) 210(5) 65

30. 7 13 25 49 97 194 385(1) 13 (2) 49(3) 97 (4) 194(5) 25

31. 4 3 4.5 8.5 20 53 162.5(1) 3 (2) 4.5(3) 8.5 (4) 20(5) 53

32. 12000 2395 472 89.8 12.96 –2.408 –5.4816(1) –5.4816 (2) 472(3) 12.96 (4) –2.408(5) 2395

33. 1 8 28 99 412 2075 12460(1) 28 (2) 99(3) 412 (4) 2075(5) 12460

34. 144 215 540 1890 8505 46777.5 304053.75(1) 215 (2) 540(3) 1890 (4) 8505(5) 46777.5

35. 2222 1879 1663 1538 1474 1447 1440(1) 1879 (2) 1538(3) 1474 (4) 1447(5) 1440

36. 1050 510 242 106 46 16 3(1) 3 (2) 106(3) 242 (4) 510(5) None of these

37. 550 546 537 521 494 460 411(1) 494 (2) 546(3) 521 (4) 460(5) None of these

38. 8 21 47 86 140 203 281(1) 47 (2) 86(3) 140 (4) 203(5) None of these

39. 4 24 161 965 4795 19176 57525(1) 161 (2) 965(3) 57525 (4) 19176(5) None of these

40. 1 2 8 24 120 720 5040(1) 120 (2) 24(3) 8 (4) 720(5) None of these

3

CAREER POWER

CAREER POWER, G-7, Roots Tower, Laxmi Nagar, District Centre, Delhi-92, www.careerpower.in

Chapterwise Bank PO Quantitative Aptitude

1. (1) The pattern is :5531 – 5506 = 25 = 52

5555 – 5506 = 49 = 72

5506 – 5425 = 81 = 92

5425 – 5304 = 121 = 112

5304 – 5135 = 169 = 132

5135 – 4910 = 225 = 152

4910 – 4621 = 289 = 172

Clearly, 5531 is wrong which should be subtituted by5555.

2. The pattern is :6 + 1 = 77 + 1 × 2 = 99 + 2 × 2 = 13

13 + 8 = 21 ¹ 26

21 + 16 = 3737 + 32 = 69

3. (4) The pattern is :1 × 1 + 2 = 33 × 2 + 4 = 1010 × 3 + 6 = 3636 × 4 + 8 = 152

152 × 5 + 10 = 770 ¹ 760

770 × 6 + 12 = 46324. (3) The pattern is :

4 + 13 = 55 + 23 = 1313 + 33 = 40

40 + 43 = 104 ¹ 105

104 + 53 = 229229 + 63= 445

5. (1) The pattern is :157.5 ÷ 3.5 = 4545 ÷ 3 = 1515 ÷ 2.5 = 66 ÷ 2 = 33 ÷ 1.5 = 22 ÷ 2 = ¹ 1

6. (4) The pattern of the number series is :7 × 2 – 2 = 1212 × 4 – (2 + 6) = 48 – 8 = 4040 × 6 – (8 + 10) = 240 – 18 = 222222 × 8 – (18 + 14 ) = 1776 – 32 = 1744 ¹ 17421744 × 10 – (32 + 18) = 17440 – 50 = 17390

7. (3) The pattern of the number series is :6 × 7 × 72 = 45 + 49 = 9191 × 6 + 62 = 546 + 36 = 582 ¹ 584582 × 5 + 52 = 2910 + 25 = 29352935 × 4 + 42 = 11740 + 16 = 1175611756 × 3 + 32 = 35268 + 9 = 35277

8. (5) The pattern of the number series is :9050 – 153 = 9050 – 3375 = 56755675 – 133 = 5675 – 2197 = 34783478 – 113 = 3478 – 1331 = 21472147 – 93 = 2147 – 729 = 14181418 – 73 = A1418 – 343 = 1075 ¹ 1077

9. (4) The pattern of the number series is :1 = 112 = 433 = 27 ¹ 2544 = 25655 = 312566 = 46656

HINTS & SOLUTIONS

.ANSWER KEY.1. (1) 2. (2) 3. (4) 4. (3) 5. (1) 6. (4)

7. (3) 8. (5) 9. (4) 10. (2) 11. (2) 12. (4)

13. (1) 14. (3) 15. (2) 16. (4) 17. (4) 18. (1)

19. (5) 20. (3) 21. (3) 22. (5) 23. (3) 24. (4)

25. (5) 26. (4) 27. (2) 28. (1) 29. (5) 30. (4)

31. (3) 32. (2) 33. (5) 34. (1) 35. (5) 36. (2)

37. (1) 38. (3) 39. (2) 40. (3)

4

CAREER POWER

CAREER POWER, G-7, Roots Tower, Laxmi Nagar, District Centre, Delhi-92, www.careerpower.in

Chapterwise Bank PO Quantitative Aptitude

10. (2) The pattern of the number series is :8424 ÷ 2 = 42124212 ÷ 2 = 2106

2106 ÷ 2 = 1053 ¹ 1051

1053 ÷ 2 = 526.5526.5 ÷ 2 = 263.25

11. (2) The pattern of the number series is :8 + 2 = 1010 + 8 (= 2 × 3 + 2) = 1818 + 26 (= 3 × 8 + 2) = 4444 + 80 (= 3 × 26 + 2) = 124

124 + 242 (= 3 × 80 + 2) = 366

12. (4) The pattern of the number series is :13 + 1 × 12 = 13 + 12 = 2525 + 3 × 12 = 25 + 36 = 6161 + 5 × 12 = 61 + 60 = 121121 + 7 × 12 = 121 + 84 = 205

205 + 9 × 12 = 205 + 108 = 313

13. (1) The pattern of the number series is :

6562

+ 24 = 328 + 24 = 352

3522

+ 24 = 176 + 24 = 200

2002

+ 24 = 100 + 24 = 124

1242

+ 24 = 62 + 24 = 86

862

+ 24 = 43 + 24 = 67

14. (3) The pattern of the number series is :454 + 18 = 472472 – 27 = 445445 + 18 = 463463 – 27 = 436

436 + 18 = 454

15. (2) The pattern of the number series is :12 × 4 – 30 = 48 – 30 = 1818 × 4 – 36 = 72 – 36 = 3636 × 4 – 42 = 144 – 42 = 102102 × 4 – 48 = 408 – 48 = 380

360 × 4 – 54 = 1440 – 54 = 1386

16. (5) The pattern of the number series is :1 × 7 = 77 × 7 = 4949 × 7 = 343

343 × 7 = 2401

17. (4) The pattern of the number series is :13 + 7 = 2020 + 19 (= 7 + 12) = 3939 + 39 (= 19 + 20) = 7878 + 67 (= 39 + 28) = 145

145 + 103 (= 67 + 36) = 248

18. (1) The pattern of the number series is :12 + 1 × 23 = 3535 + 2 × 23 = 35 + 46 = 8181 + 2 × 46 = 81 + 92 = 173173 + 2 × 92 = 173 + 184 = 357

357 + 2 × 184 = 357 + 368 = 725

19. (5) The pattern of the number series is :3 + 97 = 100100 + 197 = 297297 + 297 = 594594 + 397 = 991

991 + 497 = 1488

20. (3) The pattern of the number series is :112 + 1 × 7 = 119119 + 3 × 7 = 119 + 21 = 140140 + 5 × 7 = 140 + 35 = 175175 + 7 × 7 = 175 + 49 = 224

224 + 9 × 7 = 224 + 63 = 287

21. (3) The pattern of the number series is :28 + 11 = 3939 + 24 = (= 11 + 13) = 6363 + 39 (= 24 + 15) = 102102 + 56 (= 39 + 17) = 158

158 + 75 (= 46 + 19) = 233

22. (5) The pattern of the number series is :7 + 32 = 7 + 9 = 1616 + 53 = 16 + 125 = 141141 + 72 = 141 + 49 = 190190 + 93 = 190 + 729 = 919

919 + 112 = 919 + 121 = 1040

5

CAREER POWER

CAREER POWER, G-7, Roots Tower, Laxmi Nagar, District Centre, Delhi-92, www.careerpower.in

Chapterwise Bank PO Quantitative Aptitude

23. (3) The pattern of the number series is :12 + 5 × 1 = 1717 + 5 × 3 = 3232 + 5 × 5 = 5757 + 5 × 7 = 92

92 + 5 × 7 = 137

24. (4) The pattern of the number series is :19 + 2 × 3 = 19 + 6 = 2525 + 4 × 5 = 25 + 20 = 4545 + 6 × 7 = 45 + 42 = 8787 + 8 × 9 = 87 + 72 = 159

159 + 10 × 11 = 159 + 110 = 269

25. (5) The pattern of the number series is :83 + 41 × 1 =124124 + 41 × 2 = 124 + 82 = 206206 + 41 × 4 = 206 + 164 = 370370 + 41 × 8 = 370 + 328 = 698

698 + 41 × 16 = 698 + 656 = 1354

26. (4) The pattern of the number series is :3601 ÷ 2 + 1 = 36023602 ÷ 2 + 2 = 1801 + 2 = 18031803 ÷ 3 + 3 = 601 + 3 = 604

604 ÷ 4 + 4 = 151 + 4 = 155 ¹ 154

155 ÷ 5 + 5 = 31 ÷ 5 = 3636 ÷ 6 + 6 = 6 + 6 = 12

27. (1) The pattern of the number series is :4 × 2 + 22 = 8 + 4 = 12

12 × 3 + 32 = 36 + 9 = 45 ¹ 42

45 × 4 + 42 = 180 + 16 = 196196 × 5 + 52 = 980 = + 25 = 10051005 × 6 + 62 = 6030 + 36 = 6066

28. (1) The pattern of the number series is :

2 + 4 = 6 ¹ 8

6 + 6 = 1212 + 8 = 2020 + 10 = 3030 + 12 = 42

29. (5) The pattern of the number series is :

32 × 12

= 16

16 × 32

= 24

24 × 52

= 60 ¹ 65

60 × 72

= 210

210 × 92

= 945

945 × 112

= 5197.5

30. (4) The pattern of the number series is :7 × 2 – 1 = 14 – 1 = 1313 × 2 – 1 = 26 – 1 = 2525 × 2 – 1 = 50 – 1 = 4949 × 2 – 1 = 98 – 1 = 97

97 × 2 – 1 = 194 – 1 = 193 ¹ 194

31. (3) The pattern of the number series is :4 × 0.5 + 1 = 2 + 1 = 33 × 1 + 1.5 = 3 + 1.5 = 4.54.5 × 1.5 + 2 = 6.75 + 2

= 8.75 ¹ 8.5

8.75 × 2 + 2.5 = 17.5 + 2.5 = 2020 × 2.5 + 3 = 50 + 3 = 53

32. (2) The pattern of the number series is :12000 ÷ 5 – 5 = 2400 – 5 = 23952395 ÷ 5 – 5 = 479 – 5

= 474 ¹ 472

474 ÷ 5 – 5 = 94.8 – 5 = 89.889.8 ÷ 5 – 5 = 17.96 – 5 = 12.96

33. (5) The pattern of the number series is :1 × 1 + 7 × 1 = 1 + 7 = 88 × 2 + 6 × 2 =16 + 12 = 2828 × 3 + 5 × 3 = 84 + 15 = 9999 × 4 + 4 × 4 = 396 + 16 = 412412 × 5 + 3 × 5 = 2060 + 15 = 20752075 × 6 + 2 × 6 = 12450 + 12

= 12462 ¹ 12460

34. (1) The pattern of the number series is :

144 × 1.5 = 216 ¹ 215

216 × 2.5 = 540540 × 3.5 = 18901890 × 4.5 = 85058505 × 5.5 = 46777.5

6

CAREER POWER

CAREER POWER, G-7, Roots Tower, Laxmi Nagar, District Centre, Delhi-92, www.careerpower.in

Chapterwise Bank PO Quantitative Aptitude

35. (5) The pattern of the number series is :2222 – 73 = 2222 – 343 = 18791879 – 63 = 1879 – 216 = 16631663 – 53 = 1663 – 125 = 15381538 – 43 = 1538 – 64 = 14741474 – 33 = 1474 – 27 = 1447

1447 – 23 = 1447 – 8 = 1439 ¹ 1440

36. (2) The pattern is :

1050 302-

= 510 :

510 262-

= 242

242 222-

= 110 ¹ 106

110 182-

= 46

46 142-

= 16

37. (1) The pattern is :550 – 22 = 550 – 4 = 546546 – 32 = 546 – 9 = 537

537 – 42 = 537 – 16 = 521

521 – 52 = 521 – 25 = 496 ¹ 494

496 – 62 = 496 – 36 = 46038. (3) The pattern is :

8 + 1 × 13 = 2121 + 2 × 13 = 21 + 26 = 4747 + 3 × 13 = 47 + 39 = 86

86 + 4 × 13 = 86 + 52 = 138 ¹ 140

138 + 5 × 13 = 138 + 65 = 203203 + 6 × 13 = 203 + 78 = 281

39. (2) The pattern is :4 × 8 – 8 = 32 – 8 = 2424 × 7 – 7 = 168 – 7 = 161

161 × 6 – 6 = 966 – 6 = 960 ¹ 965

960 × 5 – 5 = 4800 – 5 = 479540. (3) The pattern is :

1 × 2 = 2

2 × 3 = 6 ¹ 8

6 × 4 = 2424 × 5 = 120120 × 6 = 720